MedEd Flashcards

1
Q

A 65 year old lady presents with diarrhoea. She has struggled with infrequent bowel motions for most of her life so is surprised by this.

A. Gastroenteritis
B. UC
C. Crohn's Disease
D. Colorectal Carcinoma
E. Thyrotoxicosis
F. Chronic Pancreatitis
G. Pregnancy
H. Overflow Diarrhoea
I. Ischaemic Colitis
J. Lactose Intolerance
K. Laxative Abuse
L. Cimetidine Use
A

Overflow diarrhoea: History of chronic constipation/obstruction. Liquid stool passes obstruction.

Key words: Infrequent bowel motion

How well did you know this?
1
Not at all
2
3
4
5
Perfectly
2
Q

An 18 year old girl presents with diarrhoea. She reports no other changes other than starting the OCP. She has had similar episodes in the past which she manages with lifestyle modification but doesn’t understand why this is happening now.

A. Gastroenteritis
B. UC
C. Crohn's Disease
D. Colorectal Carcinoma
E. Thyrotoxicosis
F. Chronic Pancreatitis
G. Pregnancy
H. Overflow Diarrhoea
I. Ischaemic Colitis
J. Lactose Intolerance
K. Laxative Abuse
L. Cimetidine Use
A

Lactose Intolerance:
- Hypersensitivity= reproducible adverse reaction (includes allergy and intolerance)
Intolerance is non-immune mediated
Allergy is either IgE or non-IgE mediated

Key Words: OCP, Lifestyle modifications.

OCP and POP both contain lactose

How well did you know this?
1
Not at all
2
3
4
5
Perfectly
3
Q

A 62 year old man presents with bloody diarrhoea and intense abdominal pain. He is currently on therapy for his heart palpitations.

A. Gastroenteritis
B. UC
C. Crohn's Disease
D. Colorectal Carcinoma
E. Thyrotoxicosis
F. Chronic Pancreatitis
G. Pregnancy
H. Overflow Diarrhoea
I. Ischaemic Colitis
J. Lactose Intolerance
K. Laxative Abuse
L. Cimetidine Use
A

Ischaemic Colitis: Bloody, abdo pain, palpitations

How well did you know this?
1
Not at all
2
3
4
5
Perfectly
4
Q

A 45 year old woman presents with diarrhoea. She also reports weight loss but states that her appetite has increased. You notice that her hands shake slightly when she is reaching for things around the room.

A. Gastroenteritis
B. UC
C. Crohn's Disease
D. Colorectal Carcinoma
E. Thyrotoxicosis
F. Chronic Pancreatitis
G. Pregnancy
H. Overflow Diarrhoea
I. Ischaemic Colitis
J. Lactose Intolerance
K. Laxative Abuse
L. Cimetidine Use
A

Thyrotoxicosis: Weight loss, appetite increase, fine tremor, heat intolerance, sweating, palpitations, eye signs & pretibial myxoedema (Graves’ Disease)

How well did you know this?
1
Not at all
2
3
4
5
Perfectly
5
Q

A 24 year old man presents with diarrhoea. He is normally fit and well and complains of nothing else. He has not eaten anything dodgy recently, but returned from Malaysia 2 weeks ago.

A. Gastroenteritis
B. UC
C. Crohn's Disease
D. Colorectal Carcinoma
E. Thyrotoxicosis
F. Chronic Pancreatitis
G. Pregnancy
H. Overflow Diarrhoea
I. Ischaemic Colitis
J. Lactose Intolerance
K. Laxative Abuse
L. Cimetidine Use
A

Gastroenteritis: look for travel, dietary changes and contact history

How well did you know this?
1
Not at all
2
3
4
5
Perfectly
6
Q

A 56 year old man presents with constipation. He also reports abdominal pain, weight loss and suffers with depression. He has no FH of note, but has had some problems with kidney stones.

A. Aganglionosis
B. Dehydration
C. Colorectal Carcinoma
D. Hypokalaemia
E. Environmental
F. Furosemide use
G. Hypothyroidism
H. Fibroid
I. Diabetes
J. Anal fissure
K. Opiate use
L. Hypercalcaemia
A

Hypercalcaemia: “bones, stones, groan, psychic moans”

Abdo pain, vomiting, constipation, depression, FH kidney stones, polyuria, polydipsia, anorexia, pyrexia, fatigue, hypertension, confusion, ectopic calcification

Due to: Malignancy, primary hyperparathyroidism, sarcoidosis

How well did you know this?
1
Not at all
2
3
4
5
Perfectly
7
Q

A 26 year old woman presents with constipation. She has struggled with this for a few years but her GPs advice of topical GTN and dietary advice is not helping.

A. Aganglionosis
B. Dehydration
C. Colorectal Carcinoma
D. Hypokalaemia
E. Environmental
F. Furosemide use
G. Hypothyroidism
H. Fibroid
I. Diabetes
J. Anal fissure
K. Opiate use
L. Hypercalcaemia
A

Anal Fissue: Causes constipation due to pain

Tx: high fibre and fluid diet, topical lidocaine, topical GTN (SE - headaches), botox, surgical

How well did you know this?
1
Not at all
2
3
4
5
Perfectly
8
Q

A 70 year old woman presents with constipation. She has just been discharged from hospital following a #NOF. On admission she was found to be on a number of drugs for her heart which have not changed upon discharge.

A. Aganglionosis
B. Dehydration
C. Colorectal Carcinoma
D. Hypokalaemia
E. Environmental
F. Furosemide use
G. Hypothyroidism
H. Fibroid
I. Diabetes
J. Anal fissure
K. Opiate use
L. Hypercalcaemia
A

Opiate use: commonly prescribed for analgesia, also used as an anti-tussive in heart failure to provide patients symptomatic relief (desensitises central chemoreceptors to the response to PaCO2).

Acts on receptors on myenteric and submucosal plexus to decrease gastric motility and emptying, and increase gut water reabsorption = Constipation

Also lead to pupillary constriction (edinger Westphal nucleus), nausea&vomiting (CTZ), and priuritis, uricaria&hypotension (histamine release from mast cells)

Euphoric effects via action of dopamine release on nucleus accumbens

How well did you know this?
1
Not at all
2
3
4
5
Perfectly
9
Q

A 75 year old man presents with constipation. He also reports abdominal pain and weight loss.

A. Aganglionosis
B. Dehydration
C. Colorectal Carcinoma
D. Hypokalaemia
E. Environmental
F. Furosemide use
G. Hypothyroidism
H. Fibroid
I. Diabetes
J. Anal fissure
K. Opiate use
L. Hypercalcaemia
A

Colorectal carcinoma: over 50% of presentations in >70 years

Tenesmus, blood PR, weight loss

L sided = 25% can be obstructed

Tx: surgical excision - Upper third = anterior resection (colo-anal anastomoses)
Lower 2/3’s = abdominoperineal resection (permanent colostomy - increased incidence sexual and urinary dysfunction

How well did you know this?
1
Not at all
2
3
4
5
Perfectly
10
Q

An 8 year old boy presents with constipation. His abdomen is also slightly distended. He is normally fir and his diet is balanced and healthy.

A. Aganglionosis
B. Dehydration
C. Colorectal Carcinoma
D. Hypokalaemia
E. Environmental
F. Furosemide use
G. Hypothyroidism
H. Fibroid
I. Diabetes
J. Anal fissure
K. Opiate use
L. Hypercalcaemia
A

Aganglionosis: e.g. Hirchsprung’s disease
Absence of ganglion cells in the GIT therefore there is impaired/absent nervous control.
Usually diagnosed as infant

How well did you know this?
1
Not at all
2
3
4
5
Perfectly
11
Q
Hepatitis B Virus
sAg negative
eAg negative
eAb negative
cAb positive
sAg = surface antigen
eAg = e antigen
eAb = e antibody
cAb = core antibody
A. Acute Infection
B. Resolved Infection
C. Naive
D. Low infectivity carrier
E. High Infectivity carrier
F. Vaccinated
A

Resolved Infection

Hepatitis B: double stranded DNA virus, transmitted percutaneously and permucosally.

Signs: Fever, malaise, nausea, jaundice, hepatomegaly, arthralgia, urticaria, deranged LFTs

Long term consequences: fibrosis -> cirrhosis -> hepatocellular carcinoma

sAg = appears 2-10 weeks after exposure and is undetectable after 4-6 months in self limiting acute cases. Persistence >6 months indicates chronic infection.

sAb = appears several weeks after sAg disappears. Indicates a resolved infection and life long immunity (vaccine).

eAg = soluble viral protein in serum, part of early infection and disappears soon after peak ALT levels. Presence >3months indicates chronic infection is likely.

cAb = appears within weeks of acute infection and remains detectable for 4-8 months. May be only way to diagnose acute infection when sAg has disappeared and sAb hasn’t yet appeared.

eAg and sAg indicates a greater infectivity, high level of viral activity and replication.

How well did you know this?
1
Not at all
2
3
4
5
Perfectly
12
Q
Hepatitis B Virus
sAg positive
eAg negative/positive
eAb negative/positive
cAb positive
sAg = surface antigen
eAg = e antigen
eAb = e antibody
cAb = core antibody
A. Acute Infection
B. Resolved Infection
C. Naive
D. Low infectivity carrier
E. High Infectivity carrier
F. Vaccinated
A

Acute Infection

Hepatitis B: double stranded DNA virus, transmitted percutaneously and permucosally.

Signs: Fever, malaise, nausea, jaundice, hepatomegaly, arthralgia, urticaria, deranged LFTs

Long term consequences: fibrosis -> cirrhosis -> hepatocellular carcinoma

sAg = appears 2-10 weeks after exposure and is undetectable after 4-6 months in self limiting acute cases. Persistence >6 months indicates chronic infection.

sAb = appears several weeks after sAg disappears. Indicates a resolved infection and life long immunity (vaccine).

eAg = soluble viral protein in serum, part of early infection and disappears soon after peak ALT levels. Presence >3months indicates chronic infection is likely.

cAb = appears within weeks of acute infection and remains detectable for 4-8 months. May be only way to diagnose acute infection when sAg has disappeared and sAb hasn’t yet appeared.

eAg and sAg indicates a greater infectivity, high level of viral activity and replication.

How well did you know this?
1
Not at all
2
3
4
5
Perfectly
13
Q
Hepatitis B Virus
sAb positive
eAg negative
eAb negative
cAb negative
sAb = surface antibody
eAg = e antigen
eAb = e antibody
cAb = core antibody
A. Acute Infection
B. Resolved Infection
C. Naive
D. Low infectivity carrier
E. High Infectivity carrier
F. Vaccinated
A

Vaccinated

Hepatitis B: double stranded DNA virus, transmitted percutaneously and permucosally.

Signs: Fever, malaise, nausea, jaundice, hepatomegaly, arthralgia, urticaria, deranged LFTs

Long term consequences: fibrosis -> cirrhosis -> hepatocellular carcinoma

sAg = appears 2-10 weeks after exposure and is undetectable after 4-6 months in self limiting acute cases. Persistence >6 months indicates chronic infection.

sAb = appears several weeks after sAg disappears. Indicates a resolved infection and life long immunity (vaccine).

eAg = soluble viral protein in serum, part of early infection and disappears soon after peak ALT levels. Presence >3months indicates chronic infection is likely.

cAb = appears within weeks of acute infection and remains detectable for 4-8 months. May be only way to diagnose acute infection when sAg has disappeared and sAb hasn’t yet appeared.

eAg and sAg indicates a greater infectivity, high level of viral activity and replication.

How well did you know this?
1
Not at all
2
3
4
5
Perfectly
14
Q
Hepatitis B Virus
sAg positive
eAg negative
eAb positive
cAb positive
sAg = surface antigen
eAg = e antigen
eAb = e antibody
cAb = core antibody
A. Acute Infection
B. Resolved Infection
C. Naive
D. Low infectivity carrier
E. High Infectivity carrier
F. Vaccinated
A

Low Infectivity Carrier

Hepatitis B: double stranded DNA virus, transmitted percutaneously and permucosally.

Signs: Fever, malaise, nausea, jaundice, hepatomegaly, arthralgia, urticaria, deranged LFTs

Long term consequences: fibrosis -> cirrhosis -> hepatocellular carcinoma

sAg = appears 2-10 weeks after exposure and is undetectable after 4-6 months in self limiting acute cases. Persistence >6 months indicates chronic infection.

sAb = appears several weeks after sAg disappears. Indicates a resolved infection and life long immunity (vaccine).

eAg = soluble viral protein in serum, part of early infection and disappears soon after peak ALT levels. Presence >3months indicates chronic infection is likely.

cAb = appears within weeks of acute infection and remains detectable for 4-8 months. May be only way to diagnose acute infection when sAg has disappeared and sAb hasn’t yet appeared.

eAg and sAg indicates a greater infectivity, high level of viral activity and replication.

How well did you know this?
1
Not at all
2
3
4
5
Perfectly
15
Q
Hepatitis B Virus
sAg negative
eAg negative
eAb negative
cAb negative
sAg = surface antigen
eAg = e antigen
eAb = e antibody
cAb = core antibody
A. Acute Infection
B. Resolved Infection
C. Naive
D. Low infectivity carrier
E. High Infectivity carrier
F. Vaccinated
A

Naive

Hepatitis B: double stranded DNA virus, transmitted percutaneously and permucosally.

Signs: Fever, malaise, nausea, jaundice, hepatomegaly, arthralgia, urticaria, deranged LFTs

Long term consequences: fibrosis -> cirrhosis -> hepatocellular carcinoma

sAg = appears 2-10 weeks after exposure and is undetectable after 4-6 months in self limiting acute cases. Persistence >6 months indicates chronic infection.

sAb = appears several weeks after sAg disappears. Indicates a resolved infection and life long immunity (vaccine).

eAg = soluble viral protein in serum, part of early infection and disappears soon after peak ALT levels. Presence >3months indicates chronic infection is likely.

cAb = appears within weeks of acute infection and remains detectable for 4-8 months. May be only way to diagnose acute infection when sAg has disappeared and sAb hasn’t yet appeared.

eAg and sAg indicates a greater infectivity, high level of viral activity and replication.

How well did you know this?
1
Not at all
2
3
4
5
Perfectly
16
Q

Drug Side Effects:

Isoniazid

A. Arrhythmias
B. Ototoxicity
C. Increased appetite
D. Hepatic enzyme inducer
E. Cushing's syndrome
F. Hepatic enzyme reducer
G. Peripheral neuropathy
H. Liver toxicity
I. OCP failure
J. Colour Blindness
A

Peripheral neuropathy

Used in treatment TB.
Prescribed with pyridoxine prophylactically.

How well did you know this?
1
Not at all
2
3
4
5
Perfectly
17
Q

Drug Side Effects:

Ethambutol

A. Arrhythmias
B. Ototoxicity
C. Increased appetite
D. Hepatic enzyme inducer
E. Cushing's syndrome
F. Hepatic enzyme reducer
G. Peripheral neuropathy
H. Liver toxicity
I. OCP failure
J. Colour Blindness
A

Colour Blindness - optic neuritis, colour vision is effected first

TB treatment

How well did you know this?
1
Not at all
2
3
4
5
Perfectly
18
Q

Drug Side Effects:

Rifampicin

A. Arrhythmias
B. Ototoxicity
C. Increased appetite
D. Hepatic enzyme inducer
E. Cushing's syndrome
F. Hepatic enzyme reducer
G. Peripheral neuropathy
H. Liver toxicity
I. OCP failure
J. Colour Blindness
A

Hepatic Enzyme Inducer

  • Inactivates oral contraceptive, causes orange discolouration of bodily fluids, hepatitis
How well did you know this?
1
Not at all
2
3
4
5
Perfectly
19
Q

Drug Side Effects:

Pyrazinamide

A. Arrhythmias
B. Ototoxicity
C. Increased appetite
D. Hepatic enzyme inducer
E. Cushing's syndrome
F. Hepatic enzyme reducer
G. Peripheral neuropathy
H. Liver toxicity
I. OCP failure
J. Colour Blindness
A

Liver toxicity

Also arthralgia

How well did you know this?
1
Not at all
2
3
4
5
Perfectly
20
Q

Drug Side Effects:

Streptomycin

A. Arrhythmias
B. Ototoxicity
C. Increased appetite
D. Hepatic enzyme inducer
E. Cushing's syndrome
F. Hepatic enzyme reducer
G. Peripheral neuropathy
H. Liver toxicity
I. OCP failure
J. Colour Blindness
A

Ototoxicity

How well did you know this?
1
Not at all
2
3
4
5
Perfectly
21
Q

A 23 year old man comes into A&E with abdominal pain which started over his belly button and now is in his RLQ. he is tachycardic, has a tender RIF on palpation and is Rovsing’s sign positive.

A. OGD
B. Colonoscopy
C. Barium Swallow
D. Flexi-Sig
E. Rigid Sig
F. CT KUB
G. CT CAP
H. USS
I. Abdo Xray
J. Proctoscopy
K. Barium Enema
L. None of the above
A

None of the above!

Appendicitis is a clinical diagnosis

Inflammation of the vermiform appendix due to faecolith, lymphoid hyperplasia, infection (parasitic/viral)

Tender over McBurney’s point and Rovsing positive.

How well did you know this?
1
Not at all
2
3
4
5
Perfectly
22
Q

65 year old man is referred by his GP with altered bowel habit and rectal bleeding over the past 6 weeks. The blood is bright red and is not associated with any pain. He reports tenesmus and weight loss.

A. OGD
B. Colonoscopy
C. Barium Swallow
D. Flexi-Sig
E. Rigid Sig
F. CT KUB
G. CT CAP
H. USS
I. Abdo Xray
J. Proctoscopy
K. Barium Enema
L. None of the above
A

Flexible Sigmoidoscopy: visualisation rectum and sigmoid plus biopsies

Key words: altered bowel habit, rectal bleeding, tenesmus, weight loss

This patient has colorectal carcinoma: predisposing factors = IBD, polyps, FAP, smoking, low fibre diet

Genetics: one 1st degree relative 1:17, two first degree relatives 1:10

Stage using TMN, treat with chemoradiation/surgery

How well did you know this?
1
Not at all
2
3
4
5
Perfectly
23
Q

83 year old lady complains of tiredness and weight loss over the last 6 weeks. On examination a mass is felt in her right iliac fossa and currently weighs 40kgs, though she was over 50kg at her last appointment.

A. OGD
B. Colonoscopy
C. Barium Swallow
D. Flexi-Sig
E. Rigid Sig
F. CT KUB
G. CT CAP
H. USS
I. Abdo Xray
J. Proctoscopy
K. Barium Enema
L. None of the above
A

Colonoscopy:

Key words: tiredness, weight loss, mass

How well did you know this?
1
Not at all
2
3
4
5
Perfectly
24
Q

45 year old man complains to his GP of passing bloody stool without pain for the past 3 weeks. He reports a small amount of bright red blood with no mucus which he finds mostly on the paper.

A. OGD
B. Colonoscopy
C. Barium Swallow
D. Flexi-Sig
E. Rigid Sig
F. CT KUB
G. CT CAP
H. USS
I. Abdo Xray
J. Proctoscopy
K. Barium Enema
L. None of the above
A

Proctoscopy:

Key words: bloody stool, painless, on the paper

This patient has haemorrhoids = disrupted and dilated anal cushions. Usually at 3, 7, 11 o’clock when in lithotomy position.

1st degree: internal

2nd: Prolapse on defaecation, spontaneously reduce
3rd: Prolapse but requires digital reduction
4th: persistant prolapse

Treatment: Consevation, sclerosing agents, banding haemorrhoidectomy

How well did you know this?
1
Not at all
2
3
4
5
Perfectly
25
Q

45 year old woman complains of RUQ and shoulder pain. She has a fever and is Murphy’s sign positive.

A. OGD
B. Colonoscopy
C. Barium Swallow
D. Flexi-Sig
E. Rigid Sig
F. CT KUB
G. CT CAP
H. USS
I. Abdo Xray
J. Proctoscopy
K. Barium Enema
L. None of the above
A

USS

Acute Cholecystitis = impaction at the neck of the gall bladder

Will show a thick walled, shrunken GB, dilated CBD, stones, pericholestatic fluid

Tx: NBM, pain relief, IV Abx, surgery

Always consider Courvoisier’s Law - painless enlarged gall bladder and jaundice is unlikely to be gall stones

How well did you know this?
1
Not at all
2
3
4
5
Perfectly
26
Q

25 year old woman with UC involving the rectum has just undergone curative surgery.

A. Tracheostomy
B. Loop colostomy
C. End Ileostomy
D. Gastrostomy
E. De-functioning colostomy
F. Nephrostomy
G. Loop Ileostomy
H. End colostomy
I. Cystostomy
J. Urostomy
K. De-functioning ileostomy
L. None of the above
A

End Ileostomy: single lumen, spouted, right sided, liquid output.

When the entire colon is removed, the surgeon creates a stoma, in the abdominal wall. The tip of the lower small intestine is brought through the stoma. An external bag, or pouch, is attached to the stoma. This is called a permanent ileostomy. Stools pass through this opening and collect in the pouch. The pouch must be worn at all times.

Whereas an end colostomy has a single lumen, unspouted, left sided, stool like output.

How well did you know this?
1
Not at all
2
3
4
5
Perfectly
27
Q

A 50 year old man had surgery following mid-stage bladder cancer (T2-3) last year and is left with a stoma.

A. Tracheostomy
B. Loop colostomy
C. End Ileostomy
D. Gastrostomy
E. De-functioning colostomy
F. Nephrostomy
G. Loop Ileostomy
H. End colostomy
I. Cystostomy
J. Urostomy
K. De-functioning ileostomy
L. None of the above
A

Urostomy - most commonly performed after cystectomy or bladder cancer, for urinary diversion where drainage of the urine through the bladder and urethra isn’t possible.

How well did you know this?
1
Not at all
2
3
4
5
Perfectly
28
Q

A 65 year old man with a high rectal carcinoma has had an anterior resection.

A. Tracheostomy
B. Loop colostomy
C. End Ileostomy
D. Gastrostomy
E. De-functioning colostomy
F. Nephrostomy
G. Loop Ileostomy
H. End colostomy
I. Cystostomy
J. Urostomy
K. De-functioning ileostomy
L. None of the above
A

Loop ileostomy: double lumen, spout, liquid output, mucous fistula.

Allows the distal bowel to rest - designed to be temporary and reversed at a later date.

How well did you know this?
1
Not at all
2
3
4
5
Perfectly
29
Q

89 year old lady with a known history of colorectal cancer in the transverse colon. She has ESRF. She presents with symptoms of large bowel obstruction.

A. Tracheostomy
B. Loop colostomy
C. End Ileostomy
D. Gastrostomy
E. De-functioning colostomy
F. Nephrostomy
G. Loop Ileostomy
H. End colostomy
I. Cystostomy
J. Urostomy
K. De-functioning ileostomy
L. None of the above
A

De-functioning Ileostomy

How well did you know this?
1
Not at all
2
3
4
5
Perfectly
30
Q

What is a hernia?

A

A. Protrusion of a viscous or part of a viscus through a defect of the walls of its containing cavity into an abnormal position.

How well did you know this?
1
Not at all
2
3
4
5
Perfectly
31
Q

A patient has an inguinal hernia that has become ischaemic, what is this hernia described as?

A. Reducible
B. Incarcerated
C. Irreducible
D. Strangulated
E. Obstructed
A

Strangulated

How well did you know this?
1
Not at all
2
3
4
5
Perfectly
32
Q

What type of hernia forms through Hesselbach’s triangle?

A. Femoral
B. Direct Inguinal
C. Indirect Inguinal
D. Para-umbilical
E. Hiatus
A

Direct Inguinal

How well did you know this?
1
Not at all
2
3
4
5
Perfectly
33
Q

A 35 year old woman presents with a lump on her scalp. It is painless, attached to the skin and has a punctum.

A. Hernia
B. Keloid
C. Lipoma
D. Rheumatoid Nodule
E. Cutaneous Abscess
F. Ganglion
G. Fibroma
H. Neurofibroma
I. Sebaceous cyst
J. Lymph Node
A

Sebaceous cyst: can occur anywhere with sebaceous glands (i.e. not soles of feet/palms), they are intradermal and easily become infected.

Tx: excision

How well did you know this?
1
Not at all
2
3
4
5
Perfectly
34
Q

A 68 year old lady presents with a firm, painless lump on the back of her elbow. In her PMH she note she has arthritis and pulmonary fibrosis.

A. Hernia
B. Keloid
C. Lipoma
D. Rheumatoid Nodule
E. Cutaneous Abscess
F. Ganglion
G. Fibroma
H. Neurofibroma
I. Sebaceous cyst
J. Lymph Node
A

Rheumatoid Nodule: subcutaneous colleagenous granulomas, usually found on extensor surfaces (elbows and hands)

Appear in established rheumatoid arthritis patients (up to 25%)

How well did you know this?
1
Not at all
2
3
4
5
Perfectly
35
Q

A 45 year old man presents with a lump on a scar on his abdomen. He recently had a stoma reversal. He reports it is there all the time, is solid and itches sometimes.

A. Hernia
B. Keloid
C. Lipoma
D. Rheumatoid Nodule
E. Cutaneous Abscess
F. Ganglion
G. Fibroma
H. Neurofibroma
I. Sebaceous cyst
J. Lymph Node
A

Keloid: hypertrophy of vascularised collagen that extends outside the scar.

Tx: excision or steroids

How well did you know this?
1
Not at all
2
3
4
5
Perfectly
36
Q

A 22 year old man presents with a lump under his jaw. It is firm and is not fixed to the skin. He has recently been diagnosed with EBV.

A. Hernia
B. Keloid
C. Lipoma
D. Rheumatoid Nodule
E. Cutaneous Abscess
F. Ganglion
G. Fibroma
H. Neurofibroma
I. Sebaceous cyst
J. Lymph Node
A

Lymph node: can be due to infection (EBV, TB, HIV, Syphilis etc) or infiltration (malignancy, sarcoidosis)

How well did you know this?
1
Not at all
2
3
4
5
Perfectly
37
Q

A 21 year old man presents with a lump on the top of his foot. It is painless and appeared a few weeks ago. It transilluminates.

A. Hernia
B. Keloid
C. Lipoma
D. Rheumatoid Nodule
E. Cutaneous Abscess
F. Ganglion
G. Fibroma
H. Neurofibroma
I. Sebaceous cyst
J. Lymph Node
A

Ganglion (cyst): appear commonly on the dorsum of the hand or foot due to herniation of a synovial joint or tendon sheath.

They are thin walled cysts containing mutinous fluid.

“Bible cyst” - people used to hit them with a bible to make them go away!

Tx: Leave it! Aspiration or excision

How well did you know this?
1
Not at all
2
3
4
5
Perfectly
38
Q

A 27 year old man complains of pain in his jaw, he says it hurts most when he eats and is swollen, tender and red. He is afebrile.

A. MRI
B. FBC
C. FNA
D. USS
E. Sialogram
F. Angiography
G. X-ray
H. Barium Swallow
J. Excisional Biopsy
K. Reassure patient
A

Sialogram: this sounds like sialolithiasis - recurrent, unilateral pain especially on eating.

How well did you know this?
1
Not at all
2
3
4
5
Perfectly
39
Q

A 66 year old woman complains of pain in her jaw. O/E you find a small, hard lump adjacent to her ear lobe. She says it’s been getting larger over the past few months.

A. MRI
B. FBC
C. FNA
D. USS
E. Sialogram
F. Angiography
G. X-ray
H. Barium Swallow
J. Excisional Biopsy
K. Reassure patient
A

Excision biopsy: this sounds like a parotid tumour.

Salivary gland tumours:

  • 80% in parotid
  • 80% of these are pleomorphic adenomas
  • 80% of these are in the superficial lobe

Tx: surgery +/- radiotherapy (beware CNVII)

Parotid Pathology:

Acute/unilateral: stone
Acute/bi/uni-lateral: Mumps/HIV
Chronic/Unilateral: tumour, ALL, sarcoid
Chronic/Bilateral: Autoimmune disease

How well did you know this?
1
Not at all
2
3
4
5
Perfectly
40
Q

An 18 year old man presents with a lump in his neck. It is on the anterior border of his right sternocleidomastoid, a little over half way up his neck. He says it has been growing slowly but is causing him no trouble.

A. MRI
B. FBC
C. FNA
D. USS
E. Sialogram
F. Angiography
G. X-ray
H. Barium Swallow
J. Excisional Biopsy
K. Reassure patient
A

FNA: this sounds like a branchial cyst, which is a congenital malformation (t tend to be problematic unless they fistulae of become infected.
FNA reveals cholesterol crystals

Tx: Leave/excise

How well did you know this?
1
Not at all
2
3
4
5
Perfectly
41
Q

A 23 year old man presents with a firm lump in his neck. It is painless and mobile from side to side, but not up and down. It sits anterior to sternocleidomastoid.

A. MRI
B. FBC
C. FNA
D. USS
E. Sialogram
F. Angiography
G. X-ray
H. Barium Swallow
J. Excisional Biopsy
K. Reassure patient
A

Angiography: Sounds like carotid body pathology! May be aneurysmal or a tumour, and angiography will distinguish between the two, whereas duplex USS doesn’t.

They are rare and usually firm, painless, mobile lumps (but can be soft and pulsatile)

How well did you know this?
1
Not at all
2
3
4
5
Perfectly
42
Q

A 35 year old woman presents with a lump on her neck. It has uneven borders, is smooth, slightly fluctuant and is not fixed to the skin or deeper structures.

A. MRI
B. FBC
C. FNA
D. USS
E. Sialogram
F. Angiography
G. X-ray
H. Barium Swallow
J. Excisional Biopsy
K. Reassure patient
A

Reassure patient: Lipoma - uneven borders, smooth, slightly fluctuant, not fixed, benign fatty lump that occurs wherever fat can expand (not scalp/palms)
Only tend to be symptomatic with pressure and malignant change is rare

How well did you know this?
1
Not at all
2
3
4
5
Perfectly
43
Q

54 year old man with stage 3 CRF come to renal OP clinic. You see that his calcium is low and ALP is high. What test would you do to confirm suspected pathology?

A. Abdo USS
B. DEXA scan
C. Lumbar spine X-ray
D. FBC, U&E, LFT, urine dipstick

A

DEXA Scan - Renal osteodystrophy

  • High phosphate is due to inadequate excretion
  • Low calcium is due to low calcitriol production and poor renal reabsorption
  • Increased PTH
  • Increased bone resorption
  • Osteoporosis like picture on DEXA
How well did you know this?
1
Not at all
2
3
4
5
Perfectly
44
Q

35 year old woman underwent a kidney transplant for CRF 6 months ago. She has recently noticed a swelling in her neck. She is also complaining of constipation and low mood. Her GFR is normal. Abdominal CT scan shows some small calcifications in her transplanted kidney.

A. Primary Hyperparathyroidism
B. Secondary Hyperparathyroidism
C. Tertiary Hyperparathyroidism
D. Transplant rejection

A

Tertiary Hyperparathyroidism: after prolonged secondary hyperparathyroidism(due to CRF), parathyroid gland becomes autonomous.

Primary= something wrong with parathyroid (e.g. adenoma secreting excess PTH)
Secondary= Physiological response to hypocalcaemia/hyperphosphataemia
How well did you know this?
1
Not at all
2
3
4
5
Perfectly
45
Q

Which of these drugs is contraindicated in CRF?

A. Potassium Sparing diuretics
B. Gentamicin
C. NSAIDS
D. All of above

A

All of above

CRF: GFR 3 months
5 stages, end stage is GFR <15ml/min

Causes: Hypertension, Diabetes, Glomerulonephritis, renal vascular disease, recurrent pyelonephritis

Symptoms and Signs: fatigue, weakness, anorexia, vomiting, metallic taste, pruritis, bone pain, dyspnoea, ankle swelling, pallor, easy bruising, hypertension

Investigations:

  • Urine dip: proteinuria, haematuria
  • Bloods: low Hb (normocytic normochromic), raised urea and creatinine, hypocalcaemia, hyperphosphataemia (secondary hyperparathyroidism), Hyperkalaemia
  • USS small kidneys
  • DEXA: renal osteodystrophy
How well did you know this?
1
Not at all
2
3
4
5
Perfectly
46
Q

60 year old man with a 40 pack year history comes to see his GP with frank haematuria. Most likely diagnosis?

A. Schistosomiasis
B. Abdominal USS
C. Transitional Cell Carcinoma
D. Flucloxacillin
E. Anti-GBM antibody test
F. Nitrofurantoin
G. Giardia Lamblia
H. Penicillin V
I. CT KUB
J. Metronidazole
K. Malaria
L. Renal Biopsy
M. BPH
A

Transitional Cell Carcinoma

How well did you know this?
1
Not at all
2
3
4
5
Perfectly
47
Q

23 year old woman comes to her GP complaining of burning on passing urine. Urine dipstick is positive for blood, leukocytes and nitrites. What is the most appropriate treatment?

A. Schistosomiasis
B. Abdominal USS
C. Transitional Cell Carcinoma
D. Flucloxacillin
E. Anti-GBM antibody test
F. Nitrofurantoin
G. Giardia Lamblia
H. Penicillin V
I. CT KUB
J. Metronidazole
K. Malaria
L. Renal Biopsy
M. BPH
A

Nitrofurantoin

Commonest Pathogens: E.Coli, Klebsiella, Proteus

Abx: Nitrofurantoin, ciprofloxacin, augmentin

How well did you know this?
1
Not at all
2
3
4
5
Perfectly
48
Q

An 18 year old gap student is referred to a urologist because of a 4 week history of terminal haematuria. He also complains of some intermittent fevers and malaise. Eosinophil count is markedly raised. The patient mentions that he returned from Malawi about a month ago.
What is the most likely diagnosis?

A. Schistosomiasis
B. Abdominal USS
C. Transitional Cell Carcinoma
D. Flucloxacillin
E. Anti-GBM antibody test
F. Nitrofurantoin
G. Giardia Lamblia
H. Penicillin V
I. CT KUB
J. Metronidazole
K. Malaria
L. Renal Biopsy
M. BPH
A

Schistosomiasis: numerous species.

S. Haematobium causes urinary schisto (africa, middle east) whereas the other species cause intestinal schisto.

Transmission: drinking or swimming in water with contaminated schisto eggs

Increases the risk of squamous cell carcinoma of the bladder

How well did you know this?
1
Not at all
2
3
4
5
Perfectly
49
Q

A 35 year old man comes to A&E with severe left flank pain, that comes and goes in waves. What is the most appropriate next step?

A. Schistosomiasis
B. Abdominal USS
C. Transitional Cell Carcinoma
D. Flucloxacillin
E. Anti-GBM antibody test
F. Nitrofurantoin
G. Giardia Lamblia
H. Penicillin V
I. CT KUB
J. Metronidazole
K. Malaria
L. Renal Biopsy
M. BPH
A
CT KUB: Renal colic!
Always ask about:
- Loin to groin pain (often writhing in agony)
- Haematuria
- Frequency, dysuria
- Rigors, fever
- Hx of stones, recurrent UTIs

Abx: Nitrofurantoin, ciprofloxacin, augmentin

Differentials: biliary colic, pyelonephritis, acute pancreatitis, acute appendicitis, dissecting aortic aneurysm, epididymo-orchititis

Management renal colic: analgesia and anti-emetic, CT KUB, consider surgical drainage if septic

How well did you know this?
1
Not at all
2
3
4
5
Perfectly
50
Q

45 year old woman with a BMI of 35 develops RUQ pain. She rushes to A&E because she thinks she’s passing blood in her urine. What test should you do to confirm the most likely diagnosis?

A. Schistosomiasis
B. Abdominal USS
C. Transitional Cell Carcinoma
D. Flucloxacillin
E. Anti-GBM antibody test
F. Nitrofurantoin
G. Giardia Lamblia
H. Penicillin V
I. CT KUB
J. Metronidazole
K. Malaria
L. Renal Biopsy
M. BPH
A

Abdo USS

Fat Female Fair Forties

Biliary colic - dark urine

Differentials for Haematuria:

  • Bilirubinuria (obstructive jaundice)
  • Myoglobinuria
  • Food (beetroot)
  • Drugs (rifampicin, nitrofurantoin)
  • Porphyria (urine darken on standing)
How well did you know this?
1
Not at all
2
3
4
5
Perfectly
51
Q

Liver Enzymes:

Bil 38 umol/L (s

A

Gilbert’s: Genetic predisposition to having jaundice (unconjugated Bilirubin).
Harmless, other LFTs are normal.

Triggers: dehydration, fasting, illness, stress, PMS

How well did you know this?
1
Not at all
2
3
4
5
Perfectly
52
Q

Liver Enzymes:

Bil 246 umol/L (<40)
ALP 1124 U/L (30-130)
GGT 60 U/L (11-42)

A. Cholestasis
B. Viral Hepatitis
C. Harmful Alcohol use
D. Cirrhosis

A

Cholestasis: ALP most elevated!

When bile cannot flow from the gallbladder to the duodenum.

Obstructive jaundice: gallstones, PBC/PSC, pancreatic cancer - pale stools, dark urine

Requires USS to look for cause

How well did you know this?
1
Not at all
2
3
4
5
Perfectly
53
Q

Liver Enzymes:

Bil 30 umol/L (<40)
ALP 57 U/L (30-130)
GGT 112 U/L (11-42)

A. Paracetamol OD
B. Viral Hepatitis
C. Cholestasis
D. Harmful Alcohol use

A

Alcohol! - GGT is high in isolation
In alcoholic liver disease: AST>ALT ratio >2:1
In cirrhosis AST>ALT as well
Later on may progress to alcoholic liver disease, fibrosis and cirrhosis.

How well did you know this?
1
Not at all
2
3
4
5
Perfectly
54
Q

Liver Enzymes:

Bil 25 umol/L (<40)
ALP 280 U/L (30-130)
GGT 470 U/L (11-42)
INR 2.7 (0.8-1.2)

A. Paracetamol OD
B. Alcoholic Liver Disease
C. Cholestasis
D. Hepatitis C

A

Paracetamol OD: transferases are very high = high level of liver damage

Acute marker of liver function is affected (INR), but chronic is not (albumin)

How well did you know this?
1
Not at all
2
3
4
5
Perfectly
55
Q

Liver Enzymes:

Bil 178 umol/L (<40)
ALP 180 U/L (30-130)
GGT 35 U/L (11-42)

A. Alcoholic Liver Disease
B. Acute Vital Hepatitis
C. Obstructive Jaundice
D. Cirrhosis

A

Viral Hepatitis: very high transaminases.

HepA - Acute, shellfish
HepB - mostly acute, some chronic
HepC - chronic

How well did you know this?
1
Not at all
2
3
4
5
Perfectly
56
Q

21 year old male comes to the GP practise with intermittent fever and abdo pain (2/12 duration). He also mentions getting up in the night to pass stool and has had some bloody diarrhoea.

A. Haemorrhoids
B. Alcoholic Liver Disease
C. Bowel Cancer
D. IBS
E. Viral Gastroenteritis
F. IBD
G. Rectal Varices
H. Bleeding Duodenal Ulcer
I. Septicaemia
J. Malnutrition
A

IBD - diarrhoea (bloody, chronic and often nocturnal)
Abdo pain, fevers, weight loss
Nutritional deficiencies (Iron, B12, folate)
Extra-intestinal manifestations - mouth, skin, eyes, joints (pyoderma gangrenosum, link with PSC)
Inflammation - elevated CRP and platelets
FH of IBD

UC always affects the rectum and never affects the small bowel

How well did you know this?
1
Not at all
2
3
4
5
Perfectly
57
Q

60 year old female complains of tiredness and constipation, but no PR bleeding.

A. Haemorrhoids
B. Alcoholic Liver Disease
C. Bowel Cancer
D. IBS
E. Viral Gastroenteritis
F. IBD
G. Rectal Varices
H. Bleeding Duodenal Ulcer
I. Septicaemia
J. Malnutrition
A

Bowel Cancer: Patients may not recognise PR bleeding if it is black/mixed into stool.
Anaemia in men or post-menopausal women MUST be investigated with upper and lower GI endoscopy
Sometimes the only sign may be a change in bowel habit

How well did you know this?
1
Not at all
2
3
4
5
Perfectly
58
Q

30 year old male has been suffering from anal itch, and bright red blood splashed over the toilet bowel after bowel movements.

A. Haemorrhoids
B. Alcoholic Liver Disease
C. Bowel Cancer
D. IBS
E. Viral Gastroenteritis
F. IBD
G. Rectal Varices
H. Bleeding Duodenal Ulcer
I. Septicaemia
J. Malnutrition
A

Haemorrhoids: 4 degrees of haemorrhoids, ranging from completely internal to completely prolapsed.

Causes anal itch and soreness. Strangulated haemorrhoids may thrombose which is intensely painful.

Tx: High fibre, pain relief, ligation, injection or surgery if necessary

How well did you know this?
1
Not at all
2
3
4
5
Perfectly
59
Q

32 year old Asian male come to A&E with dizziness and collapse. His Hb is 7.2g/dL. His wife mentions that he has been taking omeprazole for the last couple of months.

A. Haemorrhoids
B. Alcoholic Liver Disease
C. Bowel Cancer
D. IBS
E. Viral Gastroenteritis
F. IBD
G. Rectal Varices
H. Bleeding Duodenal Ulcer
I. Septicaemia
J. Malnutrition
A

Bleeding duodenal ulcer: can cause massive melaena - black tarry stools with a characteristic smell.
Hx of dyspepsia

Tx: ABC, Endoscopy, Surgery

How well did you know this?
1
Not at all
2
3
4
5
Perfectly
60
Q

50 year old man is rushed into A&E because of profuse PR bleeding. O/E he has very red palms, a swollen abdomen and yellow sclera.

A. Haemorrhoids
B. Alcoholic Liver Disease
C. Bowel Cancer
D. IBS
E. Viral Gastroenteritis
F. IBD
G. Rectal Varices
H. Bleeding Duodenal Ulcer
I. Septicaemia
J. Malnutrition
A

Rectal Varices: Cirrhotic liver disease causes portal hypertension causing varicose (both oesophageal and rectal) - massive bleed of fresh red blood

How well did you know this?
1
Not at all
2
3
4
5
Perfectly
61
Q

65 year old woman comes into A&E with sudden onset right sided weakness of the face and arm. She has a history of high BP, for which she takes amlodipine. O/E her right arm is rigid and hyper-reflexic.

A. Lambert-Eaton Syndrome
B. Parkinson's Disease
C. MS
D. Myasthenia Gravis
E. Hypothyroidism
F. Muscular Dystrophy
G. Cerebrovascular Accident
H. Epilepsy
I. Alzheimer's Dementia
A

CVA - UMN signs: Spasticity, hyper-reflexia, Babinski sign
As well as focal signs according to lesion e.g. hemiparesis, hemianopias or general signs e.g. headaches, vomiting

LMN signs = muscle wasting, floppy paralysis, fasciculations

How well did you know this?
1
Not at all
2
3
4
5
Perfectly
62
Q

50 year old smoker with a 6 month history of SOB and a 2 day Hx of haemoptysis. He has also noticed weight loss of about a stone in the last month. Peripheral nerve exam showed significant weakness in the arms and legs, but it improved on repeated use.

A. Lambert-Eaton Syndrome
B. Parkinson's Disease
C. MS
D. Myasthenia Gravis
E. Hypothyroidism
F. Muscular Dystrophy
G. Cerebrovascular Accident
H. Epilepsy
I. Alzheimer's Dementia
A

Lambert Eaton Syndrome: Paraneoplastic (associated with small cell lung cancer) or autoimmune - antibodies against the presynaptic calcium channel.
Causes proximal muscle weakness and gait abnormalities.
Improves with exercise

How well did you know this?
1
Not at all
2
3
4
5
Perfectly
63
Q

40 year old woman comes to her GP complaining that her eyelids are too droopy. She also cannot complete a full meal because she finds it hard to swallow after a while. PMH: hyperthyroidism. The GP notices that her voice keeps trailing off at the end of sentences.

A. Lambert-Eaton Syndrome
B. Parkinson's Disease
C. MS
D. Myasthenia Gravis
E. Hypothyroidism
F. Muscular Dystrophy
G. Cerebrovascular Accident
H. Epilepsy
I. Alzheimer's Dementia
A

Myasthenia Gravis: antibody against the ACh receptor at NMJ.
LMN signs, affects multiple muscle groups
Oculomotor/Bulbar signs predominate.
Worsens with exercise

Other NMJ: organophosphate poisoning, botulism

How well did you know this?
1
Not at all
2
3
4
5
Perfectly
64
Q

A 4 year old boy is brought to his GP by his mother who is concerned that he is not able to run like the other boys at nursery. He seems to “waddle” when he walks and falls over frequently. He also has great difficulty climbing stairs. He is positive for Gowers sign.

A. Lambert-Eaton Syndrome
B. Parkinson's Disease
C. MS
D. Myasthenia Gravis
E. Hypothyroidism
F. Muscular Dystrophy
G. Cerebrovascular Accident
H. Epilepsy
I. Alzheimer's Dementia
A

Muscular Dystrophy: 2 commonest types are Duchenne and Beckers, abnormality in dystrophin protein
X-linked recessive
Muscle wasting and specific functional weakness: signs of proximal myopathy include difficulty in getting out of chairs, and difficulty climbing stairs

How well did you know this?
1
Not at all
2
3
4
5
Perfectly
65
Q

30 year old female presents with a 2 day history of pain and loss of vision in the left eye. This happened once before 6 months ago, but resolved spontaneously. OE she has reduced fine touch sensation in both feet, as well as weakness.

A. Lambert-Eaton Syndrome
B. Parkinson's Disease
C. MS
D. Myasthenia Gravis
E. Hypothyroidism
F. Muscular Dystrophy
G. Cerebrovascular Accident
H. Epilepsy
I. Alzheimer's Dementia
A

MS - optic neuritis is the most common presentation.
Also weakness, fatigue, paraesthesia, paralysis
Main types are:
- Relapsing-remitting
- Secondary progressive
- Primary progressive

How well did you know this?
1
Not at all
2
3
4
5
Perfectly
66
Q

A 45 year old woman is recovering on a ward after gall bladder surgery. You are called to see her by the night nurse on duty because she has become very agitated, is shouting incoherently and is convinced that she is covered in spiders. What do you do next?

A. Refer her to psychiatry in the morning for a full work up
B. Administer anti-psychotics
C. Start her on fluids to help her sober up
D. Consider prescribing some benzodiazepines

A

Benzodiazepines: delirium tremens!!! - chlordiazepoxide

Alcohol withdrawal state:
- Agitation, confusion, fluctuating consciousness, seizures, arrhythmias, death (

How well did you know this?
1
Not at all
2
3
4
5
Perfectly
67
Q

A known alcoholic comes into A&E clearly drunk, his speech is slurred, he cannot walk straight and he seems to be quite floppy sitting in his chair. However you measure his blood alcohol level and are surprised to find it is 0.
What do you suspect he has?

A. Alcohol withdrawal
B. Delayed alcohol intoxication
C. Methanol intoxication
D. Cerebellar Syndrome

A

Cerebellar Syndrome: dysdiadochokinesia, ataxia, nystagmus, intention tremor, slurred speech, hypotonia

How well did you know this?
1
Not at all
2
3
4
5
Perfectly
68
Q

A 39 year old man, who has been drinking 40 units a week for the past 5 years come to his GP complaining of numbness and tingling in his toes. It has been present for about 4 months. His wife also noted that “he’s been having a bit of trouble in the bedroom recently”.
Which blood test would help you confirm a reason for this man’s symptoms?

A. Hb
B. Blood alcohol level
C. B12 and folate
D. Calcium

A

B12 and folate: The commonest causes of peripheral neuropathy are diabetes and B12 deficiency.
Tends to occur in a glove and stocking distribution, is usually symmetrical and can also cause impotence and autonomic dysfunction

How well did you know this?
1
Not at all
2
3
4
5
Perfectly
69
Q

A 60 year old publican is brought to A&E by a friend who says he’s been more confused over the last few weeks, and has vomited a few times in the last few days. When you see him he seems to intermittently drift off.

A. Acute Alcohol intoxication
B. Delirium tremens
C. Subdural Haemorrhage
D. Alzheimer’s Dementia

A

Subdural haemorrhage: cerebral atrophy, frequent falls, clotting disorder (liver failure)

How well did you know this?
1
Not at all
2
3
4
5
Perfectly
70
Q

A 50 year old Irish man stumbles into A&E mumbling incoherently, not exactly sure where he is. He complains that he is having double vision. When asked he recalls the Prime Minister as being “Bertrand Ebstein”, and tells you that he came here on a moped hired from his friend “Robbie”.
Most likely diagnosis?

A. Acute Alcohol intoxication
B. Delirium tremens
C. Wernicke-Korsakoff’s Syndrome
D. Alzheimer’s Dementia

A

Wernicke’s Encephalopathy: confusion, ataxia, opthalmoplegia

Tx: IV Pabrinex - thiamine (B1), riboflavin (B2), nicotinamide (B3), pyridoxine (B6) and ascorbic acid (vit C)

Korsakoff’s Syndrome: untreated Wernicke’s progresses to this: anterograde amnesia, confabulation

Alcoholics can get other forms of dementia too (10% of cases)

How well did you know this?
1
Not at all
2
3
4
5
Perfectly
71
Q

A 60 year old man is rushed into A&E with a reducing GCS. OE you find a swollen abdomen, yellow sclera, and a sickly sweet smell on his breath.
What is the most likely diagnosis?

A. Acute intoxication
B. DKA
C. Decompensated liver failure
D. Sepsis

A

Decompensated liver failure: hepatic encephalopathy
Due to ammonia and other waste products crossing BBB.
- Inverted sleep wake cycle
- Lethargy, personality change
- Worsening confusion and LOC
- Signs of liver failure e.g. jaundice, ascites, fetor hepaticas

How well did you know this?
1
Not at all
2
3
4
5
Perfectly
72
Q

30 year old male presents with a one week history of SOB and 2 days of coughing up blood. Temp 39 degrees.

A. Bronchiectasis
B. Staph aureus pneumonia
C. Foreign body
D. TB
E. Wegener's granulomatosis
F. PE
G. Oesophageal varices
H. Goodpasture's syndrome
I. Lung Cancer
J. Kaposi's Sarcoma
A

Staph aureus pneumonia: commonest cause of haemoptysis (~60%)

Other pneumonias that commonly cause haemoptysis: Pseudomonas, influenza

How well did you know this?
1
Not at all
2
3
4
5
Perfectly
73
Q

70 year old male with a 40 pack year history sees his GP because he has been coughing up blood in the last 2 weeks.

A. Bronchiectasis
B. Staph aureus pneumonia
C. Foreign body
D. TB
E. Wegener's granulomatosis
F. PE
G. Oesophageal varices
H. Goodpasture's syndrome
I. Lung Cancer
J. Kaposi's Sarcoma
A

Lung Cancer: elderly smokers - lung cancer until proven otherwise.

Non Small Cell (80%): squamous cell (PTHrp release, treated with radiotherapy), adenocarcinoma (usually located peripherally in the lung and are more common in non-smokers although most cases are still associated with smoking), large cell carcinoma - associated with clubbing. The paraneoplastic syndromes may include Lambert-Eaton myasthenic syndrome.

Small Cell (20%) - more aggressive, treated with chemotherapy and is associated with SIADH and ectopic ACTH.

How well did you know this?
1
Not at all
2
3
4
5
Perfectly
74
Q

A 3 year old girl is brought into Paeds A&E because she coughed up some blood on her way home from nursery.

A. Bronchiectasis
B. Staph aureus pneumonia
C. Foreign body
D. TB
E. Wegener's granulomatosis
F. PE
G. Oesophageal varices
H. Goodpasture's syndrome
I. Lung Cancer
J. Kaposi's Sarcoma
A

Foreign Body: the commonest causes of haemoptysis in young children is a foreign body and infection.

How well did you know this?
1
Not at all
2
3
4
5
Perfectly
75
Q

A 50 year old alcoholic is brought in by ambulance. His wife says he has coughed up about 2-3 cups of blood in the last hour. The husband remarks the blood looked like coffee.

A. Bronchiectasis
B. Staph aureus pneumonia
C. Foreign body
D. TB
E. Wegener's granulomatosis
F. PE
G. Oesophageal varices
H. Goodpasture's syndrome
I. Lung Cancer
J. Kaposi's Sarcoma
A

Oesophageal Varices: dilated veins in the distal oesophagus and/or proximal stomach, caused by portal hypertension often due to chronic liver disease.

How well did you know this?
1
Not at all
2
3
4
5
Perfectly
76
Q

A 35 year old Bangladeshi immigrant comes to see her GP with unintentional weight loss of 10kg in 2 months and a 4 week history of coughing up blood.

A. Bronchiectasis
B. Staph aureus pneumonia
C. Foreign body
D. TB
E. Wegener's granulomatosis
F. PE
G. Oesophageal varices
H. Goodpasture's syndrome
I. Lung Cancer
J. Kaposi's Sarcoma
A

TB: Cough, weight loss/anorexia, fever, night sweats haemoptysis, chest pain.

RFs: HIV infection, history of prior TB, travel to an area where TB is endemic. homelessness, shelter dwelling

Dx: Chest X-ray, AFB smear and culture (sputum) - Ziehl-Neelsen staining, HIV serology

How well did you know this?
1
Not at all
2
3
4
5
Perfectly
77
Q

A 5 year old girl comes into the GP surgery with her mother, who says her daughter is still coughing throughout the night despite regularly using her salbutamol inhaler.

A. Use the salbutamol more frequently.
B. Add a leukotriene receptor antagonist
C. Add a minimum dose of inhaled steroids
D. Start with the maximum dose of inhaled steroids and slowly taper down

A

Add a minimum dose of inhaled steroids

How well did you know this?
1
Not at all
2
3
4
5
Perfectly
78
Q

A 3 year old boy has trouble using his steroid inhaler properly as he cannot co-ordinate his breathing with the pump.

A. Education, education, education
B. Try a spacer device
C. Start him on a different steroid inhaler
D. Start him on oral treatment

A

Try a spacer device

How well did you know this?
1
Not at all
2
3
4
5
Perfectly
79
Q

A 15 year old comes into A&E brought in by his mother, with an acute attack.

A. Start IV steroids
B. Ask the mother to keep giving him his inhaler
C. Start oxygen and nebuliser salbutamol via a facemask
D. Do a peak flow and pulse oximetry

A

Do a peak flow and pulse oximetry

How well did you know this?
1
Not at all
2
3
4
5
Perfectly
80
Q

A 21 year old asthmatic has been in A&E for 4 hours with no improvement in her acute attack despite ongoing nebulised therapy. She is starting to become confused, and has poor respiratory effort. You try to listen to her chest but cannot hear anything at all.

A. Panic
B. Get help
C. Start IV steroids
D. Continue nebulisers and hope for improvement

A

Get help!

Asthma attacks:
Moderate: PEFR >50-70% of expected
Acute Severe: PEFR 33-50% of predicted, RR >25/min, HR>100/min, inability to complete sentences in one breath
Life threatening: PEFR 6kPa

How well did you know this?
1
Not at all
2
3
4
5
Perfectly
81
Q

A 28 year old man has a 2 day history of malaise, fever and dyspnoea. He is coughing up reddish-brown sputum and his X-ray shows right lower lobe consolidation.

A. Chlamydia psittaci
B. Haemophilis influenzae
C. Legionella pneumophilia
D. Mycobacterium tuberculosis
E. Pneumocystis jiroveci
F. Mycoplasma pneumoniae
G. Pseudomonas aeruginosa
H. Staphylococcus aureus
I. Streptococcus pneumoniae
J. Klebsiella pneumoniae
A

Streptococcus pneumoniae: affects all ages, fever, malaise, productive cough with reddish-brown sputum, consolidation on CXR

How well did you know this?
1
Not at all
2
3
4
5
Perfectly
82
Q

A 53 year old man is brought to the emergency room following a seizure. His wife said he has been having muscle pains and a dry cough for the past week since they returned from their holiday. His sodium level is 124mmol/L and potassium is 5.4mmol/L

A. Chlamydia psittaci
B. Haemophilis influenzae
C. Legionella pneumophilia
D. Mycobacterium tuberculosis
E. Pneumocystis jiroveci
F. Mycoplasma pneumoniae
G. Pseudomonas aeruginosa
H. Staphylococcus aureus
I. Streptococcus pneumoniae
J. Klebsiella pneumoniae
A

Legionella pneumophilia: colonises water tanks in AC (hotels, plumbers), dry cough and dyspnoea, low sodium, deranged LFTs

How well did you know this?
1
Not at all
2
3
4
5
Perfectly
83
Q

A 42 year old prostitute presents with a 2 month history of a dry cough and progressive dyspnoea. She also complains of night sweats. Her oxygen saturations are 89%. She does not have any other signs or symptoms.

A. Chlamydia psittaci
B. Haemophilis influenzae
C. Legionella pneumophilia
D. Mycobacterium tuberculosis
E. Pneumocystis jiroveci
F. Mycoplasma pneumoniae
G. Pseudomonas aeruginosa
H. Staphylococcus aureus
I. Streptococcus pneumoniae
J. Klebsiella pneumoniae
A

Pneumocystis jiroveci: Immunosuppressed e.g. HIV, dry cough, dyspnoea, CXR may be normal, BAL: boat shaped organisms using a silver stain.

How well did you know this?
1
Not at all
2
3
4
5
Perfectly
84
Q

A 67 year old presents with malaise and a productive cough. She has just recovered from a bout of influenza. Her CXR demonstrates 2 cavities with air-fliud levels.

A. Chlamydia psittaci
B. Haemophilis influenzae
C. Legionella pneumophilia
D. Mycobacterium tuberculosis
E. Pneumocystis jiroveci
F. Mycoplasma pneumoniae
G. Pseudomonas aeruginosa
H. Staphylococcus aureus
I. Streptococcus pneumoniae
J. Klebsiella pneumoniae
A

Staphylococcus aureus: affects all ages, complicates influenza, IVDU, bilateral caveatting lesions.

How well did you know this?
1
Not at all
2
3
4
5
Perfectly
85
Q

A 33 year old presents with a 2 day history of ‘eyes that are really red and painful.’ She finds it too difficult to move her eyes whilst the doctor is testing her cranial nerves. She has elevated serum ACE and improves with corticosteroids.

Which sarcoid complication:
A. Anterior uveitis
B. Choroidoretinitis
C. Facial nerve palsy
D. Diabetes Insipidus
E. DIabetes Mellitus
F. Erythema ab ignae
G. Lupus pernio
H. Erythema marginatum
I. Peripheral Neuropathy
J. Erythema nodosum
A

Anterior uveitis

Sarcoidosis is a chronic granulomatous lung disorder (multinucleate giant cells). More common in young adults, and more severe in Afro-Caribbeans. It is often as symptomatic but respiratory symptoms most commonly are dry cough and SOB.

Dx:

  • CXR: bilateral hilar lymphadenopathy (think sarcoidosis, lymphoma, TB)
  • Transbronchial biopsy: histology
  • Lung function tests: restrictive
  • Serum ACE: assessment purposes

Tx: Corticosteroids

Extra pulmonary features:

  • Skin: erythema nodosum (shin), Lupus pernio (face)
  • Eyes: anterior uveitis (red painful eye)
  • Diabetes Insipidus: normal glucose, treated with vasopressin (DDAVP)
How well did you know this?
1
Not at all
2
3
4
5
Perfectly
86
Q

A 26 year old man presents to his GP with dyspnoea. On examination, he has a maculopapular violaceous rash over his nose. His CXR reveals bilateral hilar lymphadenopathy.

Which sarcoid complication:
A. Anterior uveitis
B. Choroidoretinitis
C. Facial nerve palsy
D. Diabetes Insipidus
E. DIabetes Mellitus
F. Erythema ab ignae
G. Lupus pernio
H. Erythema marginatum
I. Peripheral Neuropathy
J. Erythema nodosum
A

Lupus pernio

Sarcoidosis is a chronic granulomatous lung disorder (multinucleate giant cells). More common in young adults, and more severe in Afro-Caribbeans. It is often as symptomatic but respiratory symptoms most commonly are dry cough and SOB.

Dx:

  • CXR: bilateral hilar lymphadenopathy (think sarcoidosis, lymphoma, TB)
  • Transbronchial biopsy: histology
  • Lung function tests: restrictive
  • Serum ACE: assessment purposes

Tx: Corticosteroids

Extra pulmonary features:

  • Skin: erythema nodosum (shin), Lupus pernio (face)
  • Eyes: anterior uveitis (red painful eye)
  • Diabetes Insipidus: normal glucose, treated with vasopressin (DDAVP)
How well did you know this?
1
Not at all
2
3
4
5
Perfectly
87
Q

A 29 year old lady with sarcoidosis is admitted for polyuria and polydipsia. Her blood sodium is 154mmol/L and her fasting plasma glucose is 4.8. Her urine had a very low osmolality.

Which sarcoid complication:
A. Anterior uveitis
B. Choroidoretinitis
C. Facial nerve palsy
D. Diabetes Insipidus
E. DIabetes Mellitus
F. Erythema ab ignae
G. Lupus pernio
H. Erythema marginatum
I. Peripheral Neuropathy
J. Erythema nodosum
A

Diabetes insipidus

Sarcoidosis is a chronic granulomatous lung disorder (multinucleate giant cells). More common in young adults, and more severe in Afro-Caribbeans. It is often as symptomatic but respiratory symptoms most commonly are dry cough and SOB.

Dx:

  • CXR: bilateral hilar lymphadenopathy (think sarcoidosis, lymphoma, TB)
  • Transbronchial biopsy: histology
  • Lung function tests: restrictive
  • Serum ACE: assessment purposes

Tx: Corticosteroids

Extra pulmonary features:

  • Skin: erythema nodosum (shin), Lupus pernio (face)
  • Eyes: anterior uveitis (red painful eye)
  • Diabetes Insipidus: normal glucose, treated with vasopressin (DDAVP)
How well did you know this?
1
Not at all
2
3
4
5
Perfectly
88
Q

A 33 year old lady presents to her GP with worsening dyspnoea and a large lesion over the anterior surface of her shin. Her CXR shows bilateral hilar lymphadenopathy and lung function tests demonstrate a restrictive defect.

Which sarcoid complication:
A. Anterior uveitis
B. Choroidoretinitis
C. Facial nerve palsy
D. Diabetes Insipidus
E. DIabetes Mellitus
F. Erythema ab ignae
G. Lupus pernio
H. Erythema marginatum
I. Peripheral Neuropathy
J. Erythema nodosum
A

Erythema nodosum

Sarcoidosis is a chronic granulomatous lung disorder (multinucleate giant cells). More common in young adults, and more severe in Afro-Caribbeans. It is often as symptomatic but respiratory symptoms most commonly are dry cough and SOB.

Dx:

  • CXR: bilateral hilar lymphadenopathy (think sarcoidosis, lymphoma, TB)
  • Transbronchial biopsy: histology
  • Lung function tests: restrictive
  • Serum ACE: assessment purposes

Tx: Corticosteroids

Extra pulmonary features:

  • Skin: erythema nodosum (shin), Lupus pernio (face)
  • Eyes: anterior uveitis (red painful eye)
  • Diabetes Insipidus: normal glucose, treated with vasopressin (DDAVP)
How well did you know this?
1
Not at all
2
3
4
5
Perfectly
89
Q

A 67 year old lady presents to A&E with pyrexia (38.1 degrees) and a productive cough. HR 101, BP 80/60, RR 32. She is alert and orientated in time and space. WCC 21, Urea 8.5, CXR shows consolidation of the right upper lobe.
CURB-65 Score?

A. 6
B. 8
C. 4
D. 0
E. 1
A
4
CURB-65:
- Confusion
- Urea > 7 mmol/L
- RR > 30/min
- BP: systolic
How well did you know this?
1
Not at all
2
3
4
5
Perfectly
90
Q

A 72 year old lady presents with a 4 month history of SOB, initially on exertion but now at rest.
PMH: AF, HTN, high cholesterol.
O/E: SpO2 92% (RA), RR 19, apyrexial, fine bibasal inspiratory crackles.

Which of these drugs is most likely to cause the symptoms experienced by the patient?

A. Amlodipine
B. Simvastatin
C. Aspirin
D. Amiodarone
E. Alendronate
A

Amiodarone - pulmonary fibrosis

How well did you know this?
1
Not at all
2
3
4
5
Perfectly
91
Q

A 65 year old man presents with a painless wound in his right leg which has been present for 2 months. O/E you notice a 3cm by 4cm leg ulcer covering the medial malleolus. The shallow bed of the ulcer is covered in granulation tissue which is surrounded by sloping edges.

A. Anthrax
B. Chancrous ulcer
C. Gummatous ulcer 
D. Arterial ulcer
E. Neuropathic ulcer
F. Squamous cell carcinoma
G. Tuberculous ulcer
H. Venous ulcer
I. Pyoderma gangrenosum
J. Marjolin's ulcer
A

Venous Ulcer: caused by superficial/deep venous incompetence

Clues: gaiter region, covering medial mallelous, lipodermatosclerosis, brown discolouration (haemosiderin deposition), extensive - can be painless

How well did you know this?
1
Not at all
2
3
4
5
Perfectly
92
Q

A 60 year old man with T2DM presents with a large ulcerating lesion over his left toes. The lesion has well demarcated edges. The leg is cool to touch and is hairless and dorsal is pedis and posterior tibial pulses are absent.

A. Anthrax
B. Chancrous ulcer
C. Gummatous ulcer 
D. Arterial ulcer
E. Neuropathic ulcer
F. Squamous cell carcinoma
G. Tuberculous ulcer
H. Venous ulcer
I. Pyoderma gangrenosum
J. Marjolin's ulcer
A

Arterial ulcer: caused by large vessel (atherosclerosis) or small vessel (vasculitis) disease

Clues: often located distally, punched out appearance absent pulses, low ABPI, shiny, hairless skin, arteriopath, smoking, DM, CVD, very PAINFUL

How well did you know this?
1
Not at all
2
3
4
5
Perfectly
93
Q

A 66 year old retired Marines officer presents with a large scar from a burn sustained in a war with a painless 3 cm area of irregular ulceration within the region of scarring. It is surrounded by a raised edge and has bloodstained discharge from the base. There is no regional lymphadenopathy.

A. Anthrax
B. Chancrous ulcer
C. Gummatous ulcer 
D. Arterial ulcer
E. Neuropathic ulcer
F. Squamous cell carcinoma
G. Tuberculous ulcer
H. Venous ulcer
I. Pyoderma gangrenosum
J. Marjolin's ulcer
A

Marjolin’s ulcer: eponymous name for a squamous call carcinoma arising in an area of previous inflammation e.g. long standing venous ulcer or 3rd degree burn (clue: war veterans)

Features of a SCC: can be painless, blood stained discharge, raised edge, spread to LN is rare

How well did you know this?
1
Not at all
2
3
4
5
Perfectly
94
Q

A 27 year old lady with altered bowel habit, and rectal bleeding presents with a large ulcerating lesion on her left leg with dark red edges.

A. Anthrax
B. Chancrous ulcer
C. Gummatous ulcer 
D. Arterial ulcer
E. Neuropathic ulcer
F. Squamous cell carcinoma
G. Tuberculous ulcer
H. Venous ulcer
I. Pyoderma gangrenosum
J. Marjolin's ulcer
A

Pyoderma gangrenosum: recurring nodulo-pustular ulcers with tender red/blue necrotic edges, purulent.
Associated mainly with IBD, but also AI hepatitis, myelodysplasias and Wegener’s granulomatosis.

How well did you know this?
1
Not at all
2
3
4
5
Perfectly
95
Q

A 60 year old obese man presents with ulceration on the sole of his right foot. The ulcer is deep and penetrating but the skin around it appears well perfused.

A. Anthrax
B. Chancrous ulcer
C. Gummatous ulcer 
D. Arterial ulcer
E. Neuropathic ulcer
F. Squamous cell carcinoma
G. Tuberculous ulcer
H. Venous ulcer
I. Pyoderma gangrenosum
J. Marjolin's ulcer
A

Neuropathic ulcer: repetitive trauma with absent/reduced sensation.

Clues: loss of sensation, diabetics, lack of ankle reflexes, weight bearing areas

How well did you know this?
1
Not at all
2
3
4
5
Perfectly
96
Q

A 30 year old man presents with a lump in the posterior triangle of the neck which has been enlarging slowly for the last 2 months. He has also felt feverish, itchy and sweaty at night, and has lost 8kg unintentionally.

A. Pharyngeal pouch
B. Thyroglossal cyst
C. Sternocleidomastoid tumour
D Cystic hygroma
E. Carotid body tumour
F. Branchial cyst
G. Lymphoma
H. Thyroid goitre
I. Salivary duct carcinoma
J. Acoustic neuroma
A

Lymphoma: lethargy, weight loss, painful lymph nodes after alcohol, night sweats, itchiness

How well did you know this?
1
Not at all
2
3
4
5
Perfectly
97
Q

A 25 year old presents with a painful swelling which bulges from under the anterior border of the sternocleidomastoid. It is soft, fluctuant but tender on examination. He says that whilst it has been present since he was a child, it only recently started causing him trouble after he suffered from pneumonia.

A. Pharyngeal pouch
B. Thyroglossal cyst
C. Sternocleidomastoid tumour
D Cystic hygroma
E. Carotid body tumour
F. Branchial cyst
G. Lymphoma
H. Thyroid goitre
I. Salivary duct carcinoma
J. Acoustic neuroma
A

Branchial cyst: remnant of 2nd branchial cleft (present from childhood), smooth, non-tender, fluctuant lump, 2/3 up sternocleidomastoid, may become enlarged and painful after respiratory infection.
FNA: yellow fluid with cholesterol crystals

How well did you know this?
1
Not at all
2
3
4
5
Perfectly
98
Q

A 70 year old woman complains of a mass on her right jaw which has been slowly growing for the past 6 months. For the past week she has been unable to move the right side of her face. O/E you notice a hard, irregular lump which extends up behind the angle of the mandible.

A. Pharyngeal pouch
B. Thyroglossal cyst
C. Sternocleidomastoid tumour
D Cystic hygroma
E. Carotid body tumour
F. Branchial cyst
G. Lymphoma
H. Thyroid goitre
I. Salivary duct carcinoma
J. Acoustic neuroma
A

Salivary duct carcinoma: hard irregular mass, facial nerve involvement.

How well did you know this?
1
Not at all
2
3
4
5
Perfectly
99
Q

A 28 year old man presents with a lump in the posterior triangle of the neck. He mentions difficulty in finishing meals and sometimes regurgitates food at night.

A. Pharyngeal pouch
B. Thyroglossal cyst
C. Sternocleidomastoid tumour
D Cystic hygroma
E. Carotid body tumour
F. Branchial cyst
G. Lymphoma
H. Thyroid goitre
I. Salivary duct carcinoma
J. Acoustic neuroma
A

Pharyngeal pouch: protrusion of pharyngeal wall through Killian’s dehiscence, commonly affecting elderly men, presenting with dysphagia, regurgitation of food, gurgling sensation, halitosis, coughing and aspiration at night, posterior triangle.

How well did you know this?
1
Not at all
2
3
4
5
Perfectly
100
Q

A 3 month old baby presents with a lump on the base of his neck, posterior to the sternocleidomastoid. O/E it transilluminates brightly.

A. Pharyngeal pouch
B. Thyroglossal cyst
C. Sternocleidomastoid tumour
D Cystic hygroma
E. Carotid body tumour
F. Branchial cyst
G. Lymphoma
H. Thyroid goitre
I. Salivary duct carcinoma
J. Acoustic neuroma
A

Cystic Hygroma: congenital collection of lymphatic sacks (babies!), clear fluid (transilluminates), soft, fluctuant, posterior triangle

How well did you know this?
1
Not at all
2
3
4
5
Perfectly
101
Q

Elective AAA: select the most likely size that warrants elective repair provided the patient is fit for surgery.

A. Greater than 5.0 cm
B. Greater than 5.5 cm
C. Less than 5 cm
D. Greater than 4.5 cm
E. Less than 5.5 cm
A

Greater than 5.5 cm

Or:

  • Smaller aneurysm that is now causing symptoms (back pain)
  • Smaller aneurysm that is growing at a rate of > 1 cm/year

AAA’s

How well did you know this?
1
Not at all
2
3
4
5
Perfectly
102
Q

A 56 year old homeless man presents to A&E with severe pain in his right leg which started over 6 hours ago. O/E the right leg is pale compared to the left from below the knee to the toes. The right leg is cold, with absent popliteal, posterior tibial and dorsal is pedis pulses. There is no sensation in the right leg and the patient is unable to flex the toes due to fixed flexion deformities. Apyrexial, HR 85.
Which is the most appropriate management option?

A. Percutanous transluminal angioplasty
B. Revascularisation through endarterectomy
C. Revascularisation through bypass grafting
D. Thrombolysis
E. Amputation

A

Amputation: Acute Limb Ischaemia

  • Surgical emergency requiring revascularisation within 4-6 hours to save the limb, mainly due to in situ thromboses or emboli.

6 P’s: Pale, pulseless, painful, perishing cold, paraesthesia, paralysis.

Management options:

  • Percutaneous transluminal angioplasty: short stenoses in large arteries
  • Surgical bypass: extensive atheromatous disease with good distal run-off e.g. femoral politeal bypass, femoral distal bypass
  • Acutely: surgical embolectomy/thrombolysis
  • Amputation: Useless limb (e.g. fixed flexion deformity), Dead Limb (extensive necrosis), loss of sensation and paralysis point to gangrene
How well did you know this?
1
Not at all
2
3
4
5
Perfectly
103
Q

A 35 year old man presents to the emergency department with severe upper abdominal pain and blood stained vomit. On questioning he admits to a long history of dyspepsia that is worse when he is hungry and at night. Which of the following is the most likely diagnosis?

A. Arteriovenous malformation
B. Mallory-Weiss tear
C. Gastric Ulcer
D. Peutz-Jeghers syndrome
E. Duodenal ulcer
A

Duodenal ulcer: worse when hungry and at night

Gastric ulcer: worse when full

How well did you know this?
1
Not at all
2
3
4
5
Perfectly
104
Q

A 79 year old lady is treated for a pneumonia and develops severe diarrhoea and incontinence. Faecal analysis isolates a cytopathic toxin. The microbe involved in anaerobic.

A. Campylobacter jejuni
B. Clostridium difficile
C. Clostridium perfringens
D. Escherichia coli
E. Salmonella typhi
F. Staphylococcus aureus
G. Giardia lamblia
H. Streptococcus pyogenes
I. Bacillus cereus
J. Helicobacter pylori
A

Clostridium difficile: diarrhoea (can be bloody), pseudomembranous colitis, diagnosed by detection of toxin in stool.

Clues: Abx/hospital associated, produces a cytopathic toxin, gram positive anaerobe

Tx: metronidazole

Anaerobes = clostridium, bacteroides, actinomyces

How well did you know this?
1
Not at all
2
3
4
5
Perfectly
105
Q

A 45 year old woman develops diarrhoea and abdominal pain 2 days after eating fried chicken in a restaurant. A gram-stain of the faeces shows gram negative, motile, spiral shaped rods. Faecal culture reveals oxidase positive colonies.

A. Campylobacter jejuni
B. Clostridium difficile
C. Clostridium perfringens
D. Escherichia coli
E. Salmonella typhi
F. Staphylococcus aureus
G. Giardia lamblia
H. Streptococcus pyogenes
I. Bacillus cereus
J. Helicobacter pylori
A

Campylobacter jejuni: major cause of bloody diarrhoea, gram negative, oxidase positive, 2-6 days after exposure (BBQ)

Salmonella: diarrhoea, fever, abdo pain, normally at least 12 hours later, eggs, meat, poultry, gram negative

Staphylococcus aureus: very quick: 1-6 hours, gram positive clusters

How well did you know this?
1
Not at all
2
3
4
5
Perfectly
106
Q

A couple present to their GP with a one-week history of watery diarrhoea, and foul smelling belches after returning from a trip to Moscow. Stool microscopy reveals cysts and trophozoites.

A. Campylobacter jejuni
B. Clostridium difficile
C. Clostridium perfringens
D. Escherichia coli
E. Salmonella typhi
F. Staphylococcus aureus
G. Giardia lamblia
H. Streptococcus pyogenes
I. Bacillus cereus
J. Helicobacter pylori
A

Giardia lamblia: faeco-oral spread

Clues: travel to eastern europe/Russia, bloating, abdo pain, diarrhoea, foul smelling belches/flatus

Dx: direct fluorescent antibody (gold standard), cysts and trophozoites in the stool.
Tx: Metronidazole

Entamoeba histolytica: faeco-oral spread.

Clues: bloody diarrhoea (ameobic dysentry) - but has a slow onset of action,m liver abscess on USS, ‘flask shaped GI ulcers - RUQ pain

Dx: cysts and trophozoites in the stool

Tx: metronidazole

How well did you know this?
1
Not at all
2
3
4
5
Perfectly
107
Q

A 22 year old student presents to A&E with abdominal cramps, vomiting and watery diarrhoea. He has not eaten anything apart from Chinese take away from the night before. Stool microscopy confirms Gram positive bacilli.

A. Campylobacter jejuni
B. Clostridium difficile
C. Clostridium perfringens
D. Escherichia coli
E. Salmonella typhi
F. Staphylococcus aureus
G. Giardia lamblia
H. Streptococcus pyogenes
I. Bacillus cereus
J. Helicobacter pylori
A

Bacillus cereus: re-heated rice!!!! Very quick 1-6 hours, gram positive rod!

How well did you know this?
1
Not at all
2
3
4
5
Perfectly
108
Q

A 31 year old presents with an 8 month history of worsening dysphagia to solids, with occasional regurgitation of undigested food. He claims that dysphagia is not as bad if he eats a small amount of soft food, drinking lots of water with each bite. Barium swallow shows a ‘birds beak’ deformity in the lower oesophagus.

A. Achalasia
B. External oesophageal compression
C. Oesophageal candidiasis
D. Oesophageal carcinoma
E. Oesophageal diverticulum
F. Oesophageal peptic stricture
G. Pharyngeal pouch
H. Motor Neurone disease
I. Presbyoesophagus
J. Systemic Sclerosis
A

Achalasia: the lower oesophageal sphincter fails to relax due to degeneration of the myenteric plexus.

Symptoms: dysphagia (improves with smaller mouthfuls and with large volumes of liquids), retro-sternal cramps, regurgitation of undigested food.

Ix: Barium swallow - dilated tapering oesophagus (bird beak deformity)

Tx: Surgical (Heller’s cardiomyotomy), endoscopic balloon dilatation, botox, CCBs, nitrates

How well did you know this?
1
Not at all
2
3
4
5
Perfectly
109
Q

A 49 year old who has been in hospital with a severe infective exacerbation of his COPD complains of worsening dysphagia and retrosternal discomfort. OGD shows circumferential erosions and ulceration with white linear plaques.

A. Achalasia
B. External oesophageal compression
C. Oesophageal candidiasis
D. Oesophageal carcinoma
E. Oesophageal diverticulum
F. Oesophageal peptic stricture
G. Pharyngeal pouch
H. Motor Neurone disease
I. Presbyoesophagus
J. Systemic Sclerosis
A

Oesophageal candidiasis: immunosuppression (HIV, steroids)

Intraluminal: foreign objects (children and psych patients), oesophageal inflammation, oesophageal candidiasis

How well did you know this?
1
Not at all
2
3
4
5
Perfectly
110
Q

A 91 year old presents to Geriatrics outpatients with a 6 month history of worsening dysphagia to solids and liquids. A barium swallow reveals no intrinsic lesion but a “corkscrew” dysmotility.

A. Achalasia
B. External oesophageal compression
C. Oesophageal candidiasis
D. Oesophageal carcinoma
E. Oesophageal diverticulum
F. Oesophageal peptic stricture
G. Pharyngeal pouch
H. Motor Neurone disease
I. Presbyoesophagus
J. Systemic Sclerosis
A

Presbyoesophagus: corkscrew dymotility associated with old age

Extra-luminal:
- Intramural: benign/malignant strictures, achalasia, oesophageal web (Plummer-Vinson syndrome), Nutcracker oesophagus (high pressure on manometry with normal peristalsis), presbyoesophagus (corkscrew dysmotility associated with old age).

  • Extra-mural: retrosternal goitre, lung cancer, rolling hiatus, pharyngeal pouch
  • Systemic: myasthenia gravis, MND, Parkinson’s
How well did you know this?
1
Not at all
2
3
4
5
Perfectly
111
Q

A 54 year old Chinese man presents with dysphagia to solids and liquids. A barium swallow confirms a long, irregular stricture several cm long.

A. Achalasia
B. External oesophageal compression
C. Oesophageal candidiasis
D. Oesophageal carcinoma
E. Oesophageal diverticulum
F. Oesophageal peptic stricture
G. Pharyngeal pouch
H. Motor Neurone disease
I. Presbyoesophagus
J. Systemic Sclerosis
A

Oesophageal peptic stricture

How well did you know this?
1
Not at all
2
3
4
5
Perfectly
112
Q

A 62 year old presents with a 3 month history of worsening dysphagia with nasal regurgitation. She has now developed speech problems and limb weakness.

A. Achalasia
B. External oesophageal compression
C. Oesophageal candidiasis
D. Oesophageal carcinoma
E. Oesophageal diverticulum
F. Oesophageal peptic stricture
G. Pharyngeal pouch
H. Motor Neurone disease
I. Presbyoesophagus
J. Systemic Sclerosis
A

Motor Neuron Disease:

How well did you know this?
1
Not at all
2
3
4
5
Perfectly
113
Q

A 25 year old gentleman who was previously well presented with a 1 week history of SOB, fever and a productive cough. O/E there was dullness to percussion and increased tactile vocal fremitus in the right base.

A. Amoxicillin
B. Clarithromycin
C. Flucloxacillin
D. Metronidazole
E. Co-trimoxazole
F. Cefuroxime
G. Gentamicin
H. Ciprofloxacin
I. Chloramphenicol
J. Nitrofurantoin
A

Amoxicillin: strep pneumonia - penicillin, amoxicillin

How well did you know this?
1
Not at all
2
3
4
5
Perfectly
114
Q

A 38 year old prostitute presents with a 1 week history of a dry cough and SOB on exertion. CXR normal.

A. Amoxicillin
B. Clarithromycin
C. Flucloxacillin
D. Metronidazole
E. Co-trimoxazole
F. Cefuroxime
G. Gentamicin
H. Ciprofloxacin
I. Chloramphenicol
J. Nitrofurantoin
A

Co-trimoxazole: Pneumocystis carinii - immunosuppressed e.g. HIV
Boat-shaped organisms on BAL with a silver stain

How well did you know this?
1
Not at all
2
3
4
5
Perfectly
115
Q

A 29 year old business man was visiting Edinburgh on a 3 day business trip. He has not been well for a week with muscle cramps and SOB, he also forgot his wife’s birthday which he never normally does. Sodium 128 mmol/L and potassium 4.6 mmol/L

A. Amoxicillin
B. Clarithromycin
C. Flucloxacillin
D. Metronidazole
E. Co-trimoxazole
F. Cefuroxime
G. Gentamicin
H. Ciprofloxacin
I. Chloramphenicol
J. Nitrofurantoin
A

Clarithromycin: atypical pneumonia (legionella and mycoplasma - macrolide: erthromycin, clarithromycin)

How well did you know this?
1
Not at all
2
3
4
5
Perfectly
116
Q

A 49 year old lady presents with a 1 week history of lower abdominal pain and pain on urination. She complains that she is finding her teaching job increasingly difficult as she has to leave the class and go urinate very often.

A. Amoxicillin
B. Clarithromycin
C. Flucloxacillin
D. Metronidazole
E. Co-trimoxazole
F. Cefuroxime
G. Gentamicin
H. Ciprofloxacin
I. Chloramphenicol
J. Nitrofurantoin
A

Nitrofurantoin - UTI, NICE guidelines: trimethropim/nitrofurantoin

How well did you know this?
1
Not at all
2
3
4
5
Perfectly
117
Q

This serological marker is used to measure immunity following hepatitis B immunisation.

A. AFP
B. Alanine aminotransferase
C. HepB core antibody cAb
D. HepB core antigen cAg
E. HepB e antigen eAg
F. HepB surface antibody sAb
G. HepB surface antigen sAg
H. HepB RNA titre
A

HepB surface antibody - marker of vaccination

How well did you know this?
1
Not at all
2
3
4
5
Perfectly
118
Q

This serological marker is the first to indicate acute hepatitis B infection.

A. AFP
B. Alanine aminotransferase
C. HepB core antibody cAb
D. HepB core antigen cAg
E. HepB e antigen eAg
F. HepB surface antibody sAb
G. HepB surface antigen sAg
H. HepB RNA titre
A

HepB surface antigen: can be detected within 4 weeks and is the earliest marker of infection.
If it is still detected 6 months later it indicates chronic infection

How well did you know this?
1
Not at all
2
3
4
5
Perfectly
119
Q

This serological marker would mark a chronic hepatitis B infection if still detected 6 months after the original infection.

A. AFP
B. Alanine aminotransferase
C. HepB core antibody cAb
D. HepB core antigen cAg
E. HepB e antigen eAg
F. HepB surface antibody sAb
G. HepB surface antigen sAg
H. HepB RNA titre
A

HepB surface antigen: can be detected within 4 weeks and is the earliest marker of infection.
If it is still detected 6 months later it indicates chronic infection

How well did you know this?
1
Not at all
2
3
4
5
Perfectly
120
Q

This serological marker indicates high infectivity in a chronic hepatitis B carrier.

A. AFP
B. Alanine aminotransferase
C. HepB core antibody cAb
D. HepB core antigen cAg
E. HepB e antigen eAg
F. HepB surface antibody sAb
G. HepB surface antigen sAg
H. HepB RNA titre
A

HepB e antigen: indicates high levels of infectivity and requires aggressive treatment.

How well did you know this?
1
Not at all
2
3
4
5
Perfectly
121
Q

This serological marker, if high, indicates low infectivity in a chronic hepatitis B carrier.

A. AFP
B. Alanine aminotransferase
C. HepB core antibody cAb
D. HepB core antigen cAg
E. HepB e antigen eAg
F. HepB surface antibody sAb
G. HepB surface antigen sAg
H. HepB RNA titre
A

HepB core antibody: marker of infection seen after 4 weeks.
High titres of HepBcAb and absent HepBeAg indicates low infectivity and disease activity

HepB core antigen is only found in the liver and can’t be detected in the blood. Indicates they were infected at some point: IgM is acute, IgG is chronic

How well did you know this?
1
Not at all
2
3
4
5
Perfectly
122
Q

A 30 year old woman presents to A&E with worsening stiffness in her hands, wrists and feet. She mentions that pain is particularly bad in the mornings. O/E there is a palpable spleen. Initial blood tests reveal a low neutrophil count and a raised CRP.

The most likely diagnosis is:
A. Felty's syndrome
B. Still's Disease
C. Caplan's Syndrome
D. Churg-Strauss Syndrome
E. Kawasaki's Disease
A

Felty’s syndrome: Triad of RA, splenomegaly and neutropenia

How well did you know this?
1
Not at all
2
3
4
5
Perfectly
123
Q

A 30 year old Turkish man presents to A&E with oral ulcers, genital ulcers and painful legs. O/E there are apthous ulcers in the mouth, genital ulceration, erythema nodosum over the shin. He is admitted and skin pathergy test is positive.

The most likely diagnosis is:
A. Henoch-Schonlein purpura
B. Lyme Disease
C. Berger's Disease
D. Caplan's Syndrome
E. Behcet's Disease
A

Behcet’s Disease: AI systemic vasculitis. Most common in Middle eastern, Turkish.

Signs and symptoms: oral and genital ulcers, mouth sores, inflammation of the eyes (uveitis), skin rashes (cutaneous lesions) and neuro features (encephalitis, CN lesions, confusion)

Dx: skin pathergy test positive (needle prick leads to papule formation)

Mx: steroids and immunosuppressives

How well did you know this?
1
Not at all
2
3
4
5
Perfectly
124
Q

A 42 year old man presents with a 2 month history of recurrent nosebleeds. More recently he started coughing up blood as well. O/E there is a deformity in the bridge of his nose.

Which of the following blood markers is most associated with his condition?

A. pANCA
B. cANCA
C. ANA
D. Anti-Ro
E. Anti-La
A

cANCA: Wegener’s Granulomatosis

Features: Triad of:

  • Upper airway disease (nosebleed, saddle shaped nose deformity, rhinitis, sinusitis)
  • Respiratory disease (pulmonary nodules, pulmonary haemorrhage), may cause haemoptysis
  • Renal disease: glomerulonephritis often with proteinuria/haematuria

Dx: cANCA
Mx: depends on severity, steroids +/- cyclophosphamide (severe disease)

How well did you know this?
1
Not at all
2
3
4
5
Perfectly
125
Q

An 85 year old lady presents with acute breathlessness and chest pain. An ECG shows an irregular rhythm and HR 190bpm.

Paroxysmal AF
Mobitz type 1
STEMI
LBBB
Stokes Adams attack
Mobitz type 2
Persistent AF
Complete Heart Block
Wolf Parkinson White Syndrome
Fast AF
Atrial flutter
Ventricular Tachycardia
NSTEMI
RBBB
A

Fast AF: no discernible p waves, affects over 1 million people in the UK, irregularly irregular rhythm, chaotic atrial excitation.

Tends to be older patients with palpitations, SOB, fatigue, syncope.

Paroxysmal: usually lasts 7 days, often requires cardioversion

Permanent: no hope of sinus rhythm.

Causes:

  • Cardiac: IHD, MV disease, HTN
  • Non Cardiac: Infections, thyrotoxicosis, PE, electrolyte depletion

Mx:

  • Rate control: Beta blockers
  • Rhythm control: Anti-arrhythmics
  • Ablation
How well did you know this?
1
Not at all
2
3
4
5
Perfectly
126
Q

A 36 year old man comes to your GP and complains of palpitations. HR 150bpm

Paroxysmal AF
Mobitz type 1
STEMI
LBBB
Stokes Adams attack
Mobitz type 2
Persistent AF
Complete Heart Block
Wolf Parkinson White Syndrome
Fast AF
Atrial flutter
Ventricular Tachycardia
NSTEMI
RBBB
A

Atrial flutter: narrow complex tachycardia, 250-350bpm, atrial rate often 300 or 150 (with 2:1 block)

ECG: saw toothed appearance - reflects atrial rate

Mx:

  • Acute: electrical cardioversion
  • Recurrent: catheter ablation
How well did you know this?
1
Not at all
2
3
4
5
Perfectly
127
Q

A 56 year old man with a previous MI has a broad complex tachycardia.

Paroxysmal AF
Mobitz type 1
STEMI
LBBB
Stokes Adams attack
Mobitz type 2
Persistent AF
Complete Heart Block
Wolf Parkinson White Syndrome
Fast AF
Atrial flutter
Ventricular Tachycardia
NSTEMI
RBBB
A

Ventricular Tachycardia: activation from ventricles, not SAN

RFs: previous MI, inherited syndromes

Medical emergency as VF is on horizon!

How well did you know this?
1
Not at all
2
3
4
5
Perfectly
128
Q

A 24 year old man complains of palpitations. His ECG shows evidence of pre-excitation.

Paroxysmal AF
Mobitz type 1
STEMI
LBBB
Stokes Adams attack
Mobitz type 2
Persistent AF
Complete Heart Block
Wolf Parkinson White Syndrome
Fast AF
Atrial flutter
Ventricular Tachycardia
NSTEMI
RBBB
A

Wolff-Parkinson White Syndrome: 1-3 per 1000, often benign, accessory pathway (Bundle of Kent)

ECG: delta wave

How well did you know this?
1
Not at all
2
3
4
5
Perfectly
129
Q

An 84 year old man has a HR 32 bpm. His ECG shows broad QRS complexes.

Paroxysmal AF
Mobitz type 1
STEMI
LBBB
Stokes Adams attack
Mobitz type 2
Persistent AF
Complete Heart Block
Wolf Parkinson White Syndrome
Fast AF
Atrial flutter
Ventricular Tachycardia
NSTEMI
RBBB
A

Complete/3rd degree Heart Block: complete dissociation between atria and ventricles

Pacemaker potentials:

AV - 50bpm
Ventricle - 30bpm

1st degree: fixed prolonged PR interval
Mobitz Type 1: progressive prolonging of PR interval until QRS is dropped, next PR is short then cycle continues (Wenckebach phenomenon)
Mobitz Type 2: Fixed prolonged PR with occasional dropped QRS
3rd degree: no association between p waves and QRS complexes

How well did you know this?
1
Not at all
2
3
4
5
Perfectly
130
Q

A 54 year old diabetic presents with breathlessness and central crushing chest pain.

Best investigation:
Troponin T
Troponin I
TTE
Electrophysiology study
Coronary Angiography
ECG
Primary percutaneous coronary intervention
Exercise ECG
TOE
Holter monitor
BP
BNP
Stress Echo
CT calcium scoring
A

ECG

How well did you know this?
1
Not at all
2
3
4
5
Perfectly
131
Q

A 65 year old man complains of fever and back pain 4 weeks after having a prosthetic valve implanted.

Best investigation:
Troponin T
Troponin I
TTE
Electrophysiology study
Coronary Angiography
ECG
Primary percutaneous coronary intervention
Exercise ECG
TOE
Holter monitor
BP
BNP
Stress Echo
CT calcium scoring
A

TOE: Infective endocarditis, DUKES criteria

Suspicion of IE + prosthetic valve or suspicion of complication of virulent organisms or negative TTE but high suspicion = TOE

Otherwise TTE!
TOE has better resolution, but can require LA or sedative.

TTE has a lower sensitivity - if negative disease may still be present.

Both have a high specificity - if positive it is highly likely that disease is present.

How well did you know this?
1
Not at all
2
3
4
5
Perfectly
132
Q

A 72 year old man complains that his shoes have become too small for him. He also admits to worsening exertional dyspnoea, he says he’s never had any heart problems.

Best investigation:
Troponin T
Troponin I
TTE
Electrophysiology study
Coronary Angiography
ECG
Primary percutaneous coronary intervention
Exercise ECG
TOE
Holter monitor
BP
BNP
Stress Echo
CT calcium scoring
A

BNP - Heart failure: dyspnoea, orthopnoea, PND, ankle oedema

If previous MI - stress echo
No Mi - measure BNP

How well did you know this?
1
Not at all
2
3
4
5
Perfectly
133
Q

A 66 year old woman has developed a pan systolic murmur after a recent MI.

Best investigation:
Troponin T
Troponin I
TTE
Electrophysiology study
Coronary Angiography
ECG
Primary percutaneous coronary intervention
Exercise ECG
TOE
Holter monitor
BP
BNP
Stress Echo
CT calcium scoring
A

TTE

TOE has better resolution, but can require LA or sedative.

TTE has a lower sensitivity - if negative disease may still be present.

Both have a high specificity - if positive it is highly likely that disease is present.

How well did you know this?
1
Not at all
2
3
4
5
Perfectly
134
Q

A 55 year old hyperlipidaemic man complains of central chest pain on exertion that is relieved by rest.

Best investigation:
Troponin T
Troponin I
TTE
Electrophysiology study
Coronary Angiography
ECG
Primary percutaneous coronary intervention
Exercise ECG
TOE
Holter monitor
BP
BNP
Stress Echo
CT calcium scoring
A

Coronary angiography: to rule in, for high risk patients, presenting with typical pain

Intermediate likelihood - low risk factors, atypical pain and women = Functional imaging (stress echo, MPS with SPECT, cardiac MR)

Low likelihood - to rule out disease, low risk factors, atypical pain

How well did you know this?
1
Not at all
2
3
4
5
Perfectly
135
Q

A 26 year old diabetic female is brought into A&E by LAS. She doesn’t know where she is and is clutching her stomach. You notice her eyes are sunken in.

Management:
Lisinopril
Subcut Insulin
Sulfonylurea
Metformin
Thiazolidinedione
Amlodipine
Metoprolol
Lucozade
Lifestyle advice
IV saline
IV insulin
IM glucagon
Exenatide
A

IV saline - DKA
Characterised by hyperglycaemia, ketoacidosis, ketonuria.

Complete lack of insulin. Increase in glucagon, cortisol, GH, epinephrine. Stimulates hepatic gluconeogenesis, glycogenolysis, lipolysis.
The lack of insulin leads to fatty acid metabolism, leading to an increased ketone production

Presentation: N&V (hydroxybutyrate), sweet smelling breath (acetone), severe dehydration (glycosuria, osmotic diuresis), Ketoacidosis (ketones), confusion (hyperosmolarity, dehydration, acidosis), diffuse abdo pain.

Mx: IV normal saline to correct dehydration, insulin infusion, correct hypokalaemia

How well did you know this?
1
Not at all
2
3
4
5
Perfectly
136
Q

A 54 year old diabetic man has a HbA1C of 8.5% 6 weeks after being started on Metformin.

Management:
Lisinopril
Subcut Insulin
Sulfonylurea
Metformin
Thiazolidinedione
Amlodipine
Metoprolol
Lucozade
Lifestyle advice
IV saline
IV insulin
IM glucagon
Exenatide
A

Sulfonylurea

DM Tx:
HbA1C > 6.5%  
1. Metformin (or sulfonylurea if not overweight e.g. glibenclamide)
2. Metformin + sulfonylurea
3. Add thiazolidinedione (pioglitazone)
4. Insulin, metformin and sulfonylurea
How well did you know this?
1
Not at all
2
3
4
5
Perfectly
137
Q

A 65 year old white diabetic man has a BP of 155/85 on 2 visits to your GP surgery.

Management:
Lisinopril
Subcut Insulin
Sulfonylurea
Metformin
Thiazolidinedione
Amlodipine
Metoprolol
Lucozade
Lifestyle advice
IV saline
IV insulin
IM glucagon
Exenatide
A

Amlodipine - CCB

How well did you know this?
1
Not at all
2
3
4
5
Perfectly
138
Q

A 67 year old diabetic lady on MAU suddenly becomes aggressive and starts shouting at the nurses for imprisoning her. She calms down and after a few minutes appears to be falling asleep.

Management:
Lisinopril
Subcut Insulin
Sulfonylurea
Metformin
Thiazolidinedione
Amlodipine
Metoprolol
Lucozade
Lifestyle advice
IV saline
IV insulin
IM glucagon
Exenatide
A

IM glucagon

Hypoglycaemia: ABCDEFG!!!

  • Neuroglycopenia: fatigue and drowsiness, aggression, dizziness, confusion, LOC
  • Headaches
  • Nausea
  • Palpitation
  • Speech difficulty

Mx: If conscious, orientated and able to swallow = Lucozade
Otherwise IV glucose / IM glucagon

How well did you know this?
1
Not at all
2
3
4
5
Perfectly
139
Q

A 78 year old lady attempted to commit suicide by overdosing on her prescription medications. After a change of heart she presented to A&E. She is fatigued, bradycardic and can see yellow-green halos.

Treatment of choice:
Flumezanil
Haemodialysis
Naloxone
Activated charcoal
Protamine sulphate
Lorazepam
Acetyl-cysteine
Dimercaprol
Vitamin K
Reassure and discharge
Atropine
Digibind
Desferrioxamine
Alcohol
A
Digibind: Digoxin toxicity - affects cardiac ion channels, with the ultimate affect of increasing intracellular calcium. 
- Increased contractility
- Increased vagal tone to heart 
- AVN block: bradycardia
- Enhanced automaticity: arrhythmias (SVTs), tachycardia
- Xanthopsia: yellow-green visual halos
- Nausea and anorexia
Mx: Digibind
How well did you know this?
1
Not at all
2
3
4
5
Perfectly
140
Q

A 14 year old girl was brought in by her mother after an intentional paracetamol overdose. She refuses to say how many tablets she’s taken.

Treatment of choice:
Flumezanil
Haemodialysis
Naloxone
Activated charcoal
Protamine sulphate
Lorazepam
Acetyl-cysteine
Dimercaprol
Vitamin K
Reassure and discharge
Atropine
Digibind
Desferrioxamine
Alcohol
A

Acetylcysteine: Paracetomol OD is a primary cause of liver failure.
In the first few hours it presents as nausea and vomiting.
Hepatotoxicity occurs 24-48 hours later: hepatic tenderness, jaundice, asterixis, fetor hepaticus, haemorrhage. (If above 150mg/kg, if less than 75mg/kg no hepatotoxicity occurs)
N-acetly cysteine is most effective in first 8 hours.
If under 1 hour since ingestion = activated charcoal, measure levels and use treatment graph

How well did you know this?
1
Not at all
2
3
4
5
Perfectly
141
Q

A 16 year old girl is brought in by ambulance after getting drunk and taking her mothers ‘anxiety medication.’ Her mother says the medications name ends in -pam. The girl’s RR is 6 per minute.

Treatment of choice:
Flumezanil
Haemodialysis
Naloxone
Activated charcoal
Protamine sulphate
Lorazepam
Acetyl-cysteine
Dimercaprol
Vitamin K
Reassure and discharge
Atropine
Digibind
Desferrioxamine
Alcohol
A

Flumezanil: benzodiapine overdose (lorazepam, diazepam, midazolam) - bind to inhibitory GABAa receptors. A solitary overdose causes sleepiness, but with alcohol causes severe respiratory depression

How well did you know this?
1
Not at all
2
3
4
5
Perfectly
142
Q

A 4 year old child is brought into A&E with his 15 year old brother. The 15 year old notes that his younger brother drank a bottle of his mother’s bright green medication. The 4 year old has respiratory depression and pinpoint pupils.

Treatment of choice:
Flumezanil
Haemodialysis
Naloxone
Activated charcoal
Protamine sulphate
Lorazepam
Acetyl-cysteine
Dimercaprol
Vitamin K
Reassure and discharge
Atropine
Digibind
Desferrioxamine
Alcohol
A

Naloxone: Opiate intoxication! Bright green liquid - methadone
Causes respiratory depression and pinpoint pupils. Treatment is IV naloxone

How well did you know this?
1
Not at all
2
3
4
5
Perfectly
143
Q

A 75 year old man has overdosed on prescription medication. An ABG shows a mixed respiratory alkalosis and metabolic acidosis. As you return from the ABG machine you notice he is having a seizure.

Treatment of choice:
Flumezanil
Haemodialysis
Naloxone
Activated charcoal
Protamine sulphate
Lorazepam
Acetyl-cysteine
Dimercaprol
Vitamin K
Reassure and discharge
Atropine
Digibind
Desferrioxamine
Alcohol
A

Haemodialysis: salicyclate poisioning!
Presentation: vominting, dehydration, hyperventilation, tinnitus, vertigo, metabolic acidosis, mixed resp alk and metabolic acidodis.

Mx: Activated charcoal ASAP within first 2 hours. Urinary alkalisation (IV sodium bicarbonate).
Haemodialysis if: renal failure, heart failure or seizures.

Others:

  • Organophosphorous insecticides: farmer, salivation, lacrimation, urination, diarrhoea - Tx: Atropine (and pralidoxime if severe)
  • Heparin: bleeding. Tx: Protamine
  • Warfarin: bleeding. Tx: Vitamin K
  • Beta blockers: severe bradycardia/hypotension. Tx: atropine and glucagon
  • Carbon monoxide: low sats but normal blood gas, drowsy. Tx: 100% oxygen hyperbaric chamber
  • Paraquat: weed killers, oral ulcers, alveolitis, renal failure. Tx: activated charcoal
How well did you know this?
1
Not at all
2
3
4
5
Perfectly
144
Q

A 68 year old ex-smoker complains of SOB on exertion which has been getting worse over the past few years. He is frequently wheezy and has a productive cough.

Diagnosis:
Asthma
CF
CAP
Idiopathic Pulmonary Fibrosis
Pneumoconiosis
TB
PE
Cor pulmonale
COPD
Pneumothorax
Small cell carcinoma
Alpha-1 anti-trypsin deficiency
Mesothelioma
Extrinsic Allergic Alveolitis
A

COPD: a combination of emphysema and chronic bronchitis. Over 5 million people in the UK have it.
Emphysema: destruction of alveolar airspace leading to a reduction in surface for gas exchange.
Chronic Bronchitis: Productive cough in winter months.
Investigations: post-bronchodilator spirometry
Presentation: smoker/ex-smoker, wheeze, chronic cough, regular sputum production, exertional SOB, frequent winter ‘bronchitis’

Whereas asthma: may or may not smoke, wheeze, chronic non-productive cough, reversible, night time waking with SOB/wheeze, significant diurnal variation, FH atopy

How well did you know this?
1
Not at all
2
3
4
5
Perfectly
145
Q

A 55 year old man presents with a 2 month history of fever, cough and weight loss. A Ziehl-Nielsen stain confirms the diagnosis.

Diagnosis:
Asthma
CF
CAP
Idiopathic Pulmonary Fibrosis
Pneumoconiosis
TB
PE
Cor pulmonale
COPD
Pneumothorax
Small cell carcinoma
Alpha-1 anti-trypsin deficiency
Mesothelioma
Extrinsic Allergic Alveolitis
A

TB: mycobacterium family, cough for >2 weeks, occasional haemoptysis/chest pain, systemic symptoms (fever, night sweats, anorexia, weight loss), racial profiling.
Ix: CXR (cavitatory nodules), Sputum samples, Culture (lowenstein-jensen medium: 4-6 weeks), microscopy.
Ziehl-Neelsen stain: acid fast bacilli (rods) - most things turn blue due to the acid, whereas the acid fast aren’t affected and remain red.
Tx: Rifampicin, Isoniazid, Pyrazinamide (pyridoxine to prevent peripheral neuropathy), Ethambutol

How well did you know this?
1
Not at all
2
3
4
5
Perfectly
146
Q

A 24 year old man has come to your GP surgery for the 3rd time this winter with a mucopurelent cough. He also complains of general SOB, steatorrhoea, and you find his fingers are clubbed.

Diagnosis:
Asthma
CF
CAP
Idiopathic Pulmonary Fibrosis
Pneumoconiosis
TB
PE
Cor pulmonale
COPD
Pneumothorax
Small cell carcinoma
Alpha-1 anti-trypsin deficiency
Mesothelioma
Extrinsic Allergic Alveolitis
A

CF: AR, 1 in 22 caucausians are carriers, defective chloride excretion leads to viscous mucus. Predisposing to infections, causing bronchiectasis, steatorrhoea (pancreatic insufficiency), diabetes, infertility.

Brochiectasis: permanent dilatation of bronchioles following recurrent infections due to elastic damage. Clubbing, Kartageners, colliery dyskinesias, infertility, diabetes

How well did you know this?
1
Not at all
2
3
4
5
Perfectly
147
Q

You examine a 82 year old retired ship builder with worsening SOB. He has clubbing and on auscultation you hear fine-end inspiratory crackles.

Diagnosis:
Asthma
CF
CAP
Idiopathic Pulmonary Fibrosis
Pneumoconiosis
TB
PE
Cor pulmonale
COPD
Pneumothorax
Small cell carcinoma
Alpha-1 anti-trypsin deficiency
Mesothelioma
Extrinsic Allergic Alveolitis
A

Asbestosis: occupational exposure affecting lung parenchyma. Asbestos inhalation causes impaction leading to pulmonary fibrosis, a progressive disorder, manifesting with worsening SOB, finger clubbing, basal end-inspiratory crackles.
No treatment!

In coal workers: pneumoconiosis

Idiopathic pulmonary fibrosis: (cryptogenic fibrosing alveolitis and usual interstitial pneumonia, presenting with worsening SOB, non-productive cough, fine end-inspiratory crackles

Silicosis: silicon dioxide - stone masons, sand blaster, ceramic/pottery workers

How well did you know this?
1
Not at all
2
3
4
5
Perfectly
148
Q

A 29 year old farmer presents with worsening cough, SOB and flu-like symptoms which he’s had each winter for the past 3 years.

Diagnosis:
Asthma
CF
CAP
Idiopathic Pulmonary Fibrosis
Pneumoconiosis
TB
PE
Cor pulmonale
COPD
Pneumothorax
Small cell carcinoma
Alpha-1 anti-trypsin deficiency
Mesothelioma
Extrinsic Allergic Alveolitis
A

Extrinsic Allergic Alveolitis: hypersensitivity reaction - can be acute (4-6hours - fever, malaise, cough, SOB, chest tightness) or subacute/chronic (insidious onset, SOB, cough, fatigue, weight loss)
Dx: specific radiological features, histology

How well did you know this?
1
Not at all
2
3
4
5
Perfectly
149
Q

A 75 year old man complains of SOB and weight loss over a 3 month period, you notice his face is oedematous and flushed.

Diagnosis:
SIADH
Pancoast Tumour
Large cell carcinoma
Myasthenia Gravis
SVCO
PE
Malignant mesothelioma
Lambert Eaton Syndrome
Small cell carcinoma
Thymoma
Squamous cell carcinoma
Brain Metastases
Adenocarcinoma
Pleural effusion
A

SVCO: SVC drains head, neck, upper extremities and thorax, tumour can occlude SVC causing flushing of face, oedema of face and arms, dilated veins over arms, neck and chest, morning headaches.

Lung Cancer:

  • Small cell carcinoma: small and dangerous
  • Non small cell carcinoma: adenocarcinoma (glandular tissue), squamous cell carcinoma (epithelial cells lining airways), large cell carcinoma, large cell neuroendocrine

General Presentation: cough, weight loss, SOB, smoking! haemoptysis, lethargy.
Moderately frequent: bone pain, digital clubbing, fever
Infrequent: pain radiating down arm (Pancoast), SVCO, dysphagia, wheezing, stridor
Metastatic symptoms: brain, bone, liver, lymph nodes

How well did you know this?
1
Not at all
2
3
4
5
Perfectly
150
Q

A 68 year old female presents with shooting pain down her right arm and a droopy eyelid, on questioning she admits to weight loss and episodes of haemoptysis.

Diagnosis:
SIADH
Pancoast Tumour
Large cell carcinoma
Myasthenia Gravis
SVCO
PE
Malignant mesothelioma
Lambert Eaton Syndrome
Small cell carcinoma
Thymoma
Squamous cell carcinoma
Brain Metastases
Adenocarcinoma
Pleural effusion
A

Pancoast Tumour:

  • Brachial plexus invasion C8-T1 leading to wasting of intrinsic muscles of the hands, pain and paraesthesia in arm
  • Cervical Sympathetic plexus
  • Horner’s syndrome: ptosis, miosis, anhydrosis
How well did you know this?
1
Not at all
2
3
4
5
Perfectly
151
Q

A 54 year old man complains of weight loss and worsening cough. He also has a headache which is worse in the morning and when bending over.

Diagnosis:
SIADH
Pancoast Tumour
Large cell carcinoma
Myasthenia Gravis
SVCO
PE
Malignant mesothelioma
Lambert Eaton Syndrome
Small cell carcinoma
Thymoma
Squamous cell carcinoma
Brain Metastases
Adenocarcinoma
Pleural effusion
A

Brain Metastases: SOL

Other locations:
Liver: anorexia, nausea, weight loss, RUQ pain
Bone: pain, fractures
Adrenals: usually asymptomatic

152
Q

A 74 year old woman with SOB and tiredness complains of lower back pain. She has been getting very constipated and her mood has been low.

Diagnosis:
SIADH
Pancoast Tumour
Large cell carcinoma
Myasthenia Gravis
SVCO
PE
Malignant mesothelioma
Lambert Eaton Syndrome
Small cell carcinoma
Thymoma
Squamous cell carcinoma
Brain Metastases
Adenocarcinoma
Pleural effusion
A

Squamous cell carcinoma: 35% of all lung cancers, arises from epithelial cells, releases PTHrp, which has the same actions as PTH (bone resorption and tubular calcium reabsorption) causing hypercalcaemia.

Hypercalcaemia: bones, stones (renal or biliary), groans (constipation, nausea, pancreatitis), psychic moans (lethargy, depression, confusion)

153
Q

A 64 year old lifelong smoker presents with recent haemoptysis and dyspnoea. On examination he has truncal obesity and a moon face.

Diagnosis:
SIADH
Pancoast Tumour
Large cell carcinoma
Myasthenia Gravis
SVCO
PE
Malignant mesothelioma
Lambert Eaton Syndrome
Small cell carcinoma
Thymoma
Squamous cell carcinoma
Brain Metastases
Adenocarcinoma
Pleural effusion
A

Small cell carcinoma: 20% of all lung cancers, 2 year survival

154
Q

A 24 year old lady presents with a breast lump. O/E you find a discrete 1.5cm smooth mobile lump.

Diagnosis:
Breast abscess
Fibroadenoma
Invasive breast carcinoma
Gynaecomastia
Phyllodes tumour
Cyclical breast pain
Fibrocystic disease
Ductal carcinoma in situ
Fat necrosis
Acute mastitis
Duct ectasia
Sclerosing adenosis
Prolactinoma
Periductal Mastitis
A

Fibroadenoma: benign, most common breast lump in under 30s, arises from stroma, hard, smooth, mobile, breast mouse!

Fibroadenosis: fibrocystic change - lumpy bumpy breast with multiple nodules and cysts, oestrogen induced hyperplasia of glands and stroma so follows the menstrual cycle, sometimes pain.
Occurs due to small cysts: dilation and unfolding of the lobules, rupture of the cysts leads to bumps and firmness. There is then an increase in the number of acini per lobule -> lumpy bumpy breast changes associated with menstruation

155
Q

A 34 year old woman complains of a large lump which is hot and painful. She gave birth to a boy 6 weeks ago.

Diagnosis:
Breast abscess
Fibroadenoma
Invasive breast carcinoma
Gynaecomastia
Phyllodes tumour
Cyclical breast pain
Fibrocystic disease
Ductal carcinoma in situ
Fat necrosis
Acute mastitis
Duct ectasia
Sclerosing adenosis
Prolactinoma
Periductal Mastitis
A

Acute mastitis: infection of the breast, often in the first month of breast feeding (S.aureus or Strep.pyogenes). Inflammation, pus, sometimes fevers.

When an abscess forms there is more inflammation and more fever.

156
Q

A 56 year old woman complains of white nipple discharge. O/E you find a poor defined periareolar mass.

Diagnosis:
Breast abscess
Fibroadenoma
Invasive breast carcinoma
Gynaecomastia
Phyllodes tumour
Cyclical breast pain
Fibrocystic disease
Ductal carcinoma in situ
Fat necrosis
Acute mastitis
Duct ectasia
Sclerosing adenosis
Prolactinoma
Periductal Mastitis
A

Duct ectasia: dilation, blockage, chronic inflammation and fibrosis

Periductal mastitis: painful erythematous subarealor mass (poorly defined), no discharge, >90% are smokers, no age association

157
Q

A 31 year old woman experiences pain in her breast around the same time each month.

Diagnosis:
Breast abscess
Fibroadenoma
Invasive breast carcinoma
Gynaecomastia
Phyllodes tumour
Cyclical breast pain
Fibrocystic disease
Ductal carcinoma in situ
Fat necrosis
Acute mastitis
Duct ectasia
Sclerosing adenosis
Prolactinoma
Periductal Mastitis
A

Cyclical breast pain: breast pain associated with menstrual cycle (pain in luteal phase)

158
Q

A 52 year old woman presents with an irregular lump on her left breast and an inverted nipple. She also complains of unilateral bloody discharge and you notice some skin puckering.

Diagnosis:
Breast abscess
Fibroadenoma
Invasive breast carcinoma
Gynaecomastia
Phyllodes tumour
Cyclical breast pain
Fibrocystic disease
Ductal carcinoma in situ
Fat necrosis
Acute mastitis
Duct ectasia
Sclerosing adenosis
Prolactinoma
Periductal Mastitis
A

Invasive breast carcinoma: most common cancer in the UK, lifetime risk 1 in 8 for women.

Breast changes: ulceration, skin retraction, peau d’orange, swelling, nipple discharge, nipple eczema, nipple retraction or distortion.
Lump: hard, fixed, skin tethering

159
Q

A 65 year old woman complains of itchiness and discharge from her nipple. O/E you feel no lump.

Diagnosis:
Breast abscess
Fibroadenoma
Invasive breast carcinoma
Gynaecomastia
Phyllodes tumour
Cyclical breast pain
Fibrocystic disease
Ductal carcinoma in situ
Fat necrosis
Acute mastitis
Duct ectasia
Sclerosing adenosis
Prolactinoma
Periductal Mastitis
A

Ductal carcinoma in situ:

Paget’s disease of the breast: chronic eczematous change of the nipple: itching, erythema, scale formation, erosions, nipple discharge, occurs in 1-4% of breast cancers, 60% have invasive carcinoma, 40% have DCIS (can’t palpate lump)

160
Q

In breast clinic you examine a 32 year old woman with a 5cm unilateral irregular breast lump

Investigation:
Prolactin
Thyroid function tests
USS
CT
CXR
Ibuprofen
Reassure
MRI
Sentinel node biopsy
CA125
D-dimer
Mammography
Ultrasound and Mammogram
Abx therapy
A

USS: anyone under the age of 40
Mammogram: aged 40 or over
Both if lump and above 40!

161
Q

In breast clinic you examine a 56 year old woman with a 5cm unilateral irregular breast lump

Investigation:
Prolactin
Thyroid function tests
USS
CT
CXR
Ibuprofen
Reassure
MRI
Sentinel node biopsy
CA125
D-dimer
Mammography
Ultrasound and Mammogram
Abx therapy
A

USS and Mammography

USS: anyone under the age of 40
Mammogram: aged 40 or over
Both if lump and above 40!

162
Q

A 36 year old lady presents with a hot, red and painful breast lump.

Investigation:
Prolactin
Thyroid function tests
USS
CT
CXR
Ibuprofen
Reassure
MRI
Sentinel node biopsy
CA125
D-dimer
Mammography
Ultrasound and Mammogram
Abx therapy
A

Abx therapy

163
Q

A 47 year old lady has been experiencing white low viscosity liquid discharge from her nipples.

Investigation:
Prolactin
Thyroid function tests
USS
CT
CXR
Ibuprofen
Reassure
MRI
Sentinel node biopsy
CA125
D-dimer
Mammography
Ultrasound and Mammogram
Abx therapy
A

Prolactin: prolactinoma

164
Q

A 26 year old lady complains of painful breasts at the same time each month. O/E she has multiple hard lesions less than 2cm in diameter.

USS: anyone under the age of 40
Mammogram: aged 40 or over
Both if lump and above 40!

A

Ibuprofen

165
Q

A 77 year old lady has been experiencing SOB and chest pain on exertion for the last 6 months. O/E anacrotic pulse.

Aortic Sclerosis
Patent Ductus Arteriosus
Mitral valve prolapse
Mitral regurgitation
Aortic stenosis
Pulsus Paradoxus
Atrial Myxoma
Aortic regurgitation
Innocent murmur
Mitral stenosis
VSD
Mixed aortic valvular disease
Pericardial effusion
Prosthetic valve
Mixed valvular disease
HOCM
A

Aortic Stenosis: ejection systolic murmur radiating to the carotids, characteristic breathlessness, syncope, pain on exertion

166
Q

A 26 year old lady with a back ground of atypical chest pain presents to A&E with a MI and pink frothy sputum.

Aortic Sclerosis
Patent Ductus Arteriosus
Mitral valve prolapse
Mitral regurgitation
Aortic stenosis
Pulsus Paradoxus
Atrial Myxoma
Aortic regurgitation
Innocent murmur
Mitral stenosis
VSD
Mixed aortic valvular disease
Pericardial effusion
Prosthetic valve
Mixed valvular disease
HOCM
A

Mitral regurgitation: acute and chronic presentations

167
Q

A 35 year old IVDU was blue lighted to hospital with fever and night sweats. O/E you notice a pulsating uvula, osculating reveals a low pitched rumbling murmur head at the apex loudest on expiration.

Aortic Sclerosis
Patent Ductus Arteriosus
Mitral valve prolapse
Mitral regurgitation
Aortic stenosis
Pulsus Paradoxus
Atrial Myxoma
Aortic regurgitation
Innocent murmur
Mitral stenosis
VSD
Mixed aortic valvular disease
Pericardial effusion
Prosthetic valve
Mixed valvular disease
HOCM
A

Aortic regurgitation: multiple signs due to collapsing pulse, Austin Flint murmur

168
Q

A 30 year old lady with a background of SLE presents with chest pain. Ausculatation reveals muffled heart sounds.

Aortic Sclerosis
Patent Ductus Arteriosus
Mitral valve prolapse
Mitral regurgitation
Aortic stenosis
Pulsus Paradoxus
Atrial Myxoma
Aortic regurgitation
Innocent murmur
Mitral stenosis
VSD
Mixed aortic valvular disease
Pericardial effusion
Prosthetic valve
Mixed valvular disease
HOCM
A

Pericardial effusion

169
Q

A 22 year old athletic young man is seen in clinic with a harsh ejection systolic murmur accentuated by the Valsalva manoeuvre.

Aortic Sclerosis
Patent Ductus Arteriosus
Mitral valve prolapse
Mitral regurgitation
Aortic stenosis
Pulsus Paradoxus
Atrial Myxoma
Aortic regurgitation
Innocent murmur
Mitral stenosis
VSD
Mixed aortic valvular disease
Pericardial effusion
Prosthetic valve
Mixed valvular disease
HOCM
A

HOCM

170
Q

Mrs Jackson is a 80 year old lady with a background of IHD, who presents to A&E with a 4 month history of dyspnoea, orthopnoea and PND.
- O/E there are fine inspiratory basal crepitation. ECG shows no abnormalities but you suspect HF, what simple blood test could you do to confirm/rule this out?

CXR
Cardiac MRI
Thyroid function tests
ANP
BNP
Endothelin
ACEI
Adrenomedullin
ARB
Spironolactone
Thiazide diuretics
Beta blockers
Digoxin
TOE
TTE (Doppler)
Troponin I
A

BNP

171
Q

Mrs Jackson is a 80 year old lady with a background of IHD, who presents to A&E with a 4 month history of dyspnoea, orthopnoea and PND.
- Blood test is positive. What first line test would be performed if you suspected HF?

CXR
Cardiac MRI
Thyroid function tests
ANP
BNP
Endothelin
ACEI
Adrenomedullin
ARB
Spironolactone
Thiazide diuretics
Beta blockers
Digoxin
TOE
TTE (Doppler)
Troponin I
A

TTE (Doppler)

172
Q
Mrs Jackson is a 80 year old lady with a background of IHD, who presents to A&amp;E with a 4 month history of dyspnoea, orthopnoea and PND. 
- You decide to admit based on the history and positive findings from your examination and investigation. What class of drugs is 1st line treatment for chronic heart failure?
CXR
Cardiac MRI
Thyroid function tests
ANP
BNP
Endothelin
ACEI
Adrenomedullin
ARB
Spironolactone
Thiazide diuretics
Beta blockers
Digoxin
TOE
TTE (Doppler)
Troponin I
A

ACEI

173
Q

Mrs Jackson is a 80 year old lady with a background of IHD, who presents to A&E with a 4 month history of dyspnoea, orthopnoea and PND.
- After taking the medication for a while, Mrs Jackson complains of a persistent dry cough. What medication could replace 1st line for chronic heart failure?

CXR
Cardiac MRI
Thyroid function tests
ANP
BNP
Endothelin
ACEI
Adrenomedullin
ARB
Spironolactone
Thiazide diuretics
Beta blockers
Digoxin
TOE
TTE (Doppler)
Troponin I
A

ARB

174
Q

Mrs Jackson is a 80 year old lady with a background of IHD, who presents to A&E with a 4 month history of dyspnoea, orthopnoea and PND.
- 6 months down the line, Mrs Jackson visits her GP, still complaining of the same symptoms despite now being on enalapril, atenolol and furosemide. What additional treatment may be indicated in this patient?

CXR
Cardiac MRI
Thyroid function tests
ANP
BNP
Endothelin
ACEI
Adrenomedullin
ARB
Spironolactone
Thiazide diuretics
Beta blockers
Digoxin
TOE
TTE (Doppler)
Troponin I
A

Spironolactone

175
Q

An IVDU presents with a fever and general weight loss. Upon auscultation there is a pan systolic murmur loudest at the left sternal edge on inspiration. You suspect infective endocarditis.

What is the most likely causative organism?

Streptococcus viridans
Group B streptococcus
HACEK
Staphylococcus aureus
Enterococcus faecalis
A

Staphylococcus aureus

176
Q

Which of these is not a sign of infective endocarditis?

Microvascular haematuria
Janeway lesions
Petechia
Painful rash
Clubbing
A

Painful rash

177
Q

Which of these are not present in the diagnostic criteria for subacute bacterial endocarditis?

One positive blood culture
2 serial positive blood cultures
Persistently positive blood cultures
ECG abnormalities
Echocardiogram abnormalities
A

ECG abnormalities

Infective Endocarditis: associated with valves and procedures, fevers with a normal ECG

178
Q

A 22 year old lady has come in with a very low BP and very high HR. She has a rising fever and low HR. She has a rising fever and low GCS. Her boyfriend says she has recently been to the dentist.
What is your next step in management?

Oral Abx therapy
IV Abx therapy
M, C + S
IM Abx therapy
Surgery
A

M, C + S

179
Q

Unfortunately this woman is shown to have MRSA. The patient is placed in isolated and barrier nursing is implemented.
What is the single most appropriate management for this patient?

Conservativ management
IV Tazocin
IV Vancomycin
Oral Co-amixiclav
Oral metronidazole
A

IV Vancomycin

180
Q

A 5 year old boy presents to A&E with seizures and lymphadenopathy. Investigation reveals pancytopenia.

AML
Nodular sclerosing Hodgkin's lymphoma
Burkitt's Lymphoma
Hairy cell leukaemia
Follicular cell lymphoma
Mantle zone lymphoma
Mucosal Associated Lymphoid tissue lymphoma
Anaplastic large cell lymphoma
ABVD
R-CHOP
ALL
Chronic granulocytic leukaemia
CLL
CML
Acute Promyelocytic leukaemia
Co-trimoxazole
A

ALL - leukaemia of childhood (3-15) 80% 5 year survival if young.
Presents as kids with anaemia, lymphadenopathy, bone pain, easy bruising, multiple infections. CNS involvement can lead to headache, vomiting, meningism, CN palsy etc.
Ix: FBC - pancytopenia, increase in WCC
Immunophenotyping (risk stratification) TdT positive
LP - CNS infiltration
Dx: s are progeny of single transformed cells

181
Q

A 40 year old male presents to A&E with severe haemorrhage. Blood film shows schistocytes.

AML
Nodular sclerosing Hodgkin's lymphoma
Burkitt's Lymphoma
Hairy cell leukaemia
Follicular cell lymphoma
Mantle zone lymphoma
Mucosal Associated Lymphoid tissue lymphoma
Anaplastic large cell lymphoma
ABVD
R-CHOP
ALL
Chronic granulocytic leukaemia
CLL
CML
Acute Promyelocytic leukaemia
Co-trimoxazole
A

Acute Promyelocytic leukaemia: bloody emergency! High risk of bleeding, DIC, septic shock, hyper viscosity syndrome.
M2: t(8:21), M3 t(15:17)
Give ATRA or Arsenic Trioxide

182
Q

A 72 year old man has weight loss, fever and drenching night sweats, his symptoms get better with Imatinib.

AML
Nodular sclerosing Hodgkin's lymphoma
Burkitt's Lymphoma
Hairy cell leukaemia
Follicular cell lymphoma
Mantle zone lymphoma
Mucosal Associated Lymphoid tissue lymphoma
Anaplastic large cell lymphoma
ABVD
R-CHOP
ALL
Chronic granulocytic leukaemia
CLL
CML
Acute Promyelocytic leukaemia
Co-trimoxazole
A

CML: average age 60-70 years, philedelphia chromosome t9:22 carries a good prognosis! Bcr-abl fusion onco-protein expression of tyrosine kinase.
Typically an incidental finding in older males, splenomegaly, bleeding, bruising, gout hyper viscosity, retinal haemorrhage, ACD increased WCC, blood film with increasing blast cells, FISH analysis.
Mx: IMATINIB (if philadelphia positive and chronic phase) - tyrosine kinase inhibitor.
May transform in a blast crisis to AML

Hairy cell leukaemia: = subtype CLL TRAP positive

Marginal zone lymphoma: MALT - chronic antigen stimulation (H.pylori), associated with thyroiditis and Sjorgen’s. Tx: triple therapy

Mantle zone lymphoma: males, affects GIT, CD5 cyclin D1, Tx: R-CHOP

Lymphoplasmocytic: Waldenstrom’s macroglobulinaemia, MGUS, MM

183
Q

A 16 year old Afro-Carribbean child presents with a mass on her jaw. Blood film reveals a starry sky appearance.

AML
Nodular sclerosing Hodgkin's lymphoma
Burkitt's Lymphoma
Hairy cell leukaemia
Follicular cell lymphoma
Mantle zone lymphoma
Mucosal Associated Lymphoid tissue lymphoma
Anaplastic large cell lymphoma
ABVD
R-CHOP
ALL
Chronic granulocytic leukaemia
CLL
CML
Acute Promyelocytic leukaemia
Co-trimoxazole
A

Burkitt’s Lymphoma: Presents with a mass in the jaw/abdomen, B symptoms, associated with EBV, Africa and kids.
Starry sky appearance, associated with cmyc, be wary of tumour lysis syndrome when treating.

184
Q

A 51 year old man presents with pain upon drinking alcohol. Blood film shows binucleate Reed Sternberg cells. The haematology consultant decided to start him on radiotherapy in combination with his therapy.

AML
Nodular sclerosing Hodgkin's lymphoma
Burkitt's Lymphoma
Hairy cell leukaemia
Follicular cell lymphoma
Mantle zone lymphoma
Mucosal Associated Lymphoid tissue lymphoma
Anaplastic large cell lymphoma
ABVD
R-CHOP
ALL
Chronic granulocytic leukaemia
CLL
CML
Acute Promyelocytic leukaemia
Co-trimoxazole
A

ABVD - Hodgkin’s Lymphoma
Lymphoma = cancer of the lymph nodes, 80% non hodgkins, 20% hodgkins.
In general the more indolent the lymphoma, the worse prognosis it has!

Hodgkin’s= 1 site of swollen lymph nodes, spreads adjacently, M>F, Bimodal age distribution.
Classical: 20s, related to EBV, lymphocyte rich, mixed cellularity, lymphocyte depleted, bad prognosis
Nodular lymphocytic: predominantly 50s
Presentation: B symptoms, rubbery neck node, related to EBV, pain on drinking alcohol, tobacco intolerance, isolated lymphadenopathy.
Ix: Reed Sternberg Cells, owl’s eye inclusions, eosinophilia
Mx: ABVD and extensive radiotherapy (can cause pulmonary fibrosis, cardiomegaly, breast cancer and other secondaries, but fertility is preserved.) Allogenic SCT

Non-Hodgkins: B cell, low grade, follicular, elderly, incurable, indolent, treat symptomatically with R-CHOP

185
Q

A 60 year old man presents with multiple infections and easy bruising. Blood film shows Auer rods.

AML
Nodular sclerosing Hodgkin's lymphoma
Burkitt's Lymphoma
Hairy cell leukaemia
Follicular cell lymphoma
Mantle zone lymphoma
Mucosal Associated Lymphoid tissue lymphoma
Anaplastic large cell lymphoma
ABVD
R-CHOP
ALL
Chronic granulocytic leukaemia
CLL
CML
Acute Promyelocytic leukaemia
Co-trimoxazole
A

AML: 60-70 is mean age of presentation, short history of anaemia, easy bruising, multiple infections.
Also in very young babies (association with trisomy 21)

FBC: Pancytopenia
Blood film: Auer rods
BM aspirate > 20% blasts

Mx: Supportive, systemic chemotherapy via CVC, allogenic SCT

186
Q

A 41 year old lady presents with fever, abdo pain and nausea particularly in the RUQ. O/E you notice intense shaking and icteral sclera.
What is the single most likely diagnosis?

Haemolytic transfusion reaction
Acute Hepatitis
Isoniazid therapy
Gallstones
Haemolytic uraemic syndrome
A

Gall stones/biliary colic: contraction of the gall bladder against an obstruction, triggered by eating fatty foods. Presenting as recurrent severe pain pain radiating to the back, associated with nausea. Tender on inspiration in the RUQ (Murphy’s sign) - Female, fat, forties.
Ix: USS, ERCP Tx: Interventional elective cholecystectomy

If there is a fever and increased inflammatory markers = ascending cholangitis, Charcot’s triad = fever, rigours, jaundice. Most normally E.coli (give bacteria)

Jaundice= increase in serum bilirubin
Normal physiology:
1. Heme in rbis are broken down by macrophages in the spleen and BM to make unconjugated bilirubin
2. Conjugation occurs in the liver
3. Excreted via biliary tract -> ampulla of vater -> duodenum
4. Converted into urobilinogen in terminal ileum.
Excretion: kidneys (via blood), stercobilinogen (via faeces), reabsorbed and re-excreted via liver

Pre-hepatic: Unconjugated hyperbilirubinaemia due to haemolysis, hereditary spherocytosis, haemolytic transfusion reaction.
Hepatic: Mixed conjugated and unconjugated hyperbilirubinaemia due to problems with liver excretion; hepatitis, cirrhosis, drugs, tumours
Post-hepatic: conjugated hyperbilirubinaemia due to obstruction: gallstones, tumours, infection

187
Q

A 21 year old patient is being treated for severe burns. Whilst on ITU he becomes pyrexial and starts vomiting blood.

Diagnosis:
Gastric carcinoma
MALT
Peptic Ulcer
Barrett's oesophagus
Zollinger Ellison Syndrome
Oesophageal varices
Mallory Weiss tear
Pyloric Stenosis
Oesophageal Stricture
Duodenal Ulcer
Chronic erosive gastritis
Kaposi's sarcoma
Oesophageal carcinoma
Achalasia
Curling's Ulcer
Cushing's Ulcer
A

Curling’s Ulcer: due to burns

Zollinger-Ellison Syndrome: recurrent peptic ulceration, haematemesis. Gastrin-secreting tumour from the G cells of the pancreas (also in the stomach and duodenum) causing an increase in HCl in gastric antrum and so multiple ulcers are present throughout the GIT. Distal ulceration can cause diarrhoea.
Ix: serum gastrin is increased, ultrasonography, CT, angiography
Tx: Lansoprazole, surgical resection, 60% malignant - metastasize to the local lymph nodes/liver

188
Q

A 73 year old man has persistent unexplained dyspepsia. He comes to the GP with progressive problems swallowing and maleana.

Diagnosis:
Gastric carcinoma
MALT
Peptic Ulcer
Barrett's oesophagus
Zollinger Ellison Syndrome
Oesophageal varices
Mallory Weiss tear
Pyloric Stenosis
Oesophageal Stricture
Duodenal Ulcer
Chronic erosive gastritis
Kaposi's sarcoma
Oesophageal carcinoma
Achalasia
Curling's Ulcer
Cushing's Ulcer
A

Gastric carcinoma: acanthosis nigricans, vomiting fresh blood, microcytic anaemia, dyspepsia, nausea, anorexia, weight loss, satiety.
Persistent, unexplained dyspepsia: 2 week referral for endoscopy. CT/MRI staging: leather bottle stomach (rigid thick gastric wall), signet ring cells.
Tx: partial/complete gastretomy

Oesophageal carcinoma: progressive, rapid dysphagia, weight loss, cachexia, low serum protein.
Dx: oesophagoscopy, size of tumour assessed via barium swallow - apple core appearance

Achalasia: LOS failure, food regurgitation, dilated oesophagus, tapering, beak line

189
Q

A 56 year old patient is referred to the GI clinic with recent progressive anaemia and dyspepsia from his GP. Upon examination you find an area of hyper pigmented skin in the left axilla.

Diagnosis:
Gastric carcinoma
MALT
Peptic Ulcer
Barrett's oesophagus
Zollinger Ellison Syndrome
Oesophageal varices
Mallory Weiss tear
Pyloric Stenosis
Oesophageal Stricture
Duodenal Ulcer
Chronic erosive gastritis
Kaposi's sarcoma
Oesophageal carcinoma
Achalasia
Curling's Ulcer
Cushing's Ulcer
A

Gastric Carcinoma: acanthosis nigricans, vomiting fresh blood, microcytic anaemia, dyspepsia, nausea, anorexia, weight loss, satiety.
Persistent, unexplained dyspepsia: 2 week referral for endoscopy. CT/MRI staging: leather bottle stomach (rigid thick gastric wall), signet ring cells.
Tx: partial/complete gastretomy

Barretts’s Oesophagus:

  • Metaplasia of distal oesophagus mucosa; consequence of GORD
  • Dx with endoscopy, biopsy
  • Tx: regular surveillance - PPI/mucosal ablation
  • Can progress to gastric carcinoma
190
Q

A 32 year old patient complains of abdominal pain after meals. CLO test is positive.

Diagnosis:
Gastric carcinoma
MALT
Peptic Ulcer
Barrett's oesophagus
Zollinger Ellison Syndrome
Oesophageal varices
Mallory Weiss tear
Pyloric Stenosis
Oesophageal Stricture
Duodenal Ulcer
Chronic erosive gastritis
Kaposi's sarcoma
Oesophageal carcinoma
Achalasia
Curling's Ulcer
Cushing's Ulcer
A

Peptic Ulcer: pain immediately after meals, chronic peptic ulcer: sharp boundaries, fibrous scar tissue, inflammation, necrosis
Whereas duodenal ulcers: pain 2-3 hours after eating/at night
Malignant ulcer -> ulcerating, heaped up epithelial around crater, rolled, raised, everted

MALT -> B cell type lymphoma, intraepithelial involvement, thickened folds, atypical lymphocytes.

Kaposi’s Sarcoma: AIDS, HHV8. Purple plaqued lesions in fundus. Biopsy: slit-like vascular spaces, spindle cells.

191
Q

A 45 year old GI surgeon has been admitted for haematemesis after a stag night. INR is raised.

Diagnosis:
Gastric carcinoma
MALT
Peptic Ulcer
Barrett's oesophagus
Zollinger Ellison Syndrome
Oesophageal varices
Mallory Weiss tear
Pyloric Stenosis
Oesophageal Stricture
Duodenal Ulcer
Chronic erosive gastritis
Kaposi's sarcoma
Oesophageal carcinoma
Achalasia
Curling's Ulcer
Cushing's Ulcer
A

Oesophageal Varices: haematemesis plus macrocytic anaemia (direct effect on bone marrow), increase in INR (liver toxicity), chronic alcoholics, portal hypertension secondary to liver cirrhosis

Pyloric stenosis: projectile vomiting, succession splash, hypokalaemia, metabolic alkalosis (babies)

Oesophageal Stricture: impact pain, regurgitation (depending on degree of stricture and consistency of foods)
Dx: barium swallow; Tx: benign=PPI and dliatation; malignant= resection

Mallory-Weiss Tear: students, beer race, tear in mucosa at gastro-oesophageal junction due to repeated wrenching and vomiting. Blood normal. Self limiting.

Acute Erosive Gastritis: NSAIDS, alcohol, chemotherapy, stress. Presents with malaena and anaemia.

Burns -> Curling’s ulcer
Intracranial lesion -> cushing’s ulcer

192
Q

A 40 year old banker presents with LUQ pain radiating to the back, he is hypotensive and tachycardic. What is the single most likely diagnosis?

Bleeding duodenal ulcer
Acute pancreatitis
Acute cholecystitis
AAA
Acute Hepatitis
A

Acute Pancreatitis: alcohol abuse, epigastric pain, radiates to the back, relieved by sitting forwards.
Clinical dehydration leads to hypokalaemia, haemorrhage leads to anaemia
Ix: bloods and USS liver, WCC (inflammation - possible septic shock). Prognosis is by the modified glasgow criteria

193
Q

A 40 year old banker presents with LUQ pain radiating to the back, he is hypotensive and tachycardic. What is the single best marker of prognosis?

Serum amylase
ALT/AST
Venous oxygen
HDL
Serum Urea
A

Serum Urea: Modified Glasgow score

Amylase is not a good marker of severity but can be useful for dx if it is over 5 times normal amount!

194
Q

A 52 year old chronic alcohol presents with fullness, epigastric pain and nausea. He is jaundiced and has a palpable gall bladder. What is the diagnosis?

Gall stones
Pancreatic Pseudocyst
Pancreatic Carcinoma
Gastric carcinoma
Insulinoma
A

Pancreatic carcinoma: painless progressive jaundice or dull epigastric pain radiating to the back, weight loss, loss of appetite, cachexia. Awful prognosis - 2% alive at 5 years. RFs: DM, Smoking, lots of alcohol.
Couvoiser’s Law: if the gall bladder is palpable and the patient is jaundiced, obstruction of the bile duct is unlikely to be a stone (i.e. it’s a tumour). Older population, dark urine, pale stools, ALP>AST (obstructive jaundice), tumour marker CA19-9. Tx: Palliative, stenting, Whipple’s (pancreaticoduodenectomy = resection of head of pancreas + CBD + GB + distal stomach + duodenum)

Pancreatic pseudocyst: possible complication of pancreatitis, presents with symptoms of pancreatitis and fullness, with fever from infection and haemorrhage into the lesser sac.

Insulinoma: Pancreatic islet cell tumour, presents with hypoglycaemia upon fasting/exercise, mostly benign, palpitations, dizziness, fainting, diplopia, confusion

195
Q

A 75 year old woman presents with sudden onset pain in her right knee. O/E the knee is swollen, red and painful. Temp 36.8 degrees.

Ankylosing spondylitis
Enteropathic arthritis
Golfer's elbow
Gout
OA
Pseudogout
Psoriatic arthritis
Reiter's Syndrome
RA
Septic arthritis
Still's Disease
Tennis elbow
A

Pseudogout

Acute hot swollen joint: Trauma, septic arthritis, crystal arthropathies (gout, pseudo gout)
Joint aspiration should be done:
- Septic - may see pus, culture S.aureus, Streptococcus and Neisseria gonorrhoea (commonest organism)
- Crystal arthropathies: gout - needle shaped negatively bifringent under polarised light; pseudo gout - rhomboid shaped weakly positive bifringent under polarised light. Xray finding: chondrocalcinosis

196
Q

A 56 year old woman presents with pain in her hands and swelling of the fingers. The pain is mainly in the DIP joints. O/E the nails of the hand are pitted and appear to be lifted off the skin at the distal edges and she has a scaly rash.

Ankylosing spondylitis
Enteropathic arthritis
Golfer's elbow
Gout
OA
Pseudogout
Psoriatic arthritis
Reiter's Syndrome
RA
Septic arthritis
Still's Disease
Tennis elbow
A

Psoriatic arthritis: Asymetric and patchy involvement. Characteristic DIP involvement and dactylitis (sausage fingers). Affects 5% of those with psoriasis

Seronegative spondyoarthropathies: arthritis without RF, strongly associated with HLA-B27, most common in men: Psoriatic arthritis, ankylosing spondylitis, enteropathic arthritis, reactive arthritis

Enteropathic arthritis: Crohn’s and UC

197
Q

A 31 year old man comes to the clinic complaining of red and itchy eyes for the past 8 hours. The patient has had pain on urination and diffuse joint pain for 1 month, but tested negative for gonorrhoea and chlamydia on a previous visit 3 weeks ago. He has also tested negative for rheumatoid factor but he is HLA-B27 positive. PMH: 2 months ago bad case of diarrhoea.

Ankylosing spondylitis
Enteropathic arthritis
Golfer's elbow
Gout
OA
Pseudogout
Psoriatic arthritis
Reiter's Syndrome
RA
Septic arthritis
Still's Disease
Tennis elbow
A

Reiter’s Syndrome: Reactive arthritis: Post GI or chlamydia infections - conjunctivitis & anterior uveitis, urethritis, arthritis (can’t see, pee or climb a tree!)

198
Q

A 27 year old man sees his GP with a 6 month history of lower back pain and stiffness that wakes him up during the night and is worst in the morning. He has severe limitation of motion in his lumbar spine. Laboratory test results are negative for ANA & RF, but HLA-B27 positive.

Ankylosing spondylitis
Enteropathic arthritis
Golfer's elbow
Gout
OA
Pseudogout
Psoriatic arthritis
Reiter's Syndrome
RA
Septic arthritis
Still's Disease
Tennis elbow
A

Ankylosing spondylitis: disease of spine and sacroiliac joints - stiff due to spine fusion of joints (bamboo spine)
Extra-articular features: Anterior uveitis, achilles tendonitis, apical lung fibrosis, aortic incompetence, amyloidosis (kidneys)

199
Q

A 7 year old child from Nigeria presents to A&E screaming that the bones in her hand and leg are excruciatingly painful. Hb 90g/L. What is the most likely diagnosis?

Sickle Cell disease
B thalassaemia major
G6PD deficiency
Hereditary elliptocytosis
Paroxysmal nocturnal haemoglobinuria
Thrombotic thrombocytopenic purpura
Sickle Cell Trait
B thalassaemia minor
Hereditary Spherocytosis
Autoimmune haemolysis
Disseminated intravascular coagulation
Haemolytic uraemia syndrome
A

Sickle Cell Disease: abnormality in B-globin chain (glu ->Val at position 6), AR.
Sickling occurs in hypoxic conditions as Hb stick together in rbc (high altitude, anaesthesia, excessive exercise), dehydration (increased MCHC increases chances of sickling), infection&inflammation.
Dx: electrophoresis (neonatal screening in UK) - trait only show symptoms in extreme hypoxia

200
Q

A child is admitted to resus with septicaemia, her blood tests show platelets 30, APTT 60s, PT 20s and low fibrinogen levels.

Sickle Cell disease
B thalassaemia major
G6PD deficiency
Hereditary elliptocytosis
Paroxysmal nocturnal haemoglobinuria
Thrombotic thrombocytopenic purpura
Sickle Cell Trait
B thalassaemia minor
Hereditary Spherocytosis
Autoimmune haemolysis
Disseminated intravascular coagulation
Haemolytic uraemia syndrome
A

Disseminated intravascular coagulation:

Microangiopathic haemolytic anaemia:

  • Haemolytic uraemic syndrome: widespread activation of platelets (E.coli o157:H7 - Shiga toxin travels in blood) -> thrombocytopenia
  • Thrombotic thrombocytopenia purpura: widespread activation of platelets - genetic AI process: CNS abnormalities, renal pathology, thrombocytopenia, fever, microangiopathic haemolytic anaemia
  • DIC: widespread activation of coagulation system - common in obstetric complications, septicaemia’s, cancers (pancreatic carcinomas or AML). No thrombocytopenia unlike other two, but has raised D-dimers.
201
Q

A 24 year old teacher is diagnosed with an atypical pneumonia. The consultant asks the medical student why the patient has yellow sclera. After a prolonged period of silence the consultant helpfully adds that the patient has a positive Coombs test.

Sickle Cell disease
B thalassaemia major
G6PD deficiency
Hereditary elliptocytosis
Paroxysmal nocturnal haemoglobinuria
Thrombotic thrombocytopenic purpura
Sickle Cell Trait
B thalassaemia minor
Hereditary Spherocytosis
Autoimmune haemolysis
Disseminated intravascular coagulation
Haemolytic uraemia syndrome
A

Autoimmune Haemolytic Anaemia: Coombs positive - mycoplasma pneumonia -> cold agglutinin haemolytic anaemia

202
Q

A 28 year old Mediterranean couple go on their holiday to Namibia, however shortly into their trip the husband feels unwell and goes to the local hospital. They do a blood film and find bite cells and Heinz bodies.

Sickle Cell disease
B thalassaemia major
G6PD deficiency
Hereditary elliptocytosis
Paroxysmal nocturnal haemoglobinuria
Thrombotic thrombocytopenic purpura
Sickle Cell Trait
B thalassaemia minor
Hereditary Spherocytosis
Autoimmune haemolysis
Disseminated intravascular coagulation
Haemolytic uraemia syndrome
A

G6PD deficiency: 5-7% worlds population, XR, Mediterranean and African populations.
Presents with oxidative crisis following infection, drugs (primaquine, quinine) and lava beans. Symptoms usually 2-3 days following oxidative stress and are self limiting

203
Q

A northern European boy is referred to a haematologist after his GP felt an enlarged spleen and a subsequent FBC showed a mild anaemia. Osmotic fragility test was positive and Coombs test was negative. What condition does he have?

Sickle Cell disease
B thalassaemia major
G6PD deficiency
Hereditary elliptocytosis
Paroxysmal nocturnal haemoglobinuria
Thrombotic thrombocytopenic purpura
Sickle Cell Trait
B thalassaemia minor
Hereditary Spherocytosis
Autoimmune haemolysis
Disseminated intravascular coagulation
Haemolytic uraemia syndrome
A

Hereditary spherocytosis: 1 in 5000, commonest in northern Europe, AD in 75% of cases. Osmotic fragility test - RBC’s burst in hypotonic solution

204
Q

Which of these TB drugs is associated with colour blindness?

Pyrazinamide
Isoniazid
Ethambutol
Rifampicin
Azithromycin
Moxifloxacin
A

Ethambutol

Standard TB Tx: Rifampicin, Isoniazid, Pyrazinamide, Ethambutol for first 2 months.
Next 4 months just Rifampicin and Isoniazid
Main side effects:
- Isoniazid: peripheral neuropathy (give pyridoxine (vitB6) to avoid)
- Rifampicin: Hepatitis, orange discolourations of secretions, drug interactions
- Pyrazinamide: hepatotoxicity, hyperuricaemia
- Ethambutol: optic neuritis - colour is first to be affected

205
Q

A 32 year old British man develops profuse watery diarrhoea with abdominal cramps during a trip to Mexico. The diarrhoea resolves after a couple of days without Abx, his symptoms are likely related to?

Campylobacter jejuni
Clostridium difficile
Clostridium perfringens
E.Coli
Salmonella typhi
Staphylococcus aureus
Giardia lamblia
Shigella
Bacillus cereus
H. pylori
A

E.Coli: severity depends on virulence factors the strain of E.Coli possesses.

  • ETEC: Enterotoxigenic Escherichia coli= travellers diarrhoea (watery), no invasion or inflammation (similar to rice water stools of cholera)
  • EEC: EnteroInvasive Escherichia coli= Invades intestinal mucosa and causes necrosis and inflammation. Blood and pus in stool (clinically similar to Shigella)
  • EHEC: Enterohaemorrhagic Escherichia coli, EHEC O157:57 has a Shiga-like toxin that causes Haemolytic uraemia syndrome.
206
Q

A 22 year old student presents to A&E with abdominal cramps, vomiting and diarrhoea. All he has eaten that day is the Chinese takeaway from the night before. Stool microscopy reveals a gram positive bacilli.

Campylobacter jejuni
Clostridium difficile
Clostridium perfringens
E.Coli
Salmonella typhi
Staphylococcus aureus
Giardia lamblia
Shigella
Bacillus cereus
H. pylori
A

Bacillus cereus: gram positive rod, reheated rice (spores germinate) - chinese takeaway!
1-6 hour incubation period, sudden vomiting, minimal non-bloody diarrhoea, self limiting

Staphylococcus aureus: gram positive clusters on staining, 1-6 hour incubation period, enterotoxin causes prominent vomiting and watery, non-bloody diarrhoea, self limiting.

207
Q

An 82 year old lady hospitalised for CAP develops fever and profound diarrhoea at the end of her 3rd stay in hospital. Sigmoidoscopy reveals white/yellow membrane-like plaques that are composed of fibrin and inflammatory cells. The patient most likely has?

Campylobacter jejuni
Clostridium difficile
Clostridium perfringens
E.Coli
Salmonella typhi
Staphylococcus aureus
Giardia lamblia
Shigella
Bacillus cereus
H. pylori
A

Clostridium difficile: number 1 hospital acquired infection, anaerobic gram positive spore forming rod (usually following cephalosporin/fluoroquinolones use), 2 exotoxins: A (diarrhoea), B (cytotoxic to colonic cells), detected in stool - diagnostic
Pseudomembranous colitis
Tx: oral/IV metronidazole/vancomycin

208
Q

A couple present to their GP with a one week history of watery diarrhoea, foul smelling stools and abdominal cramping. They recently returned from a hiking trip in Ukraine. Stool microscopy reveals cysts and trophozoites.

Campylobacter jejuni
Clostridium difficile
Clostridium perfringens
E.Coli
Salmonella typhi
Staphylococcus aureus
Giardia lamblia
Shigella
Bacillus cereus
H. pylori
A

Giardia lamblia: travellers, eastern europe/russia, hikers (clear mountain streams), bloating, abdo pain, malabsorption of protein and fat, foul smelling gas, non-bloody diarrhoea, pear shaped trophozoite/cysts found in stool.
Tx: Metronidazole (also entomoeba, trichomonas)

209
Q

A 55 year old man comes to A&E confused an with a gaze palsy. Thiamine is started immediately and his condition improves significantly days later.

Arnold-Chiari malformation
Berger's disease
Churg-Strauss Syndrome
Crigler-Najjar Syndrome
Fanconi Anaemia
Dressler's Syndrome
Guillain-Barre Syndrome
Leriche's Syndrome
Osler-Weber Rendu Syndrome
Korsakoff's Syndrome
Wernicke's Encephalopathy
Menieres Disease
A

Wernicke’s Encephalopathy: due to chronic alcoholism or severe nutritional deficiencies.
TRIAD: Opthalmoplegia (horizontal nystagmus, bilateral abducens palsy and complete opthalmoplegia); Ataxia; Confusion/disorientation. Tx: thiamine before glucose, as thiamine is a cofactor for the enzymes responsible for glucose metabolism, so infusion of glucose will exacerbate the pre-existing thiamine deficiency precipitating Wernicke’s encephalopathy.
Whereas Korsakoff’s Syndrome: irreversible memory loss, anterograde and retrograde amnesia, confabulation.

210
Q

A 34 year old male asthmatic develops a wrist drop. Blood results reveal eosinophilia and auto antibodies to neutrophil myeloperoxidases. The patient most likely has?

Arnold-Chiari malformation
Berger's disease
Churg-Strauss Syndrome
Crigler-Najjar Syndrome
Fanconi Anaemia
Dressler's Syndrome
Guillain-Barre Syndrome
Leriche's Syndrome
Osler-Weber Rendu Syndrome
Korsakoff's Syndrome
Wernicke's Encephalopathy
Menieres Disease
A

Churg-Strauss Syndrome: small vessel vasculitis - palpable purpura, peripheral neuropathy (wrist, foot drop),
TRIAD: late onset asthma, eosinophilia, small vessel vasculitis
Hx: granulomatous vasculitis, p-ANCA=myeloperoxidase

211
Q

A 42 year old female complains of bouts of a “spinning sensation” associated with nausea and ringing in her left ear. She also notes having to use her right eye when talking on the phone. She says she feels fine between episodes. She is likely to have?

Arnold-Chiari malformation
Berger's disease
Churg-Strauss Syndrome
Crigler-Najjar Syndrome
Fanconi Anaemia
Dressler's Syndrome
Guillain-Barre Syndrome
Leriche's Syndrome
Osler-Weber Rendu Syndrome
Korsakoff's Syndrome
Wernicke's Encephalopathy
Menieres Disease
A

Menieres Disease: inner ear disorder caused by an increased volume of endolymph, causing spontaneous episodic vertigo lasting for minutes to hours, unilateral sensorineural hearing loss, unilateral tinnitus.
Exacerbations may last for months or years during which time episodes can occur as frequently as every few days. Can go into remission spontaneously or with treatment, can recur.

212
Q

A 32 year old male is brought to A&E due to weakness and tingling that started in his feet and is now at his knees. The weakness started a week ago and is getting worse everyday. O/E 2/5 strength in both legs and absent deep tendon reflexes in his knees. PMH unremarkable except for a respiratory tract infection 2 weeks ago. What does he have?

Arnold-Chiari malformation
Berger's disease
Churg-Strauss Syndrome
Crigler-Najjar Syndrome
Fanconi Anaemia
Dressler's Syndrome
Guillain-Barre Syndrome
Leriche's Syndrome
Osler-Weber Rendu Syndrome
Korsakoff's Syndrome
Wernicke's Encephalopathy
Menieres Disease
A

Guillain-Barre Syndrome: usually develops 1-28 days post respiratoy or GI infection (CMV, EBV, campylobacter etc)
Pathology: molecular mimicry to myelin = autoimmune response to peripheral nerves
Symptoms: typically motor more than sensory, ascending symmetrical paralysis - beginning in distal lower extremities, autonomic function may be severely affected (cardiac irregularities, hypotension, hypertension).
Important to measure peak flow regularly to monitor any diaphragmatic spread as affects trunk and respiratory muscles.
Ix: LP - albumino-cytological dissociation
Mx: mechanical ventilation if required, IVIG or plasma exchange, NO corticosteroids, 85% complete recovery, 10% unable to walk, 10% mortality

213
Q

2 weeks after an MI, a man presents to A&E complaining of central chest pain that is worse on lying down and on inspiration. The registrar notes that he has a fever and on auscultation he hears a friction rub. He discharges the patient with aspirin.

Arnold-Chiari malformation
Berger's disease
Churg-Strauss Syndrome
Crigler-Najjar Syndrome
Fanconi Anaemia
Dressler's Syndrome
Guillain-Barre Syndrome
Leriche's Syndrome
Osler-Weber Rendu Syndrome
Korsakoff's Syndrome
Wernicke's Encephalopathy
Menieres Disease
A

Dressler’s Syndrome: pericarditis - sharp pleuritic chest pain, fever, pericardial friction rub and pleural effusions on CXR.
Dressler’s occurs 1 week to a few months post MI - thought to be an autoimmune polyserositis provoked by antigens exposed or created by infarction of the cardiac muscle. Generally responds to aspirin, NSAIDS and/or steroids

214
Q

A second year ICSM athlete visits his GP because of recent weight loss and fatigue. He complains that “my coach keeps telling me to bulk up but no matter what I do I can’t seem to do it - the pounds keep shedding! I’m also drinking water non-stop and its really disturbing training as I keep having to run to the toilet.” The GP orders a….?

Random blood glucose
OGTT
Free T4
Low dose dexamethasone test
High dose dexamethasone test
Short synACTHen test
GH level
TSHR Ig
Water deprivation test
CXR &amp; head MRI
Petrosal sinus sampling
Plasma renin:aldosterone ratio
Pancreatic US
A

Random blood glucose: Diagnosis T1DM - presentation: weight loss, polyuria, polydipsia, pear drops breath, kussmal breathing, vomiting, abdo pain.
DKA: increase in glucagon, corticosteroids, adrenaline, GH, gluconeogenesis, glycogenolysis and ketogenesis.

215
Q

A 65 year old make presents with difficulty climbing the stairs and exertional dyspnoea. O/E he has centripetal obesity, bruises, darkening palmar creases and recent onset DM. Which Ix could be used to prove an extra-cranial pathology is responsible for his disease?

Random blood glucose
OGTT
Free T4
Low dose dexamethasone test
High dose dexamethasone test
Short synACTHen test
GH level
TSHR Ig
Water deprivation test
CXR &amp; head MRI
Petrosal sinus sampling
Plasma renin:aldosterone ratio
Pancreatic USL
A

High dose dexamethasone: Diagnosis Cushing’s syndrome due to small cell carcinoma - personality change, hyperglycaemia, moon face, increased infections, gynaecomastia, lollypop on sticks, dyspepsia, amenorrhoea, thin skin, purple striae, bruises, petechiae, osteoporosis, buffalo hump, hirsuitism.
Dexamethasone is a synthetic cortisol analogue, so should suppress cortisol production.
LOW dose - used to diagnose cushing’s syndrome, normal people suppress to 0.
HIGH dose - distinguishes between Cushing’s disease and other causes. Pituitary cushing’s suppresses to 50% (pituitary adenoma secreting ACTH is commonest cause after iatrogenic). Ectopic ACTH (small cell lung ca) and primary adrenal tumours will not suppress at all.

216
Q

A 25 year old man sees his GP with a 3 month history of polyuria & polydipsia. PMH includes a long stay in the ITU at Hammersmith Hospital following a car crash. His neurological examination is unremarkable and bloods reveal a glucose of 5.8 mmol/L

Random blood glucose
OGTT
Free T4
Low dose dexamethasone test
High dose dexamethasone test
Short synACTHen test
GH level
TSHR Ig
Water deprivation test
CXR &amp; head MRI
Petrosal sinus sampling
Plasma renin:aldosterone ratio
Pancreatic US
A

Water deprivation test: diagnosis Diabetes Insipidus vs psychogenic polydipsia
DI: get up at night to drink water
- Central: pituitary tumour, trauma, surgery. Central can be corrected with DDAVP (desmopressin - synthetic analogue of ADH) Tx: Intranasal desmopressin
- Nephrogenic: hereditary or secondary - hypercalcaemia, lithium and demeclocyline. Tx: thiazide diuretics (hydrochlorothiazide, indomethacin or amiloride).
In primary polydipsia urine osmolality >800, whereas in DI 800, nephrogenic still

217
Q

A 65 year old male smoker was found to have hyponatraemia on a routine check up blood test. He is not taking any medication and investigations reveal no evidence of adrenal insufficiency. His urine osmolality is high.

Random blood glucose
OGTT
Free T4
Low dose dexamethasone test
High dose dexamethasone test
Short synACTHen test
GH level
TSHR Ig
Water deprivation test
CXR &amp; head MRI
Petrosal sinus sampling
Plasma renin:aldosterone ratio
Pancreatic US
A

CXR and head MRI: Dx: SIADH (excess ADH production)
Suspect if patient has: hyponatraemia (dilutional), plasma hypo-osmolality (100mOsm/kg).
Symptoms:
Mild - nausea, vomiting, headache, anorexia, lethargy.
Moderate - muscle cramps, weakness, confusion, ataxia
Severe - drowsiness, seizures, coma
Tx: with care (central pontine demyelinolysis) captains, fluid restrict, demeclocycline
Other causes increased ADH (vasopressin): hypothyroidism, infective (pneumonia, lung abscess, brain abscess), brain (meningitis, SAH), Drugs (carbamazepine, SSRIs, morphine)

218
Q

During a GP consultation you notice a 65 year old man has coarse facial features and widely spaced teeth. He complains of headaches and his rings getting stuck on his fingers.

Random blood glucose
OGTT
Free T4
Low dose dexamethasone test
High dose dexamethasone test
Short synACTHen test
GH level
TSHR Ig
Water deprivation test
CXR &amp; head MRI
Petrosal sinus sampling
Plasma renin:aldosterone ratio
Pancreatic US
A

OGTT: Dx - acromegaly caused by excess GH in adults - pituitary tumour in 99% of cases.
Clinical Manifestations: Enlarged pituitary gland, headache, visual disturbance, coarse facial features, protruding jaw, HTN, CHF, sleep apnoea, hypertrophy soft tissue, enlargement small bones (hands and feet), menstrual changes.
Complications: raised ICP and impaired glucose tolerance
Serum IGF-1 is the screening test for acromegaly (GH secretion over past 24 hours)
OGTT = gold standard. GH is normally inhibited by increased glucose and therefore should be undetectable, but in acromegaly the production of GH is not affected by glucose therefore will be raised. (+MRI pituitary)
Tx: Trans-sphenoidal surgery, if still remains high - Somatostatin analogue (octreotide and lanreotide), GH receptor antagonists (pegvisomant), radiotherapy

219
Q

A young woman presents to you with low mood, weakness and is rather tearful. She also has a mild GI irritation. On examination she has darkened skin creases.

Random blood glucose
OGTT
Free T4
Low dose dexamethasone test
High dose dexamethasone test
Short synACTHen test
GH level
TSHR Ig
Water deprivation test
CXR &amp; head MRI
Petrosal sinus sampling
Plasma renin:aldosterone ratio
Pancreatic US
A

short synACTHen test: Dx - Addison’s disease
- Involves IM injection of synthetic ACTH
- Cortisol is measured at 30 and 60 minutes then at 2, 4, 8, 12, 24 hours and serum cortisol should rise substantially during this period of time - WILL NOT RISE in Addison’s disease.
Presentation: bronze pigmentation of skin, changes in body hair distribution, GI disturbance, weakness, hypoglycaemia, postural hypotension, weight loss
Adrenal Crisis: Profound fatigue, dehydration, vascular collapse, renal shut down, low serum sodium, elevated serum potassium.

220
Q

A 30 year old lady sees her GP as she has generalised weakness and paraesthesias. She has severe hypertension and her blood tests show raised sodium, low potassium and raised bicarbonate.

Random blood glucose
OGTT
Free T4
Low dose dexamethasone test
High dose dexamethasone test
Short synACTHen test
GH level
TSHR Ig
Water deprivation test
CXR &amp; head MRI
Petrosal sinus sampling
Plasma renin:aldosterone ratio
Pancreatic US
A

Plasma renin:aldosterone ratio - Dx: Conn’s syndrome - caused by adrenal hyperplasia or an aldosterone secreting adrenal adenoma.
Causes: HTN, hypokalaemia and metabolic alkalosis.
Can be unilateral of bilateral
Ix: plasma renin is typically low, with an inappropriately high aldosterone (increased plasma renin:aldosterone ratio)
Tx: surgery and/or spironolactone or eplerenone

221
Q

A 35 year old lady presents to her endocrinologist complaining of visual disturbances. She says that when she crosses a street she can’t see the cars on either side. PMH: hypercalcaemia treated 3 years ago with neck surgery. On top of ordering tests to study the pituitary, the consultant orders a ….?

Random blood glucose
OGTT
Free T4
Low dose dexamethasone test
High dose dexamethasone test
Short synACTHen test
GH level
TSHR Ig
Water deprivation test
CXR &amp; head MRI
Petrosal sinus sampling
Plasma renin:aldosterone ratio
Pancreatic US
A

Pancreatic US: Dx: MEN1 - think the 3 P’s!
Parathyroid, Pancreas, Pituitary!
MEN = a group of AD familial disorders characterised by specific clustering of endocrine tumours.

222
Q

A 35 year old woman presents to the GP with insomnia and heart palpitations. The lady’s irritability begins to annoy the GP until she tells him that she’s lost 5kgs over the past 2 months despite eating a whole KFC bucket each night. At this point the GP also notices that the patient is wearing shorts even though she has an unsightly rash on her shins.

Anaplastic Carcinoma
De Quervain's thyroiditis
Follicular carcinoma
Grave's Disease
Hashimoto's thyroiditis
Medullary carcinoma
Papillary carcinoma
Riedel's thyroiditis
Toxic multinodular goitre
Cretinism
A

Grave’s Disease: AI disease - hyperthyroidism, exopthalmus, pretibial myxoedema, TSH-R Ig
- Heat intolerance, bulging eyes, finger clubbing, tremors, diarrhoea, menstrual changes, localised oedema, muscle wasting, weight loss, breast enlargement, tachycardia, increased BP

223
Q

A 53 year old lady complains to her GP of weight gain, lethargy and constipation. These symptoms started roughly 5 months back but have progressed over time. O/E there is a mild, diffusely enlarged thyroid gland.

Anaplastic Carcinoma
De Quervain's thyroiditis
Follicular carcinoma
Grave's Disease
Hashimoto's thyroiditis
Medullary carcinoma
Papillary carcinoma
Riedel's thyroiditis
Toxic multinodular goitre
Cretinism
A

Hashimoto’s Disease: most common cause of hypothyroidism (AI)
O/E: moderately enlarged non-tender thyroid
Autoantibodies: anti-thyroglobulin, anti-microsomal, anti-TSH-R, anti-thyroid peroxidase (TPO)
Intolerance to cold, receding hairline, hair loss, apathy, lethargy, dry skin, muscle weakness, constipation, anorexia, brittle nails, menstrual disturbance

224
Q

A 40 year old female presents to her GP complaining of mood swings, problems with attention and a recent onset hand tremor. She also complains of discomfort in her neck, which radiates to her ear when swallowing. She says she initially thought little of the discomfort as she assumed it was due to the recent flu-like illness she had?

Anaplastic Carcinoma
De Quervain's thyroiditis
Follicular carcinoma
Grave's Disease
Hashimoto's thyroiditis
Medullary carcinoma
Papillary carcinoma
Riedel's thyroiditis
Toxic multinodular goitre
Cretinism
A

De Quervain’s thyroiditis: viral infection (influenza, coxsacchie, mumps) inflames and releases the thyroid hormone.
- Pain in neck that is worse with swallowing/moving
- Fever
- Triphasic (hyper-hypo-euthyroid)
Dx: clinical and raised ESR
Tx: mostly self limiting, symptomatic with NSAIDS

225
Q

A 61 year old woman presents to her GP with difficulty breathing and swallowing and a midline neck mass that has been increasing in size over the past few months. O/E the thyroid gland is enlarged, hard and immobile, but there is no tenderness on palpation. Ultrasound guided thyroid biopsy reveals an increased amount of fibrotic tissue.

Anaplastic Carcinoma
De Quervain's thyroiditis
Follicular carcinoma
Grave's Disease
Hashimoto's thyroiditis
Medullary carcinoma
Papillary carcinoma
Riedel's thyroiditis
Toxic multinodular goitre
Cretinism
A

Riedel’s Thyroiditis: idiopathic fibrosis of the thyroid - slow growing goitre (firm, rock like, irregular)
Dx: biopsy (to distinguish from cancer), initially TFTs appear normal but may develop hypothyroid and hypoparathyroidism.
Tx: None or palliative surgery for symptoms (stridor, dysphagia)

226
Q

An 84 year old has presented with a lump in her neck that has been growing rapidly for the past month. She has problems swallowing and examination reveals a hard, irregular mass on her right side which is fixed to the overlying skin.

Anaplastic Carcinoma
De Quervain's thyroiditis
Follicular carcinoma
Grave's Disease
Hashimoto's thyroiditis
Medullary carcinoma
Papillary carcinoma
Riedel's thyroiditis
Toxic multinodular goitre
Cretinism
A

Anaplastic Carcinoma: Rare, Bad news, Elderly patients, Very aggressive & poor response to tx (>90% die within one year)
Hard, symmetrical, rapidly enlarging goitre
Can spread to: LNs, Trachea (stridor), Recurrent laryngeal nerve (hoarseness)

Medullary Carcinoma: Rare (5%)
Sporadic or part of MEN2 syndrome, may produce calcitonin (tumour marker)

227
Q

A 25 year old lady is referred to you in the endocrinology department with a left sided neck lump. This is her only complaint. O/E: smooth, regular, firm lump that moves upwards with swallowing but not with tongue protrusion. Cervical lymph nodes are present on the same side of her neck.

Anaplastic Carcinoma
De Quervain's thyroiditis
Follicular carcinoma
Grave's Disease
Hashimoto's thyroiditis
Medullary carcinoma
Papillary carcinoma
Riedel's thyroiditis
Toxic multinodular goitre
Cretinism
A

Papillary carcinoma
Majority (90%) of thyroid tumours Adolescents & young adults
May spread to lymph nodes Histology:
- “Orphan Annie eye” nuclei - These are so named because they have a “cleared-out” appearance, similar to Little Orphan Annie’s eyes
- Psammoma bodies (sandy)
Tx: Thyroidectomy +/- radio-iodine
May be TSH dependent so thyroxine for rest of life to reduce reoccurrence

Follicular Carcinoma: 20% thyroid tumours, Middle aged, Spreads early to lung & bone
Tx: thyroidectomy & radioiodine
May be TSH dependent so thyroxine for rest of life to reduce reoccurrence

228
Q

A 57 year old man is found collapsed at work. On arrival to A&E he is complaining of mild epigastric pain that started at rest. He is pale, sweaty and has vomitted once. He is on insulin for Type 1 diabetes. He has already received aspirin, morphine, oxygen, nitrates and anti-emetics.

A.  Primary percutaneous coronary intervention
B.  High dose oral steroids
C.  Coronary artery bypass graft
D.  IV phenytoin
E.  Oral leukotriene receptor antagonist
F.  IV magnesium sulphate
G.  1mg IM glucagon
H. 50mL of 50% IV glucose
I. Continue oxygen via facemask and nebulised short acting β-agonists and steroids
J. Oral glucose
K. IV lorazepam
L. Oral diazepam 
M.  IV benzylpenicillin 
N.  Streptokinase
A

Primary percutanous coronary intervention: following MI primary PCI is gold standard and superior to thrombolysis

229
Q

A 21 year old woman with known asthma is brought to A&E with SOB. She can barely talk, has a silent chest and a resp. rate of 36/min. With oxygen,nebulised salbutamol and steroid there is some improvement. Her peak flow is 30% of best.

A.  Primary percutaneous coronary intervention
B.  High dose oral steroids
C.  Coronary artery bypass graft
D.  IV phenytoin
E.  Oral leukotriene receptor antagonist
F.  IV magnesium sulphate
G.  1mg IM glucagon
H. 50mL of 50% IV glucose
I. Continue oxygen via facemask and nebulised short acting β-agonists and steroids
J. Oral glucose
K. IV lorazepam
L. Oral diazepam 
M.  IV benzylpenicillin 
N.  Streptokinase
A

IV Magnesium sulphate: silent chest and peak flow

230
Q

A 22 year old woman who has newly diagnosed T1DM begins to fit in the outpatients waiting room. She is pale and clammy. After securing her airway the nurse finds that the capillary blood glucose is 1.0mmol/L. IV access has not yet been gained as she is very difficult to cannulate.

A.  Primary percutaneous coronary intervention
B.  High dose oral steroids
C.  Coronary artery bypass graft
D.  IV phenytoin
E.  Oral leukotriene receptor antagonist
F.  IV magnesium sulphate
G.  1mg IM glucagon
H. 50mL of 50% IV glucose
I. Continue oxygen via facemask and nebulised short acting β-agonists and steroids
J. Oral glucose
K. IV lorazepam
L. Oral diazepam 
M.  IV benzylpenicillin 
N.  Streptokinase
A

1mg IM glucagon: profound hypoglycaemia, no IV access, so glucagon mobilises glucose stores

231
Q

A 36 year old woman is found on the street having a seizure. After the fitting has finished passers-by try to rouse her. Before she regains consciousness she starts having another seizure. When the ambulance arrives 10mins later she is still fitting.

A.  Primary percutaneous coronary intervention
B.  High dose oral steroids
C.  Coronary artery bypass graft
D.  IV phenytoin
E.  Oral leukotriene receptor antagonist
F.  IV magnesium sulphate
G.  1mg IM glucagon
H. 50mL of 50% IV glucose
I. Continue oxygen via facemask and nebulised short acting β-agonists and steroids
J. Oral glucose
K. IV lorazepam
L. Oral diazepam 
M.  IV benzylpenicillin 
N.  Streptokinase
A

IV lorazepam: status epilepticus 1st line treatment. Requires resus equipment. If this fails try rectal diazepam/buccal midazolam then IV phenytoin then GA

232
Q

A 67 year old woman presents to A+E with lethargy, fever and left sided temporal headaches. There is no obvious pulsation of the temporal artery.

A.  Primary percutaneous coronary intervention
B.  High dose oral steroids
C.  Coronary artery bypass graft
D.  IV phenytoin
E.  Oral leukotriene receptor antagonist
F.  IV magnesium sulphate
G.  1mg IM glucagon
H. 50mL of 50% IV glucose
I. Continue oxygen via facemask and nebulised short acting β-agonists and steroids
J. Oral glucose
K. IV lorazepam
L. Oral diazepam 
M.  IV benzylpenicillin 
N.  Streptokinase
A

High dose oral steroids - treatment for temporal arteritis

233
Q

A 32 year old man presents with SOB. He feels that it is similar to the asthma he had as a child. He has also developed hay fever. On examination he has a palpabale purple rash on his legs.

A.  Behҫet’s disease
B.  Chrug-Strauss syndrome
C.  Giant cell arteritis
D.  Kawasaki’s disease
E.  Microscopic polyangitis
F. Polyarteritis nodosa
G. Polymyalgia rheumatica
H. Takayasu’s arteritis
I. Wegener’s disease
A

Churg Strauss: can be triphasic - rhinitis and allergies, eosinophilia, systemic vasculitis, pANCA.
Tx: steroids

234
Q

A 29 year old man complains of recurrent painful ulceration in his mouth and on his genitals. The ulceration is clearly seen on examination. Swab cultures are negative for herpes simplex virus.

A.  Behҫet’s disease
B.  Chrug-Strauss syndrome
C.  Giant cell arteritis
D.  Kawasaki’s disease
E.  Microscopic polyangitis
F. Polyarteritis nodosa
G. Polymyalgia rheumatica
H. Takayasu’s arteritis
I. Wegener’s disease
A

Behҫet’s disease: recurrent oral and genital ulceration. Also iritis, skin lesions, thrombophlebitis. Pathergy reaction. Commonly Turkey and eastern Mediterranean.
HLA-B5, M

235
Q

A 65 year old woman presents with a pain in her shoulder. The pain came on suddenly this morning and now she is barely able to move her arms. She is otherwise well. On examination, shoulder movement is limited by stiffness bilaterally.

A.  Behҫet’s disease
B.  Chrug-Strauss syndrome
C.  Giant cell arteritis
D.  Kawasaki’s disease
E.  Microscopic polyangitis
F. Polyarteritis nodosa
G. Polymyalgia rheumatica
H. Takayasu’s arteritis
I. Wegener’s disease
A

Polymyalgia rheumatica: isn’t actually a vasculitis, presents with abrupt onset muscle pain and stiffness without weakness.
Tx: corticosteroids

236
Q

A 42 year old man presents with a 2 month history of recurrent nosebleed. More recently he has started coughing up blood as well. On examination there is a deformity in the bridge of his nose.

A.  Behҫet’s disease
B.  Chrug-Strauss syndrome
C.  Giant cell arteritis
D.  Kawasaki’s disease
E.  Microscopic polyangitis
F. Polyarteritis nodosa
G. Polymyalgia rheumatica
H. Takayasu’s arteritis
I. Wegener’s disease
A

Wegener’s disease: granulomatous necrotizing vasculitis. Triad of upper airway pathology, respiratory disease and renal disease. cANCA +ve in 90% cases
Tx: steroids and immunosuppression

237
Q

A 5 year old boy is brought to the GP as he has had a fever for the past 5 days. On examination you find cervical lymphadenopathy, odema and desquamation of his legs, oral mucosal erythema and a rash on his back.

A.  Behҫet’s disease
B.  Chrug-Strauss syndrome
C.  Giant cell arteritis
D.  Kawasaki’s disease
E.  Microscopic polyangitis
F. Polyarteritis nodosa
G. Polymyalgia rheumatica
H. Takayasu’s arteritis
I. Wegener’s disease
A

Kawasaki’s disease: acute febrile systemic vasculitis, children (especially Japanese boys). Must have fever for 5 days; and 4 of: cervical lymphadenopathy, oral mucosal erythema, conjunctivitis without exudates, rash, extremity changes. May also have thrombocytosis
Tx: anti-inflammatories and immunoglobulins
Associated with coronary aneurysms.

238
Q

A 34 year old woman has been suffering diarrhoea and vomitting for the past 4 days, She has a prolonged P-R interval, depressed ST segment, flattened T waves and prominent U waves

A.  Mitral stenosis
B.  Aortic regurgitation
C.  Aortic stenosis
D.  Tricuspid stenosis
E.  Tricuspid regurgitation
F.  Hypokalaemia
G. Hyperkalaemia
H. Normal ECG
I. Pericarditis
J.  Cardiac Tamponade
K.  Pulmonary embolus
L.  Subendocardial infarction
A

Hypokalaemia: GI potassium loss, could have presented with muscle weakness and cramps.
Other causes of loss: diuretics, Conn’s, burns, sweating.
Causes of re-distribution: beta-2 agonist use, excessive insulin, alkalosis
Can cause atrial and ventricular arrhythmias

239
Q

A 34 year old man presents to A+E after a road traffic accident. The ECG shows pulseless electrical activity.

A.  Mitral stenosis
B.  Aortic regurgitation
C.  Aortic stenosis
D.  Tricuspid stenosis
E.  Tricuspid regurgitation
F.  Hypokalaemia
G. Hyperkalaemia
H. Normal ECG
I. Pericarditis
J.  Cardiac Tamponade
K.  Pulmonary embolus
L.  Subendocardial infarction
A

Cardiac Tamponade: PEA can be called electromechanical dissociation - think of 6H’s and 6Ts of reversible causes.

240
Q

ECG of an 8 year old girl shows T wave inversion in V1-3

A.  Mitral stenosis
B.  Aortic regurgitation
C.  Aortic stenosis
D.  Tricuspid stenosis
E.  Tricuspid regurgitation
F.  Hypokalaemia
G. Hyperkalaemia
H. Normal ECG
I. Pericarditis
J.  Cardiac Tamponade
K.  Pulmonary embolus
L.  Subendocardial infarction
A

Normal ECG: t wave inversion is a normal finding in children

241
Q

An ECG of a 55 year old man in end stage renal failure presents to A+E experiencing palpitations. He missed his last haemodialysis appointment because he was on holiday. His ECG shows tall tented T waves

A.  Mitral stenosis
B.  Aortic regurgitation
C.  Aortic stenosis
D.  Tricuspid stenosis
E.  Tricuspid regurgitation
F.  Hypokalaemia
G. Hyperkalaemia
H. Normal ECG
I. Pericarditis
J.  Cardiac Tamponade
K.  Pulmonary embolus
L.  Subendocardial infarction
A

Hyperkalaemia: haemodialysis removes potassium, tall tented t waves is the give away, also look for potassium sparing diuretics, ACEi and metabolic acidosis.

242
Q

A 65 year old woman with a PMH of rheumatic heart disease presents to the GP with palpitations. Her ECG shows sinus rhythm and bifid p waves best seen in II, V3 and V4.

A.  Mitral stenosis
B.  Aortic regurgitation
C.  Aortic stenosis
D.  Tricuspid stenosis
E.  Tricuspid regurgitation
F.  Hypokalaemia
G. Hyperkalaemia
H. Normal ECG
I. Pericarditis
J.  Cardiac Tamponade
K.  Pulmonary embolus
L.  Subendocardial infarction
A

Mitral stenosis: broad bifid p waves (and sinus rhythm known as p mitrale and suggestions left atrial hypertrophy - can also be seen in MR

243
Q

A 60 year old woman complaining of bilateral proximal muscle weakness in the legs and dysphagia. On examination she has a purple rash on her cheeks

A.  Myasthenia Gravis
B.  Charcot-Marie-Tooth disease
C.  Polymyalgia rheumatica
D.  Motor neuron disease
E.  Guillain-Barré syndrome
F.  Syringomyelia
G. Alcohol-induced neuropathy 
H. Parkinson’s disease
I. Polymyositis
J.  Multiple sclerosis
K.  Bilateral stroke
L.  Neurofibromatosis
A

Polymyositis: AI condition, non-suppurative inflammation of skeletal muscle and weakness of pelvic girdle and shoulders, oesophageal dysmotility, dermatomyositis, interstitial lung disease.

244
Q

A 40 year old man presents with a recent history of progressive weakness in the arms and legs. Examination reveals flaccid weakness of limbs with no reflexes.

A.  Myasthenia Gravis
B.  Charcot-Marie-Tooth disease
C.  Polymyalgia rheumatica
D.  Motor neuron disease
E.  Guillain-Barré syndrome
F.  Syringomyelia
G. Alcohol-induced neuropathy 
H. Parkinson’s disease
I. Polymyositis
J.  Multiple sclerosis
K.  Bilateral stroke
L.  Neurofibromatosis
A

Guillain-Barre: ascending polyneuropathy, may be associated with preceding infection. Symmetrical, affecting proximal before distal. CN and ANS can be involved. Be aware of respiratory muscle involvement
Tx: ventilation and immunoglobulins

245
Q

A 55 year old man presents with bilateral progressive worsening muscle weakness. There is marked wasting of the lower limb muscles and very brisk lower limb reflexes. Sensation is normal.

A.  Myasthenia Gravis
B.  Charcot-Marie-Tooth disease
C.  Polymyalgia rheumatica
D.  Motor neuron disease
E.  Guillain-Barré syndrome
F.  Syringomyelia
G. Alcohol-induced neuropathy 
H. Parkinson’s disease
I. Polymyositis
J.  Multiple sclerosis
K.  Bilateral stroke
L.  Neurofibromatosis
A

MND: progressive degenerative disease affecting upper and lower motor neurons, with no sensory involvement.
3 patterns: Progressive bulbar palsy, amyotrophic lateral sclerosis, progressive muscular atrophy.

246
Q

A 13 year old boy presents with bilateral pes cavus with clawing of the toes. There is atrophy of the peroneal muscles and reduced reflexes and sensation distally.

A.  Myasthenia Gravis
B.  Charcot-Marie-Tooth disease
C.  Polymyalgia rheumatica
D.  Motor neuron disease
E.  Guillain-Barré syndrome
F.  Syringomyelia
G. Alcohol-induced neuropathy 
H. Parkinson’s disease
I. Polymyositis
J.  Multiple sclerosis
K.  Bilateral stroke
L.  Neurofibromatosis
A

Charcot-Marie-Tooth disease: progressive peroneal atrophy, AD or AR, hereditary sensorimotor neuropathy.
Type I = dyemyelinating, insidious, 1st decade, foot deformity (pea cavus)
Type II = 2nd decade, lower limb weakness and sensory loss

247
Q

A 30 year old secretary presents with bilateral leg weakness and blurred vision. Fundoscopy reveal pale optic disks bilaterally.

A.  Myasthenia Gravis
B.  Charcot-Marie-Tooth disease
C.  Polymyalgia rheumatica
D.  Motor neuron disease
E.  Guillain-Barré syndrome
F.  Syringomyelia
G. Alcohol-induced neuropathy 
H. Parkinson’s disease
I. Polymyositis
J.  Multiple sclerosis
K.  Bilateral stroke
L.  Neurofibromatosis
A

Multiple Sclerosis: bilateral weakness a less common presentation, previous episodes of subclinical optic neuritis. MRI spine, CSF.
Tx: Interferon-beta and supportive

248
Q

A middle aged asymptomatic male with bilirubin 50μmol/l, picked up on routine blood test, normal AST, ALT and ALP. No bilirubin detected in urine

a. Gilbert’s Disease
b. Hepatitis A
c. Alcoholic liver disease
d. Hepatitis C
e. Biliary tract obstruction
f. Haemolytic anaemia
g. Hepatocellular carcinoma
h. Crigler Najjar Syndrome
i. Short term alcohol abuse
j. Hepatitis B
k. . Paget’s Disease
l. Carcinoma of the pancreas

A

Gilbert’s Disease: isolated hyperbilirubinaemia with no urinary bilirubin suggest unconjugated hyperbilirubinaemia.
Asymptomatic, normal LFTs, intermittent jaundice with infections, stress, fasting.

249
Q

A 35 year old female with right upper quadrant pain and jaundice. ALT 25, AST 30, ALP 450, GGT 85, bilirubin 60μmol/l

a. Gilbert’s Disease
b. Hepatitis A
c. Alcoholic liver disease
d. Hepatitis C
e. Biliary tract obstruction
f. Haemolytic anaemia
g. Hepatocellular carcinoma
h. Crigler Najjar Syndrome
i. Short term alcohol abuse
j. Hepatitis B
k. . Paget’s Disease
l. Carcinoma of the pancreas

A

Biliary tract obstruction: high ALP and GGT out of proportion with other liver enzymes is suggestive of a gall stone e.g. in CBD

250
Q

A 30 year old male presenting with lethargy. Bilirubin 50μmol/l (unconjugated) ALT 20, AST 80, ALP 40, LDH elevated

a. Gilbert’s Disease
b. Hepatitis A
c. Alcoholic liver disease
d. Hepatitis C
e. Biliary tract obstruction
f. Haemolytic anaemia
g. Hepatocellular carcinoma
h. Crigler Najjar Syndrome
i. Short term alcohol abuse
j. Hepatitis B
k. . Paget’s Disease
l. Carcinoma of the pancreas

A

Haemolytic anaemia: unconjugated hyperbilirubinaemia, elevated AST, but normal ALT and ALP and elevated LDH.
AST is not purely hepatic, it is also found in the heart, erythrocytes etc. LDH is also found in the heart, muscle, erythrocytes etc. Hence a patient with haemolytic anaemia would see rises in these enzymes. The clinical picture of lethargy is also consistent.

251
Q

A 56 year old male admitted to emergency department with AST 35, ALT 30, ALP 55, GGT 120, bilirubin 10μmol/l

a. Gilbert’s Disease
b. Hepatitis A
c. Alcoholic liver disease
d. Hepatitis C
e. Biliary tract obstruction
f. Haemolytic anaemia
g. Hepatocellular carcinoma
h. Crigler Najjar Syndrome
i. Short term alcohol abuse
j. Hepatitis B
k. . Paget’s Disease
l. Carcinoma of the pancreas

A

Short term alcohol abuse: there is an isolated rise in GGT, with other liver enzymes within normal range. GGT can rise even in the absence of major liver disease, but is typically seen in alcohol abuse or due to enzyme inducing drugs.

252
Q

A 24 year old male with fever, jaundice and RUQ pain. ALT 1500, AST 800,
ALP 80, bilirubin 50μmol/l. RNA virus found and raised IgM titre

a. Gilbert’s Disease
b. Hepatitis A
c. Alcoholic liver disease
d. Hepatitis C
e. Biliary tract obstruction
f. Haemolytic anaemia
g. Hepatocellular carcinoma
h. Crigler Najjar Syndrome
i. Short term alcohol abuse
j. Hepatitis B
k. . Paget’s Disease
l. Carcinoma of the pancreas

A

Hepatitis A: very high transaminases should suggest acute liver insult, coupled with the suggestive history of fever, jaundice and RUQ, the most likely dx to fit the picture is HepA.
HepB can also present with an acute hepatitis picture, but is a DNA virus.
HepC has a mostly asymptomatic acute infection phase, only a small proportion will get jaundiced and deranged liver enzymes. Around 80% of those who contract HCV go on to get chronic disease. It is also a RNA virus.

253
Q

A 25 year old female admitted to hospital with nausea/vomiting and severe hypotension, most marked when she stands up. Blood results show glucose 3mmol/l, Na 119mmol/l, K 7.2mmol/l

A: Carbimazole
B: Surgery
C: Octreotide
D: IV hydrocortisone 
E: Levothyroxine
F: IV Prednisolone
G: Gliclazide
H: Desmopressin 
I: Propylthiouracil 
J: Liothyronine 
K: Metyrapone
L: Bendroflumethiazide
A

IV hydrocortisone: this patient is suffering from an Addisonian crisis!
Postural hypotension, hypoglycaemia (due to low cortisol) and hyponatraemia and hypokalaemia (low aldosterone) are typical findings. Non specific features: nausea and vomiting, abdo pain
Basic resuscitation and IV hydrocortisone

254
Q

A 40 year old gentleman who has been suffering from persistent headaches. His wife mentions that his face appears to have changed in the last couple of years. He cannot undergo surgery

A: Carbimazole
B: Surgery
C: Octreotide
D: IV hydrocortisone 
E: Levothyroxine
F: IV Prednisolone
G: Gliclazide
H: Desmopressin 
I: Propylthiouracil 
J: Liothyronine 
K: Metyrapone
L: Bendroflumethiazide
A

Octreotide: Headaches coupled with an insidious change in appearance is suggestive of acromegaly. 99% of cases are due to a pituitary tumour. Surgery is definitive treatment, medical management include somatostatin analogues e.g. octreotide

255
Q

A 31 year old lady with weight loss, large appetite and a fine tremor. She tells you she is pregnant.

A: Carbimazole
B: Surgery
C: Octreotide
D: IV hydrocortisone 
E: Levothyroxine
F: IV Prednisolone
G: Gliclazide
H: Desmopressin 
I: Propylthiouracil 
J: Liothyronine 
K: Metyrapone
L: Bendroflumethiazide
A

She has classic symptoms of hyperthyroidism, other symptoms you might encounter are: intolerance of heat, emotional lability, diarrhoea, oligomenorrhoea. Normal anti-thyroid drugs used are Carbimazole and propylthiouracil, however carbimazole can cross the placenta and be passed through breast milk (tetratogenic) therefore propylthiouracil is preferred in pregnancy.

256
Q

A 42 year old female with centripetal obesity, abdominal striae and round face. Low dose dexamethasone suppression test fails to suppress cortisol, but high dose dexamethasone suppression test significantly reduces cortisol levels.

A: Carbimazole
B: Surgery
C: Octreotide
D: IV hydrocortisone 
E: Levothyroxine
F: IV Prednisolone
G: Gliclazide
H: Desmopressin 
I: Propylthiouracil 
J: Liothyronine 
K: Metyrapone
L: Bendroflumethiazide
A

Surgery: this lady has Cushing’s syndrome, and more specifically cushing’s disease as proven by the high dose dexamethasone test. The most appropriate treatment is trans sphenoidal resection.

257
Q

A 32 year old gentleman with hypertension that has been resistant to treatment. Plasma Na 150, K 2.8

A: Carbimazole
B: Surgery
C: Octreotide
D: IV hydrocortisone 
E: Levothyroxine
F: IV Prednisolone
G: Gliclazide
H: Desmopressin 
I: Propylthiouracil 
J: Liothyronine 
K: Metyrapone
L: Bendroflumethiazide
A

Surgery: the dx is Conn’s syndrome (primary hyperaldosteronism) - young person, resistant HTN, hypernatraemia, hypokalaemia. >80% cases are due to adrenal adenomas and surgery is the treatment of choice - medical treatment can be used prior to surgery e.g. spironolactone

258
Q

A 25 year old female complains of difficulty swallowing. Further questioning reveals that she feels a constant unpleasant sensation of a lump in the throat; however, investigations are unremarkable

a. Achalasia
b. Plummer-Vinson Syndrome
c. Oesophageal cancer
d. Peptic stricture
e. Pharyngeal pouch
f. Reflux oesophagitis
g. Oesophageal candidiasis
h. Myasthenia gravis
i. Bulbar palsy
j. Hiatus hernia
k. Globus hystericus
l. Pseudobulbar palsy

A

Globus hystericus: psychosomatic disorder with the feeling of a lump in the throat independent of eating/drinking, benign. Lack of any alarm symptoms, not really a difficulty in swallowing, more a subjective sensation of a lump. No Hx reflux

259
Q

A 43 year old gentleman who has recently undergone a kidney transplant presents with 1 week history of severe odynophagia and difficulty swallowing. He is currently taking prednisolone and cyclosporin

a. Achalasia
b. Plummer-Vinson Syndrome
c. Oesophageal cancer
d. Peptic stricture
e. Pharyngeal pouch
f. Reflux oesophagitis
g. Oesophageal candidiasis
h. Myasthenia gravis
i. Bulbar palsy
j. Hiatus hernia
k. Globus hystericus
l. Pseudobulbar palsy

A

Oesophageal candidiasis: this patient is immunocompromised following his kidney transplant. This combined with a short Hx makes oesophageal candidiasis a likely Dx, supported by painful swallowing. Also seen in HIV

260
Q

A 34 year old female presents with intermittent dysphagia of both solids and liquids along with chest pain and regurgitation. Manometry shows failure of lower oesophageal sphincter to relax

a. Achalasia
b. Plummer-Vinson Syndrome
c. Oesophageal cancer
d. Peptic stricture
e. Pharyngeal pouch
f. Reflux oesophagitis
g. Oesophageal candidiasis
h. Myasthenia gravis
i. Bulbar palsy
j. Hiatus hernia
k. Globus hystericus
l. Pseudobulbar palsy

A

Achalasia: Dysphagia is affecting both solid and liquid equally, and is not progressive but intermittent

  • Together with manometry findings, confirms diagnosis as achalasia
  • An oesophageal motility disorder due to degeneration of myenteric plexus cells -Causes failure of lower oesophageal sphincter to relax
  • Look for the characteristic “birds beak” appearance on a barium swallow
261
Q

A 55 year old gentleman presents with difficulty swallowing associated with drooling, a spastic tongue and exaggerated jaw jerk

a. Achalasia
b. Plummer-Vinson Syndrome
c. Oesophageal cancer
d. Peptic stricture
e. Pharyngeal pouch
f. Reflux oesophagitis
g. Oesophageal candidiasis
h. Myasthenia gravis
i. Bulbar palsy
j. Hiatus hernia
k. Globus hystericus
l. Pseudobulbar palsy

A

Pseudo-bulbar palsy:
-Spasticity and hyperreflexia all suggest an upper motor neurone lesion, in this case
affecting the corticobulbar tract
-This is known as pseudobulbar palsy
-Other features e.g. “Donald Duck” speech
-Cf. bulbar palsy, where there are features of a LMN lesion e.g. wasted fasciculating tongue, absent jaw jerk

262
Q

A 54 year old gentleman with worsening dysphagia, progressing from solids to liquids. Barium swallow reveals a rat’s tail appearance

a. Achalasia
b. Plummer-Vinson Syndrome
c. Oesophageal cancer
d. Peptic stricture
e. Pharyngeal pouch
f. Reflux oesophagitis
g. Oesophageal candidiasis
h. Myasthenia gravis
i. Bulbar palsy
j. Hiatus hernia
k. Globus hystericus
l. Pseudobulbar palsy

A

Oesophageal cancer:

  • Male with progressively worsening dysphagia should be alarming
  • Look for other symptoms of potential neoplasia e.g. weight loss, anorexia
  • Symptoms of local invasion e.g. hoarseness, odynophagia
  • Main risk factors for Ca oesophagus are smoking and alcohol, Barrett’s oesophagus (precursor to adenocarcinoma)
  • Rat’s tail appearance on barium swallow
263
Q

A 17 year old girl with a history of polyuria and polydipsia presents to the emergency department with severe shortness of breath, nausea and vomiting. ABG shows pH 7.25, pO2 12kPa, pCO2 3.1kPa, bicarbonate 14 mmol/l

a. Uncompensated metabolic acidosis
b. Uncompensated respiratory acidosis
c. Uncompensated respiratory alkalosis
d. Uncompensated metabolic alkalosis
e. Partially compensated metabolic acidosis
f. Type I respiratory failure
g. Partially compensated respiratory acidosis
h. Partially compensated metabolic alkalosis
i. Fully compensated respiratory acidosis
j. Type II respiratory failure
k. Fully compensated metabolic acidosis

A

Partially compensated metabolic acidosis:

  • Based on history and presentation, the patient likely has type I diabetes and is presenting in DKA
  • Low pH suggests acidosis
  • Low bicarbonate suggests it is a metabolic acidosis (consistent with clinical picture) -Low CO2 and SOB suggests degree of respiratory compensation (Kussmaul breathing) -However as pH remains acidotic, is not fully compensated
264
Q

A 65 year old long term smoker is admitted with drowsiness and confusion. ABG reveals pH 7.36, pO2 7kPa, pCO2 8kPa, bicarbonate 24 mmol/l

a. Uncompensated metabolic acidosis
b. Uncompensated respiratory acidosis
c. Uncompensated respiratory alkalosis
d. Uncompensated metabolic alkalosis
e. Partially compensated metabolic acidosis
f. Type I respiratory failure
g. Partially compensated respiratory acidosis
h. Partially compensated metabolic alkalosis
i. Fully compensated respiratory acidosis
j. Type II respiratory failure
k. Fully compensated metabolic acidosis

A

Type II respiratory failure:

  • pH is normal
  • Main abnormality is low p02 and high pCO2
  • Combination of hypoxia and hypercapnia
  • This is seen in type II respiratory failure
  • Seen in e.g. COPD, neuromuscular disorders
  • Cf. type I respiratory failure: purely hypoxia, w/o hypercapnia
265
Q

A 25 year old alcoholic presents with severe vomiting and epigastric pain. ABG demonstrates pH 7.51, pO2 12kPa, PCO2 5.2 kPa, bicarbonate 36 mmol/l

a. Uncompensated metabolic acidosis
b. Uncompensated respiratory acidosis
c. Uncompensated respiratory alkalosis
d. Uncompensated metabolic alkalosis
e. Partially compensated metabolic acidosis
f. Type I respiratory failure
g. Partially compensated respiratory acidosis
h. Partially compensated metabolic alkalosis
i. Fully compensated respiratory acidosis
j. Type II respiratory failure
k. Fully compensated metabolic acidosis

A

Uncompensated metabolic alkalosis

  • History is suggestive of pancreatitis
  • Severe vomiting has caused loss of stomach acid, so alkaline pH
  • High bicarbonate confirms this is a likely metabolic alkalosis
  • There doesn’t appear to be any effort to compensate as the PCO2 is within the normal range
266
Q

A 24 year old female presents with an aspirin overdose. She appears to be breathing heavily. pH 7.38, PO2 12kPa, pCO2 2.5kPa, bicarbonate 16 mmol/l

a. Uncompensated metabolic acidosis
b. Uncompensated respiratory acidosis
c. Uncompensated respiratory alkalosis
d. Uncompensated metabolic alkalosis
e. Partially compensated metabolic acidosis
f. Type I respiratory failure
g. Partially compensated respiratory acidosis
h. Partially compensated metabolic alkalosis
i. Fully compensated respiratory acidosis
j. Type II respiratory failure
k. Fully compensated metabolic acidosis

A

Fully compensated metabolic acidosis:

  • pH is normal -Low bicarbonate -Low pCO2
  • Based on the history of salicylate poisoning and pH being on the acidic side of normal, this indicates a likely metabolic acidosis which has been fully compensated by respiratory mechanism – hence low PCO2 and history of heavy breathing
267
Q

A 55 year old male admitted to hospital with difficulty breathing is diagnosed with Guillain Barre syndrome. pH 7.25, PO2 12kPa, pCO2 9kPa, bicarbonate 25 mmol/l

a. Uncompensated metabolic acidosis
b. Uncompensated respiratory acidosis
c. Uncompensated respiratory alkalosis
d. Uncompensated metabolic alkalosis
e. Partially compensated metabolic acidosis
f. Type I respiratory failure
g. Partially compensated respiratory acidosis
h. Partially compensated metabolic alkalosis
i. Fully compensated respiratory acidosis
j. Type II respiratory failure
k. Fully compensated metabolic acidosis

A

Uncompensated respiratory acidosis

  • pH is acid
  • High PCO2
  • With history of neuromuscular problem indicates a respiratory acidosis -No effort at metabolic compensation, as bicarbonate is normal
268
Q

A 25 year old man presented yesterday in severe pain with a distended abdomen. An AXR done upon admission shows grossly distended large bowel. He is in theatre now as conservative management over the last 24 hours has failed.

What does this patient have?
A. Tracheostomy
B. Loop colostomy
C. End ileostomy
D. Gastrostomy
E. De-functioning colostomy 
F. Nephrostomy
G. Loop ileostomy 
H. End colostomy 
I. Cystostomy
J. Urostomy
K. De-functioning ileostomy 
L. None of the above
A

End Ileostomy

269
Q

35 year old woman with UC had a curative procedure and was left with a stoma about 3 months ago. She was back in theatre yesterday for the planned second part of her treatment.

What does this patient have?
A. Tracheostomy
B. Loop colostomy
C. End ileostomy
D. Gastrostomy
E. De-functioning colostomy 
F. Nephrostomy
G. Loop ileostomy 
H. End colostomy 
I. Cystostomy
J. Urostomy
K. De-functioning ileostomy 
L. None of the above
A

None of the above

270
Q

50 year old man presents to his GP for a medication review. He has a stoma and on his notes you read that he used to work in the rubber industry.

What does this patient have?
A. Tracheostomy
B. Loop colostomy
C. End ileostomy
D. Gastrostomy
E. De-functioning colostomy 
F. Nephrostomy
G. Loop ileostomy 
H. End colostomy 
I. Cystostomy
J. Urostomy
K. De-functioning ileostomy 
L. None of the above
A

Urostomy

271
Q

65 year old man is the patient in one of your surgical finals. He has a stoma on the left side of his abdomen that is double-barreled and expelling watery, green-ish fluid. He tells you this is normal and happens on occasion.

What does this patient have?

A. Tracheostomy
B. Loop colostomy
C. End ileostomy
D. Gastrostomy
E. De-functioning colostomy 
F. Nephrostomy
G. Loop ileostomy 
H. End colostomy 
I. Cystostomy
J. Urostomy
K. De-functioning ileostomy 
L. None of the above
A

Loop ileostomy

272
Q

75 year old woman has had a Hartmann’s procedure.

A. Tracheostomy
B. Loop colostomy
C. End ileostomy
D. Gastrostomy
E. De-functioning colostomy 
F. Nephrostomy
G. Loop ileostomy 
H. End colostomy 
I. Cystostomy
J. Urostomy
K. De-functioning ileostomy 
L. None of the above
A

End colostomy

273
Q

A 50 year old African man is found to have a blood pressure of 145/95 on 3 separate occasions. How should this patient be managed?

A.  Continue monitoring
B.  Prescribe Nifedipine
C.  Prescribe Enalapril
D.  Prescribe Chlortalidone 
E.  Prescribe Atenolol
A

Prescribe Nifedipine: african man with confirmed HTN - CCB indicated

274
Q

A 22 year old woman has a 3 month history of abdominal discomfort and tiredness. She has been passing offensive stool which is difficult to flush. Select the most specific autoantibody:

A.  Anti-intrinsic factor
B.  Anti-mitochondrial
C.  Anti-smooth muscle
D.  Anti-endomyseal
E.  Anti-acetylcholinesterase
A

Anti-endomyseal: coeliac disease (or anti-gliadin)

275
Q

A 45 year old man is admitted with headache and neck stiffness. A lumbar puncture is performed and reveals; clear fluid, lymphocytes 2/mm3, neutrophils 1/mm3, low glucose, high protein, positive India ink stain. What is the cause?

A.  Mumps
B.  Neisseria meningitidis 
C. Cryptococcus neoformans 
D. Mycobacterium tuberculosis 
E.  Multiple sclerosis
A

Cryptococcus neoformans

276
Q

A 26 year old woman with chronic renal failure secondary to renal dysplasia has a corrected calcium of 2.6mmol/L, a phosphate of 2.1mmol/L and a significantly raised parathyroid hormone

A.  Primary hyperparathyroidism
B.  Secondary hyperparathyroidism
C.  Tertiary hyperparathyroidism
D.  Parathyroid hormone-related protein secretion 
E.  Iatrogenic calcium overdose
A

Tertiary hyperparathyroidism: • Occurs following prolonged secondary hyperparathyroidism
• High serum calcium and high PTH

277
Q

FBC of a 45 year old man treated for gastric carcinoma with gastrectomy reveals macrocytic anaemia

A.  Anaemia of chronic disease
B.  Vitamin B12 deficiency anaemia 
C.  Aplastic anaemia
D.  Autoimmune haemolytic anaemia 
E.  Iron deficiency anaemia
A

Vitamin B12 deficiency anaemia: • Gasterctomy = no IF • Schilling test

278
Q

A 21 year old woman took a significant overdose of propranolol that she was prescribed for anxiety. On admission she has a heart rate of 50bpm and a BP of 86/42mmHg. What is the appropriate antidote?

A.  Digibind
B.  Sodium bicarbonate infusion 
C.  Flumazenil
D.  Glucagon
E.  Atropine
A

Atropine : • Beta blocker overdose = braycardia, hypotension, syncope and HF
• If haemodynamically unstable = 3mg IV atropine and fluid resuscitation
• If fails to reverse with atropine and stable give glucagon

279
Q

A 4 year old boy presents with facial and ankle swelling. He is found to have 3+ of protein in his urine on dipstick analysis. Blood test reveals an albumin of 16g/L

A.  Mesangiocapillary glomerulonephritis 
B.  Membranous glomerulonephritis
C.  Focal segmental glomerulosclerosis 
D.  Minimal-change glomerulonephritis 
E.  Goodpasture’s syndrome
A

Minimal-change glomerulonephritis
• Though to be secondary to abnormal T cells
• Nephrotic syndrome = proteinuria, peripheral oedema, hypoalbuminaemia, HTN, hyperlipidaemia, hypercoagulability
• In children main cause = minimal change

280
Q

A 70 year old man presents with a 6 month history of worsening forgetfulness and intellectual decline. He has had frequent episodes of relative lucidness during this period. He tries to chase elephants he thinks he sees in the garden but his gait has markedly slowed down. His sleeping pattern is now irregular.

A.  Alzheimer’s disease
B.  Normal-pressure hydrocephalus 
C.  Lewy body dementia
D.  Wernicke-Korsakoff syndrome 
E.  Vascular dementia
A

Lewy body dementia: • 2nd most common dementia
• Day to day fluctuations, hallucinations, sleep disturbance, transient loss of consciousness, falls, parkinsonian features
• NO antipsychotics
• Abnormalities in cytoplasm of neurons in cerebral cortex

281
Q

A fit and well 67 year old man attends his GP for an annual check-up. OE the doctor notices an expansile pulsatile mass in the patient’s abdomen. An outpatient ultrasound scan shows a 6.5cm AAA. What is the most appropriate management?

A. Elective surgical repair
B. No intervention necessary
C. Annual ultrasound monitoring
D. Oral antihypertensive medication and close BP monitoring
E. Maintain systolic BP at 150mm/Hg with fluid resuscitation while awaiting emergency repair

A

Elective surgical repair:

• 6 elective repair

282
Q

Which of these is a relative contraindication to thrombolysis?

A.  History of haemorrhagic stroke
B.  Recent head injury
C.  Ischaemic stroke in the past 3 months 
D.  Clotting disturbance
E.  Possible dissecting aortic aneurysm
A
Recent head injury
Absolute
• Active bleeding
•  History of haemorrhagic stroke
•  Ischaemic stroke in previous 3 months
•  Clotting disturbance
•  Possible dissecting aortic aneurysm
Relative
• Recent head injury
•  PMH of peptic ulcer disease •  Cerebral neoplasm
•  Liver disease
•  Systolic BP >200mmHg
283
Q

A 60 year old woman complains of pain in her fingers on movement. On examination there is joint tenderness and bony lumps at distal interphalangeal joints. A radiograph shows loss of joint space.

A.  Osteoporosis
B.  Gout
C.  Septic arthritis
D.  Osteoarthritis
E.  Rheumatoid arthritis
A

Osteoarthritis: • Osteoarthritis = loss of joint space, subchondral sclerosis, osteophytes and subchondral cyst formation
• Proximal = Bouchard, distal = Heberden’s

284
Q

A 23 year old woman presents with left sided chest pain that is exacerbated by coughing. It is relieved by aspirin.

A.  Pulmonary embolus 
B.  Pneumothorax
C.  Oesophageal spasm 
D.  Pericarditis
E.  Costochondritis
A

• Localised and easily relieved = costochondritis

285
Q

A 65-year old man with Parkinson’s disease cannot tolerate levodopa-based therapy despite careful titration. Suggest the most appropriate management.

A.  Apomorphine 
B.  Bromocriptine 
C.  Ondansetron 
D.  Benzhexol
E.  Domperidone
A

Bromocriptine: • If levodopa inadequate/ not tolerated bromocriptine
• Can cause hypotension and fibrotic reactions

286
Q

A 75 year old man presents with headache and a painful red congested eye with a dilated non- responsive pupil. Select the most appropriate management:

A.  Amitriptyline 
B.  Prednisolone 
C.  Propranolol 
D.  Pethidine
E.  Acetazolamide
A

Acetazolamide: • Acute glaucoma • Medical emergency • Dark and vomitting
• Acetazolamide reduces aquous humour production

287
Q
A 29 year old man presents with a painless swelling on the dorsum of the right hand. Examination reveals a smooth, spherical, tense 1.5cm swelling. The overlying skin can be drawn over it.
A.  Neurofibroma 
B.  Lipoma
C.  Sebaceous cyst 
D.  Ganglion
E.  Keratoacanthoma
A

Ganglion: • Usually found around joints especially dorsal surface of the wrist
• Cystic degeneration of fibrous tissue • Excised under GA

288
Q

A stable COPD sufferer continues to be breathless despite treatment with short acting bronchodilators. An FEV1 measurement is

A

Add tiotropium (long acting muscarinic antagonist)

  • First line inhaled therapy is with short acting bronchodilators
  • If this does not control symptoms, then base step on FEV1
  • If FEV1 >50% predicted, add either long acting B2 agonist (LABA) or long acting muscarinic antagonist (LAMA)
  • If
289
Q

A 35 year old male is rushed into A and E with depressed consciousness. His eyes only open in response to pain, he can only make incomprehensible sounds, and he withdraws from painful stimulus. What is his GCS?

a. 5
b. 6
c. 7
d. 8
e. 9
f. 10

A

M-4
V- 2
E- 2

290
Q

A 24 year old male is admitted to hospital with a scrotal lump. On examination there is a well defined painless swelling separate to the testis, which is fluctuant and transilluminable. What is the most likely diagnosis?

a. Varicocele
b. Spermatocele
c. Inguinal hernia
d. Hydrocele
e. Epididymal cyst
f. Seminoma

A

Epididymal cyst: -Fluctuant and transilluminable indicates this is a cystic lump

  • Separate from the testis, so can rule out hydrocele (where testis is felt through swelling
  • From options most likely to be an epididymal cyst
  • NB: a spermatocele is an epididymal cyst filled with fluid containing spermatozoa. Can only know this through aspiration. But makes no difference to management
291
Q

A 55 year old male presents with a few months history of lethargy, weight loss and LUQ pain. Examination reveals massive splenomegaly. A peripheral blood film is leukoerythroblastic. What is the most likely diagnosis?

a. Chronic myeloid leukaemia
b. Myelofibrosis
c. Waldenstroms macroglobulinaemia
d. Polycythaemia rubra vera
e. Chronic lymphocytic leukaemia
f. Myeloma

A

Myelofibrosis: -Massive splenomegaly in haematological context can be seen in CML, myelofibrosis, CLL. Also in malaria, leishmaniasis

  • Give away is a leukoerythroblastic blood film (presence of primitive WBCs and primitive RBCs – nucleated erythrocytes). Seen in myelofibrosis
  • In myelofibrosis question may also mention teardrop poikilocytosis
292
Q

A 65 year old female presents with shortness of breath. On examination she has an early diastolic murmur, and a diastolic murmur is audible when the femoral artery is compressed. What is this sign?

a. Duroziez’s sign
b. De Musset’s sign
c. Traube’s sign
d. Corrigan’s sign
e. Muller’s sign
f. Landolfi’s sign

A

Duroziez’s sign

  • Signs of aortic regurgitation
  • Duroziez’s = diastolic murmur heard over compressed femoral
  • Traube’s = pistol-shot sounds over femoral
  • Corrigan’s = visible carotid pulsation
  • De Musset’s = head-nodding in time with heartbeat
  • Quincke’s = capillary bed pulsations
293
Q

Which of these markers would show that a person had developed chronic HBV infection?

a. Hep B core antigen (HbCAg)
b. Hep B surface antigen (HbSAg)
c. Hep B surface antibody (HbSAb)
d. Hep B core antibody (HbCAb)
e. Hep B E antigen (HbEAg)
f. Alpha fetoprotein

A

Hep B surface antigen (HbSAg)

  • Presence of Hep B surface antibody alone (HbSAb) indicates vaccination
  • Presence of both HbSAb and HbCAb alone indicate immunity to natural infection
  • Hep B surface antigen (HbSAg) is seen in acute infection. Its persistence beyond 6 months indicates chronic HBV infection
  • Presence of HbEAg indicates a person is highly infectious. It is seen in both acute and chronic HBV
  • Hep B core antibody (HbCAb) rises a few weeks after acute infection and also detectable in chronic infection
  • Hep B core antigen is NOT DETECTABLE from serology (as it is intrahepatic)
294
Q

A 45 year old Asian male with type II diabetes has failed to achieve adequate glycaemic control despite lifestyle and dietary changes, metformin, gliclazide and pioglitazone, with an HBA1c of 8.5%. What is the next step?

a. Add nateglinide
b. Add acarbose
c. Islet cell transplant
d. Increase dose of metformin
e. Add insulin
f. Add glucagon

A

Add insulin:

  • Start with lifestyle modification
  • This alone rarely achieves good glycaemic control so need oral hypoglycaemics
  • Metformin ( or if not overweight can consider a sulfonylurea)
  • If HbA1C is still not adequately controlled, add sulfonylurea (if not already on it)
  • If still failure to control, add another oral hypoglycaemic e.g. a thiazolidinedione
  • If on maximal oral hypoglycaemic therapy and lifestyle advice (as in this case), add insulin
295
Q

66 year old female presents with a cough productive of purulent sputum and fever. Chest x-ray shows evidence of consolidation. Investigations reveal that she has an AMTS of 9/10, urea 7.8mmol/l, respiratory rate 32/min, and blood pressure 107/70. What is the severity of her pneumonia based on the CURB 65 score?

a. 0
b. 1
c. 2
d. 3
e. 4
f. 5

A

3: Confusion (based on AMTS score of 8/10 or less)
Urea (>7mmol/l)
Respiratory rate (>30/min)
Blood pressure (either 65)

296
Q

Which of these tumour markers is associated with carcinoma of the colon?

a. CA19-9
b. CA15-3
c. AFP
d. B-hCG
e. CA125
f. CEA

A

CEA

  • Ca 19-9 = pancreatic cancer
  • Ca 15-3 = breast cancer
  • Ca-125 = ovarian cancer
  • AFP = HCC, testicular teratoma
  • Carcinoembryonic antigen (CEA) = colorectal cancer
  • bHCG = seminoma, teratoma
297
Q

A 47 year old male becomes acutely unwell a few days following a tooth extraction. Examination reveals a pansystolic murmur radiating to the axilla and a temperature of 39 deg. Blood cultures have been sent but results are unavailable. What is the most appropriate investigation?

a. ECG
b. Myocardial perfusion scan
c. Transthoracic echocardiogram
d. Transoesophageal echocardiogram
e. Chest x-ray
f. Serum LDH

A

Transoesophageal echocardiogram: -History is classic for infective endocarditis,with a likely mitral regurgitation murmur

  • Of all the modalities listed, best method is transoesophageal echocardiography, which detects around 90% of vegetations, and is good at viewing prosthetic valves, compared to transthoracic echocardiography
  • Latter usually is sufficient in IE, but the best modality is TOE
298
Q

A 60 year old female has been started on a new drug for her blood pressure. She develops bilateral ankle swelling a few weeks into the treatment. Which is the most likely offending drug?

a. Losartan
b. Atenolol
c. Amlodipine
d. Captopril
e. Bendroflumethiazide
f. Doxazosin

A

Amlodipine: -ACE inhibitors classically cause a dry cough

-Calcium channel blockers e.g. amlodipine can cause ankle oedema

299
Q

A 42 year old male presents with a 3 day history of abdominal pain, absolute constipation and distension. After basic management has been instigated what is the most appropriate next step?

a. MRI
b. Abdominal ultrasound
c. Diagnostic laparoscopy
d. Abdominal x-ray
e. CT abdomen
f. Colonoscopy

A

Abdominal Xray: -History indicates most likely intestinal obstruction

  • As vomiting is not a prominent feature but constipation is, suggests a large bowel obstruction
  • A plain abdominal radiograph is an appropriate next modality to confirm that there is indeed intestinal obstruction (note difference between small and large bowel obstruction appearances)
  • A contrast CT scan could be done after this if necessary to further visualise obstruction
300
Q

A 36 year old female presents with intermittent attacks of severe dizziness. This is accompanied by a persistent ringing in her ears. She also notes that her hearing in her left ear seems to have got steadily worse. What is the most likely diagnosis?

a. Meniere’s Disease
b. Benign paroxysmal positional vertigo
c. Acute labyrinthitis
d. Ramsay Hunt Syndrome
e. Multiple sclerosis
f. Otitis media

A

Meniere’s Disease: -Typical triad of tinnitus, vertigo and fluctuating sensorineural deafness seen in Meniere’s Disease

  • Other options: BPPV is very positional (usually when turning head in bed etc) and not associated with hearing problems
  • Acute labyrinthitis: usually very sudden onset severe vertigo, hearing loss and nausea and vomiting
  • Ramsay-Hunt syndrome: Herpes zoster virus affecting geniculate ganglion. Usually you see a herpetic rash around ear, and can get facial nerve palsy
301
Q

A 34 year old male presents with a history of fever, night sweats and haemoptysis. Examination reveals a tender fluctuant lump in the groin, with pain on passive extension of the hip. What is most likely diagnosis?

a. Ectopic testis
b. Strangulated inguinal hernia
c. Psoas bursa
d. Saphena varix
e. Psoas abscess
f. Inguinal lymphadenopathy

A

Psoas abscess: -The history is suggestive of tuberculosis

  • The lump could be a reactive lymphadenopathy, but as it is fluctuant and there is pain on passive hip extension, suggests a psoas abscess
  • Caused by infection of unknown origin or infection spreading from adjacent structures -In this case, possibly TB from spine has led to formation of abscess
302
Q

“AV nipping” is a retinal sign seen first in what circumstance?

a. Background diabetic retinopathy
b. Grade III hypertensive retinopathy
c. Grade II hypertensive retinopathy
d. Grade I Hypertensive retinopathy
e. Grade IV hypertensive retinopathy
f. Pre-proliferative diabetic retinopathy

A

Grade II hypertensive retinopathy:
Grade I: silver and copper wiring (narrowing of arterioles)
Grade II: As above but also with AV-nipping
Grade III: As grade II but also with cotton wool spots and flame haemorrhages Grade IV: As above but also with papilloedema

303
Q

A 28-year-old man presents with a lump in the posterior triangle of his neck. He mentions that he has had difficulty finishing his meals and sometimes regurgitates his food at night. On palpation of the lump you hear a gurgling sound.

A. Pharyngeal pouch
B. Carotid body tumour
C. Torticollosis
D. Goitre
E. Sternocleidomastoid tumour
F. Cervical rib
G. Dermoid cyst
H. Thyroglossal cyst
I. Cystic hygroma
A

A pharyngeal pouch develops from backwards protrusion of the pharyngeal mucosa through a weak area of the pharyngeal wall (known as Killian’s dehiscence). The pouch initially develops posteriorly, but then protrudes to one side, usually the left, displacing the oesophagus laterally. They occur most often in elderly men who present with a characteristic history of dysphagia, regurgitation of food and sensations of gurgling in the neck. Inhalation of regurgitated food may lead to fits of coughing and aspiration at night.

304
Q

A 19-year-old woman presents with a small painless swelling in the midline of her neck. It moves upwards on protrusion of the tongue.

A. Pharyngeal pouch
B. Carotid body tumour
C. Torticollosis
D. Goitre
E. Sternocleidomastoid tumour
F. Cervical rib
G. Dermoid cyst
H. Thyroglossal cyst
I. Cystic hygroma
A

A thyroglossal cyst is a congenital remnant of the thyroglossal duct (which begins at the base of the tongue). The cyst itself can appear anywhere between the base of the tongue and the isthmus of the thyroid gland. It usually presents in the first decade as a smooth midline lump that is painless to touch and moves up on tongue protrusion (due to its attachment to the tongue via the tract of thyroid descent).

305
Q

A 32-year-old basketball player presents with pain and paraesthesia down the medial aspect of his left arm. He claims his symptoms worsen when he raises her arms for a slam dunk. On examination, there is a firm lump at the base of his neck on the left side.

A. Pharyngeal pouch
B. Carotid body tumour
C. Torticollosis
D. Goitre
E. Sternocleidomastoid tumour
F. Cervical rib
G. Dermoid cyst
H. Thyroglossal cyst
I. Cystic hygroma
A

A cervical rib is a congenital overdevelopment of the transverse process of the seventh cervical verebrae. This ‘rib’ may interfere with the subclavian artery and the lower roots of the brachial plexus leading to a form of thoracic outlet syndrome. If the T1 root is affected (as in this patient), there is pain and paraesthesia along the medial aspect of the arm as well as wasting of the small muscles of the hand. Symptoms may worsen when the arms are elevated, such as during a slam dunk.

306
Q

A 3-month year old boy is brought into the clinic by his parents. He has a lump at the base of the neck, posterior to the left sternocleidomastoid muscle. On examination, it is compressible and transilluminates brilliantly.

A. Pharyngeal pouch
B. Carotid body tumour
C. Torticollosis
D. Goitre
E. Sternocleidomastoid tumour
F. Cervical rib
G. Dermoid cyst
H. Thyroglossal cyst
I. Cystic hygroma
A

A cystic hygroma is a congenital benign collection of lymphatic sacs. The cysts contain clear fluid and characteristically transilluminate brightly. It presents as a soft, fluctuant lump just beneath the skin and usually occurs in the posterior triangle of the neck.
Cystic hygromas are large singular or multilocular fluid-filled cavities that arise from the jugular lymph sacs. Lymph sacs are embryonic lymph node precursors: there are also iliac and retroperitoneal lymph sacs. The jugular lymph sacs lie near the junction of the subclavian veins with the anterior cardinal veins (the future internal jugular veins).

Lymphatic vessels join the sacs and pass along the head, neck and upper limbs from the jugular lymph sacs.

Most cystic hygromas appear to be derived from abnormal transformation of the jugular lymph sacs.

As you can see, therefore, the cystic hygroma could technically develop anywhere that the jugular lymph sacs spread to. Over 60% (according to GP notebook - different papers report different statistics) occur in the neck region, but they have also been reported on the chest wall and in the axilla and arm.

Classically, cystic hygromas appear in the inferolateral part of the neck.

307
Q

A 56-year-old woman presents with a lump in the anterior triangle of the neck. She says it has been slowly growing over the past few months. On examination, the mass is pulsatile and moves side to side only.

A. Pharyngeal pouch
B. Carotid body tumour
C. Torticollosis
D. Goitre
E. Sternocleidomastoid tumour
F. Cervical rib
G. Dermoid cyst
H. Thyroglossal cyst
I. Cystic hygroma
A

A carotid body tumour (or chemodectoma) is a slowly growing tumour of the paraganglion cells arising in the carotid body at the carotid bifurcation. It usually tansmits the carotid pulsation and may have an associated bruit. It characteristically moves from side-to-side, but not up-and-down. Pressure on the tumour may cause dizziness and syncope due to stimulation of vagal tone via the carotid sinus. These lesions are often taken care of by vascular surgeons as treatment sometimes requires reconstruction of the carotid arteries.

308
Q

A 2-month-year-old boy is brought in to see you by his parents as they are worried about a lump in the right side of his neck. On examination you notice that the lump is hard and fixed to the underlying muscle and the baby’s head is tilted towards it.

A. Lymphoma
B. Goitre
C. Sialolithiasis
D. Thyroglossal cyst
E. Dermoid cyst
F. Branchial cyst
G. Sternocleidomastoid tumour
H. Salivary duct carcinoma
A

The term ‘tumour’ is actually a misnomer as it is a fibrous mass in one of the sternocleidomastoid muscles. This condition is often associated with torticollis (tilting of the head) due to contraction of the sternocleidomastoid muscle itself. The etiology of this condition is uncertain, however it is thought to reuslt from intrauterine or birth trauma leading to secondary pressure necrosis and fibrosis of the muscle.

309
Q

A 30-year-old man presents with a lump in the posterior triangle of his neck. He says it has been enlarging slowly for the past 2 months and is painless. He also reveals that he has felt feverish, itchy and sweaty at night for this period of time and has lost considerable weight.

A. Lymphoma
B. Goitre
C. Sialolithiasis
D. Thyroglossal cyst
E. Dermoid cyst
F. Branchial cyst
G. Sternocleidomastoid tumour
H. Salivary duct carcinoma
A

This man’s clinical features are highly suggestive of lymphoma. The symptoms of fever, weight loss and night sweats constitute what are known as B symptoms.

310
Q

A 25-year-old woman presents with a smooth lump in the anterior triangle of the neck close to the mideline. You examine her and find that the lump is not painful and moves on swallowing, but not on protrusion of the tongue. She is otherwise asymptomatic.

A. Lymphoma
B. Goitre
C. Sialolithiasis
D. Thyroglossal cyst
E. Dermoid cyst
F. Branchial cyst
G. Sternocleidomastoid tumour
H. Salivary duct carcinoma
A

This patient has a simple thyroid goitre, evidenced by the clinical findings and normal thyroid function. Goitres are more commonly seen in women and can occur worldwide due to iodine deficiency, but also commonly occur during pregnancy and puberty (when there is increased metabolic demand).

311
Q

A 70-year-old woman complains of a mass on her right jaw, which has been slowly growing over the past 6 months. She says that since 1 week ago, she has been unable to move the right side of her face. On examination you notice a hard, immobile lump with irregular edges extending up behind the angle of the mandible.

A. Lymphoma
B. Goitre
C. Sialolithiasis
D. Thyroglossal cyst
E. Dermoid cyst
F. Branchial cyst
G. Sternocleidomastoid tumour
H. Salivary duct carcinoma
A

Salivary duct carcinoma is an uncommon tumour of the salivary glands. There are a number of variants of malignant tumours in the salivary gland, the most common of which is the mucoepidermoid tumour. Facial nerve involvement (as in this case) is highly suggestive of malignancy.

312
Q

A 25-year-old man presents with a painful swelling which bulges from beneath the anterior border of his left sternomastoid muscle. On examination, it is soft and fluctuant, but tender. He says it has been present since childhood and only started causing him trouble following a respiratory infection.

A. Lymphoma
B. Goitre
C. Sialolithiasis
D. Thyroglossal cyst
E. Dermoid cyst
F. Branchial cyst
G. Sternocleidomastoid tumour
H. Salivary duct carcinoma
A

A branchial cyst is a remnant of the second branchial cleft, which normally involutes during embryonic development. It commonly presents in young adults as a smooth, non-tender, fluctuant swelling in the anterior triangle anterior to the border of the sternocleidomastoid at the junction of its upper and middle thirds (this position is characteristic). It may become enlarged and inflamed with upper respiratory tract infections.

313
Q

A 30-year-old builder presents after noticing a lump in his groin after a day at work. On examination you notice that is has a cough impulse, is reducable and is located above and medial to the pubic tubercle. The mass descends into the scrotum on standing.

A. Direct inguinal hernia
B. Femoral hernia
C. Inguinal lymphadenopathy
D. False femoral artery aneurysm
E. Femoral artery aneurysm
F. Saphena varix
G. Indirect inguinal hernia
H. Psoas abscess
A

Inguinal hernias (both direct and indirect) occur above and medial to the pubic tubercle and are more common in men. Indirect inguinal hernias arise from the deep inguinal ring, travel down the inguinal canal, and protrude through the superficial ring and into the scrotum (if the hernial sac is sufficiently large). They are more common on the right side in males, because the right testis descends later and there is an increased incidence of failed closure of the processus vaginalis (thus predisposing to hernias). On the other hand, direct inguinal hernias protrude directly through an area of weakness in the posterior wall of the inguinal canal (an area called Hesselbach’s triangle) rather than down the canal itself. They are rare in children and common in the elderly because of aetiological factors such as chronic cough or constipation.

314
Q

A 60-year-old man complains of a lump in the groin. He says it enlarges if he coughs or laughs. On examination, he has an expansile mass just lateral to the femoral vein, which transmits a pulse. The mass is irreducible.

A. Direct inguinal hernia
B. Femoral hernia
C. Inguinal lymphadenopathy
D. False femoral artery aneurysm
E. Femoral artery aneurysm
F. Saphena varix
G. Indirect inguinal hernia
H. Psoas abscess
A

A swelling that is both pulsatile and expansile is an aneurysm. True aneurysms of the superficial femoral artery are rare, with most peripheral aneurysms occuring in the politeal artery. A femoral aneurysm may occasionally spontaneously rupture; whereas a popliteal aneurysm may thrombose and present as an acutely ischaemic leg.

315
Q

A 59-year-old man has recently undergone an angiogram of his lower limbs to investigate his claudication. He is now back on the ward and is complaining of a slightly tender mass on his medial right thigh. The mass is firm and transmits a pulse

A. Direct inguinal hernia
B. Femoral hernia
C. Inguinal lymphadenopathy
D. False femoral artery aneurysm
E. Femoral artery aneurysm
F. Saphena varix
G. Indirect inguinal hernia
H. Psoas abscess
A

False femoral aneurysms (or pseudoaneurysms), unlike true aneurysms, do not involve the vessel wall. Rather, they represent an accumulation of blood (haematoma) in a cavity that is held around the vessel by connective tissue. They are a major complication arising from arterial surgery and can occur following cannulation for angiography, or incorrect placement of needles by IVDUs as this leads to breach in the vessel wall and the extravasation of blood. They usually present as a pulsatile (not necessarily expansile) mass in the groin with a history of a surgical procedure/trauma to the region.

316
Q

A 39-year-old woman, who recently gave birth to her third child, presents with a swelling on her right thigh, below and lateral to the pubic tubercle. On examination, the swelling is golf-ball shaped, compressible and demonstrates a fluid thrill. It appears to have a bluish tinge to it.

A. Direct inguinal hernia
B. Femoral hernia
C. Inguinal lymphadenopathy
D. False femoral artery aneurysm
E. Femoral artery aneurysm
F. Saphena varix
G. Indirect inguinal hernia
H. Psoas abscess
A

A saphena varix is a dilatation of the long saphenous vein that occurs due to incompetent vales at the saphenofemoral junction. It has a cough impulse and disappears when lying down and may be mistaken for a hernia. However, it often has a bluish tinge and exhibits a fluid thrill when the long saphenous vein is tapped distally (Schwart’s percussion test). On auscultation a venous hum may be heard.

317
Q

A 35-year-old Indian man presents with a swelling lateral to the femoral artery. On examination it is tender and fluctuant, but cannot be reduced. He also describes a recent history of weight loss and night sweats.

A. Direct inguinal hernia
B. Femoral hernia
C. Inguinal lymphadenopathy
D. False femoral artery aneurysm
E. Femoral artery aneurysm
F. Saphena varix
G. Indirect inguinal hernia
H. Psoas abscess
A

This man’s ethnicity and clinical symptoms point towards a diagnosis of TB. A known complication of intraabdominal TB is abscess formation in the lumbar vertebrae (Pott’s disease). Such an abscess may track down the psoas sheath and into the groin, forming a psoas abscess. Psoas abscesses are described as ‘cold’, as they are painless and not warm to touch.

318
Q

A 35-year-old man presents with a swelling in the right groin, but is otherwise asymptomatic. It appears to descend into the scrotum, is reducible, and can be stopped from descending into the scrotum with application of pressure over the internal inguinal ring.

A. Crohn's disease
B. Ectopic testis
C. Femoral neuroma
D. Caecal carcinoma
E. Strangulated femoral hernia
F. Indirect inguinal hernia
G. Femoral aneurysm
H. Direct inguinal hernia
A

Inguinal hernias (both direct and indirect) occur above and medial to the pubic tubercle and are more common in men. Indirect inguinal hernias arise from the deep inguinal ring, travel down the inguinal canal, and protrude through the superficial ring and into the scrotum (if the hernial sac is sufficiently large). They are more common on the right side in males, because the right testis descends later and there is an increased incidence of failed closure of the processus vaginalis (thus predisposing to hernias). On the other hand, direct inguinal hernias protrude directly through an area of weakness in the posterior wall of the inguinal canal (an area called Hesselbach’s triangle) rather than down the canal itself. They are rare in children and common in the elderly because of aetiological factors such as chronic cough or constipation.

319
Q

A 78-year-old woman is brought into A&E complaining of colicky abdominal pain and constipation. She has vomited a few times and appears confused and dehydrated. On examination you spot a grape-sized swelling below the right inguinal ligament.

A. Crohn's disease
B. Ectopic testis
C. Femoral neuroma
D. Caecal carcinoma
E. Strangulated femoral hernia
F. Indirect inguinal hernia
G. Femoral aneurysm
H. Direct inguinal hernia
A

Femoral hernias emerge through the femoral canal and may be felt as a soft swelling below and lateral to the pubic tubercle. They are less common than inguinal hernias, but occur more often in women than in men (possibly because of a wider pelvis and femoral canal). They are also more likely to strangulate (as in this case) due to the tightness of the femoral ring.

320
Q

A 63-year-old man with Von Recklinghausen’s disease presents with pain in his left thigh. On examination, there is a hard well-defined swelling lying lateral to the femoral artery. There is no cough impulse and it is irreducable. Pressure on the swelling reproduces the pain in the thigh.

A. Crohn's disease
B. Ectopic testis
C. Femoral neuroma
D. Caecal carcinoma
E. Strangulated femoral hernia
F. Indirect inguinal hernia
G. Femoral aneurysm
H. Direct inguinal hernia
A

Femoral neuromas can arise due to previous trauma or as a feature of neurofibromatosis (Von Recklinghausen’s disease). The femoral nerve lies lateral to the femoral artery and compression of the neuroma results in pain along the nerve’s distribution.

321
Q

A 55-year-old man presents with a swelling below and medial to the pubic tubercle on the left side. The lump is not reduced by pressure over the internal inguinal ring.

A. Crohn's disease
B. Ectopic testis
C. Femoral neuroma
D. Caecal carcinoma
E. Strangulated femoral hernia
F. Indirect inguinal hernia
G. Femoral aneurysm
H. Direct inguinal hernia
A

Direct inguinal hernias protrude directly through an area of weakness in the posterior wall of the inguinal canal (an area called Hesselbach’s triangle) rather than down the canal itself. They are rare in children and common in the elderly because of aetiological factors such as chronic cough or constipation.

322
Q

A 69-year-old man presents to your clinic with lower abdominal pain. A blood test reveals that he has a microcytic anaemia with a Hb of 9.3. On examination you notice a hard, immobile mass in the right iliac fossa.

A. Crohn's disease
B. Ectopic testis
C. Femoral neuroma
D. Caecal carcinoma
E. Strangulated femoral hernia
F. Indirect inguinal hernia
G. Femoral aneurysm
H. Direct inguinal hernia
A

Caecal Carcinoma: The microcytic anaemia and age of the patient point towards an underlying gatrointestinal malignancy. Caecal carcinoma is a common cause of asymptomatic anaemia, however it may also present with right iliac fossa pain, malaise and symptoms of obstruction..

323
Q

A 16-year-old boy presents with sudden onset severe pain in his right scrotum and vomiting. On examination, the right testiss appears swollen and is hanging higher than the left.

A. Teratoma
B. Mumps orchitis
C. Testicular torsion
D. Seminoma
E. Varicocele
F. Torsion of the hyatid of Morgagni
G. Hydrocele
H. Epididymo-orchitis
A

The presentation of sudden-onset pain and swelling in the testicle in a male of this age strongly suggests torsion. This is a surgical emergency and commonly occurs between the age of 10 and 15 years (rarely occuring over the age of 21). The incidence of torsion is higher for undescended testes. Pain may be referred to the abdomen as the testis retains its embryological nerve supply from the T10 sympathetic pathway. Irreversible adamage occurs within 6-12 hours, so urgent surgical intervention is required. It is sometimes difficult to differentiate testicular torsion from acute epididymitis (which presents similarly), however in EMQs you can tell the difference between the two from two key features: 1) in testicular torsion, pain is not relieved by elevating the twisted testis (negative Prehn’s sign), but pain is relieved in acute epidiymitis and 2) in testicular torsion, the affected testis usually lies high in the scrotum, but does not in epididymitis.

324
Q

A 49-year-old man complains of grandually enlarging painless swelling of his left scrotum. HE says it has been present for at least 2 years. On examination, the testis is impalpable, but it is possible to get above the swelling. The swelling transilluminates brightly.

A. Teratoma
B. Mumps orchitis
C. Testicular torsion
D. Seminoma
E. Varicocele
F. Torsion of the hyatid of Morgagni
G. Hydrocele
H. Epididymo-orchitis
A

A hydrocele is an abnormal collection of serious fluid in the tunica vaginalis surrounding the testis (hence the impalpable testis). Primary hydroceles usually occur in older men, develop slowly and are painless. Secondary hydroceles can occur secondary to tumours or inflammation of the underlying testes and epididymis. Characteristically, hydroceles transilluminate brightly.

325
Q

A 28-year-old man complains of a ‘dragging’ sensation and slight discomfort in his scrotum. On examination, the swelling is located above the testis, but is limited to the scrotum. It disappears when the patient lies flat, but there is no cough impulse.

A. Teratoma
B. Mumps orchitis
C. Testicular torsion
D. Seminoma
E. Varicocele
F. Torsion of the hyatid of Morgagni
G. Hydrocele
H. Epididymo-orchitis
A

The description in this case is characteristic of a varicocele. A varicocele is a collection of varicose veins in the pampiniform plexus (the venous network that drains the testicle). Patients present with a scrotal swelling that is visible on standing and characteristically feels like a ‘bag of worms’. It is more common on the left and may occur secondary to obstruction of the left testicular vein by a renal adenocarcinoma.

326
Q

A 35-year-old man with a history of undescended testis presents with a hard painless testicular lump. Blood tests shown an elevated β-human chorionic gonadotrophin.

A. Teratoma
B. Mumps orchitis
C. Testicular torsion
D. Seminoma
E. Varicocele
F. Torsion of the hyatid of Morgagni
G. Hydrocele
H. Epididymo-orchitis
A

Seminoma: There are two main types of malignant testicular tumours: seminomas (60%) and teratomas (40%). Seminomas arise from the seminiferous tubules and occur in the 30-40 age group whereas teratomas are germ-cell tumours which occur in 20-30 year olds. They are associated with undescended testis (even after orchidopexy) and may both present as a hard, irregular, painless mass, or as a secondary hydrocele. They may spread to the lungs and liver (via the blood stream) and to the para-aortic lymph nodes (not inguinal!) Teratomas produce both alpha-fetoprotein (α-FP) and beta-human chorionic gonadotrophin (β-hCG), but seminomas only produce the latter.

327
Q

A 20-year-old man complains of mild pain in his left testicle after playing cricket in the park. On examination the left testicle is irregular in shape compared to the right. He reports having an orchidopexy when he was younger.

A. Teratoma
B. Mumps orchitis
C. Testicular torsion
D. Seminoma
E. Varicocele
F. Torsion of the hyatid of Morgagni
G. Hydrocele
H. Epididymo-orchitis
A

Teratoma: There are two main types of malignant testicular tumours: seminomas (60%) and teratomas (40%). Seminomas arise from the seminiferous tubules and occur in the 30-40 age group whereas teratomas are germ-cell tumours which occur in 20-30 year olds. They are associated with undescended testis (even after orchidopexy) and may both present as a hard, irregular, painless mass, or as a secondary hydrocele. They may spread to the lungs and liver (via the blood stream) and to the para-aortic lymph nodes (not inguinal!) Teratomas produce both alpha-fetoprotein (α-FP) and beta-human chorionic gonadotrophin (β-hCG), but seminomas only produce the latter.

328
Q

A 53-year-old woman presents with mass below the angle of the jaw. She sayd it has been gradually increasing in size for the past 6 months. On examination, it is mobile and firm to the touch. There is no associated pain or facial weakness

Ultrasound
Carotid angiography
Fine-needle aspiration
Excision biopsy
Doppler ultrasound
Sialogram
Iodine uptake scan
Technetium scan
A

The symptoms here are suggestive of a parotid tumour, probably a pleomorphic adenoma (carcinoma would be painful, rapidly growing and may cause facial nerve palsy). Excision biopsy is required to make the diagnosis and treatment.

329
Q

A 47-year-old woman presents with discomfort in her neck and a 3-month history of weight loss. Examination of the neck reveals an irregular multinodular goitre and a radioiodine uptake scan shows a cold nodule. Her TFTs are normal.

Ultrasound
Carotid angiography
Fine-needle aspiration
Excision biopsy
Doppler ultrasound
Sialogram
Iodine uptake scan
Technetium scan
A

Fine-needle aspiration is the diagnostic test of choice used to differentiate benign from malignant thyroid nodules pre-operatively. Many cold nodules are malignant but may be non-secreting adenomas, whilst hot nodules are usually adenomas but may rarely be follicular carcinomas. Therefore, tissue diagnosis is necessary to make a definitive diagnosis.

330
Q

A 32-year-old man complains of painful swelling below his jaw that comes on during mealtimes. He says he has lost weight over the pat 2 months and blames this on the pain brought about by eating. On examination, there is a small, tender swelling in the left submandibular region.

Ultrasound
Carotid angiography
Fine-needle aspiration
Excision biopsy
Doppler ultrasound
Sialogram
Iodine uptake scan
Technetium scan
A

Salivary gland stones commonly occur in the submandibular gland. The description of pain and swelling at meal times, when salivary flow is high, is classic. Confirmation of the diagnosis is made with plain X-ray or sialography, which allows examination of the architecture of the salivary gland and duct.

331
Q

A 60-year-old man presents with a mass in the anterior triangle of the neck along the border of the sternocleidomastoid. It has increased in size slowly over the last two months. It is soft, pulsatile and has a bruit on auscultation.

Ultrasound
Carotid angiography
Fine-needle aspiration
Excision biopsy
Doppler ultrasound
Sialogram
Iodine uptake scan
Technetium scan
A

This mass could be due to either a carotid artery aneurysm or a cartoid body tumour. The diagnosis would be confirmed using carotid angiography, which would be more discriminatory than carotid Doppler.

332
Q

A 16-year-old boy presents with a painless lump on the anterior border of the upper sternocleidomastoid on the left side. On examination, it is smooth and fluctuant, but does not transilluminate. He says that it previously became painful during a throat infection, but hasn’t bothered him since. There are no other abnormalities.

Ultrasound
Carotid angiography
Fine-needle aspiration
Excision biopsy
Doppler ultrasound
Sialogram
Iodine uptake scan
Technetium scan
A

This boy has a branchial cyst. The clues to the diagnosis are the position and the fact that it became inflamed during a respiratory infection. Diagnosis is by fine-needle aspiration again, which demonstrates a creamy yellow turbid fluid containing cholesterol crystals. Treatment is by excision.

333
Q

A 60 year old male smoker has a long history of hypertension & angina. 4 weeks ago he was started on captopril by his GP. His creatinine has increased from 100 to 350 during that time. Renal ultrasound shows that 1 kidney is larger than the other.

A. Hypothyroidism
B. Hyperparathyroidism
C. Chronic alcohol excess
D. Essential hypertension
E. ‘White-coat hypertension’
F. Polycystic kidney disease
G. Cushing's syndrome
H. Conn's syndrome
I. Medication
J. Phaeochromocytoma
K. Acromegaly
L. Aortic coarctation
M. Renal artery stenosis
A

Renal Artery Stenosis: Renal artery stenosis is basically narrowing of the renal artery. It occurs typically due to atherosclerosis or fibromuscular dysplasia. The history of smoking, hypertension and angina here are risk factors of the former. The presentation tends to be with accelerated or difficult to control hypertension. Acute kidney injury can be seen after starting an ACE inhibitor or an angiotensin II receptor antagonist. The afferent arteriole is stenosed in RAS and angiotensin II is needed to maintain GFR by constricting the efferent arteriole. ACE inhibitors prevent conversion of angiotensin I to angiotensin II, which is needed to maintain renal perfusion pressure in those with RAS.
There may not be any clinical consequences of RAS – just because someone’s renal arteries are narrowed does not mean they are suffering worsening kidney function, although this may be the case, especially after blockade of the renin-angiotensin system. A definitive diagnosis is made on imaging, where there is some controversy on what is most appropriate to use. USS is safe and non-invasive but the sensitivity and specificity is low. CT/MR angiography has the risk of contrast nephropathy and nephrogenic systemic fibrosis. Conventional angiography (the best test available) has the risk of bleeding and emboli as well as contrast related risks already mentioned.

334
Q

A 40 year old woman has high blood pressure despite treatment with bendrofluazide & atenolol. Blood tests show Na 140mmol/l, K 3mmol/l, urea 6mmol/l. His bendrofluazide is stopped & he is prescribed potassium supplements, but 2 weeks later his K is still 3mmol/l. Plasma renin activity is low.

A. Hypothyroidism
B. Hyperparathyroidism
C. Chronic alcohol excess
D. Essential hypertension
E. ‘White-coat hypertension’
F. Polycystic kidney disease
G. Cushing's syndrome
H. Conn's syndrome
I. Medication
J. Phaeochromocytoma
K. Acromegaly
L. Aortic coarctation
M. Renal artery stenosis
A

Conn’s Syndrome: The normal range for potassium 3.5-5mmol/l. You should really know the normal ranges for common values like sodium, potassium and urea by this stage. In Conn’s, potassium is normal or low. It is important when drawing blood to avoid haemolysing the sample, which will cause a falsely elevated potassium level. It is important for screening to calculate the aldosterone/renin ratio, with >30 being suggestive of Conn’s. In Conn’s, aldosterone is raised and renin is low due to negative feedback. This is in contrast to renal artery stenosis where both aldosterone and renin will be raised. It is important to discontinue diuretics and other interfering medications for at least 6 weeks prior to measuring the ratio. The most reliable diagnostic test is a fludrocortisone suppression test. Treatment can be surgical with excision of the adenoma (if aldosterone production is lateralised to one side) or medical with spironolactone and amiloride. There are also familial forms of primary hyperaldosteronism which show an autosomal dominant mode of inheritance.
Spironolactone is an aldosterone receptor antagonist. Amiloride inhibits aldosterone-sensitive sodium channels. They are both examples of potassium sparing diuretics acting on the DCT.

335
Q

A 45 year old woman is hypertensive & complains that she is putting on weight. On examination, she is centrally obese & has a moon face. There are purple striae on her abdomen. She has glycosuria.

A. Hypothyroidism
B. Hyperparathyroidism
C. Chronic alcohol excess
D. Essential hypertension
E. ‘White-coat hypertension’
F. Polycystic kidney disease
G. Cushing's syndrome
H. Conn's syndrome
I. Medication
J. Phaeochromocytoma
K. Acromegaly
L. Aortic coarctation
M. Renal artery stenosis
A

Cushing’s Syndrome: There is weight gain (truncal obesity), hypertension, moon face and striae in Cushing’s due to hypercorticolism. Cushing’s disease is due to an ACTH secreting pituitary adenoma and is responsible for most cases of Cushing’s syndrome. A low dose 1mg overnight dexamethasone suppresion test can be done, or a 24 hour urinary free cortisol collection to diagnose Cushing’s syndrome. Plasma ACTH should guide further investigation. If ACTH is suppressed, the problem is likely to be with the adrenals. If it not suppressed, pituitary or ectopic disease is more likely.

336
Q

A 40 year old man is hypertensive & complains that he is putting on weight. On examination he has a prominent jaw and brow. You notice that he is sweating and has large hands and feet. Urine dipstick reveals glycosuria.

A. Hypothyroidism
B. Hyperparathyroidism
C. Chronic alcohol excess
D. Essential hypertension
E. ‘White-coat hypertension’
F. Polycystic kidney disease
G. Cushing's syndrome
H. Conn's syndrome
I. Medication
J. Phaeochromocytoma
K. Acromegaly
L. Aortic coarctation
M. Renal artery stenosis
A

Acromegaly: Acromegaly is caused by excess growth hormone and is most often due to a pituitary adenoma. The diagnosis is often made late as the symptoms are insidious in onset. This patient is putting on weight, has prognathism, large hands and feet and glycosuria. There may also be an enlarged nose, soft tissue changes and organomegaly, all as a result of excess GH/IGF-1. Visual impairment may be present is due to the pituitary adenoma putting pressure on the optic chiasm. Tumour mass effect may also cause headaches and there may also be hypopituitarism from stalk compression and CN palsies. IGT and DM are associated and this may result not only in glycosuria but possible polydipsia and polyuria. There may also be altered sexual function, Carpal Tunnel Syndrome and joint dysfunction.
Treatment is either sugical with a transsphenoidal approach, or medical (if the tumour cannot be resected/completely resected) with a somatostatin analogue like octreotide and an adjunctive dopamine agonist like cabergoline. If the patient does not respond to SSAs then pegvisomant which is a GH receptor antagonist can be used, although it is very costly. Gigantism occurs as a result of excess GH during childhood.

337
Q

A 41 year old woman with a history of neurofibromatosis has erratic BP readings. Some readings are as high as 220/120 where as otheres are normal. She comes to you complaining of intermittent headaches, sweating and palpitations.

A. Hypothyroidism
B. Hyperparathyroidism
C. Chronic alcohol excess
D. Essential hypertension
E. ‘White-coat hypertension’
F. Polycystic kidney disease
G. Cushing's syndrome
H. Conn's syndrome
I. Medication
J. Phaeochromocytoma
K. Acromegaly
L. Aortic coarctation
M. Renal artery stenosis
A

Phaeocromocytoma: Phaeochromocytomas presents with paroxysmal episodes of palpitations, anxiety, excessive sweating, pallor and hypertension. It can be inherited in MEN2, von Hippel-Lindau syndrome and NF1. Diagnosis is based on raised urinary and serum catecholamines, metanephrines and normetanephrines. 24 hour urinary VMA will be elevated. CT is used to localise the tumour. Treatment includes medical with the use of phenoxybenzamine, phentolamine and surgical options. Surgical excision is carried out under alpha and beta blockade to protect against the release of catecholamines into circulation when the tumour is being manipulated. The 10% rule is often quoted: 10% are bilateral, 10% malignant, 10% extraadrenal and 10% hereditary.

338
Q

A 10 year old boy presents with stridor. He reports three episodes of face and tongue swelling, each of which prompted him to report to A&E. There are also red, raised and itchy lesions that cover his body, including face. His sister also suffers from similar attacks.

A. Kawasaki disease
B. Acute rheumatic fever
C. Aortic stenosis
D. Hereditary angio-oedema
E. Congestive cardiac failure
F. Toxic synovitis
G. Juvenile idiopathic arthritis
H. Pericarditis
I. Primary pulmonary hypertension
J. Congenital nephritic disease
K. Myocarditis
A

Hereditary Angioedema: This patient has urticaria (erythematous, blanching, oedematous, pruritic lesions) and angio-oedema (swelling). A positive family history of angio-oedema raises a suspicion for a diagnosis of hereditary angio-oedema. There are two forms of this condition. One is manifest by absence of C1 esterase inhibitor whereas the other is due to normal levels of dysfunctional C1 esterase inhibitor. This allows the uncontrolled activation of the complement cascade which therefore gives rise to angio-oedema. This is a condition which is inherited in an autosomal dominant manner although it should be noted that some 50% of cases have no previous FH and are thought to be due to new mutations. Laboratory investigations may reveal a decreased level of C1 and decreased levels or function of C1 esterase which would support the diagnosis. In acquired angio-oedema, C1q levels are low unlike in the hereditary form where it is normal – this differentiates the two forms. The mainstay of treatment is with antihistamines. Airway compromise like the stridor this patient is experiencing is an indication for prompt treatment with adrenaline. The stridor here is a sign of severe laryngeal angio-oedema, which is a sign of impending airway obstruction – this needs to be taken seriously and is an emergency.

339
Q

A 6 year old girl presents with stiffness and a limp which has lasted for a few weeks now. The onset is reported as insidious and her parents tell you she has not had any injury or infections. One of her knees is swollen and cannot be straightened. The symptoms are worse in the mornings but improve throughout the day. There is also involvement of the small joints of the hands and feet.

A. Kawasaki disease
B. Acute rheumatic fever
C. Aortic stenosis
D. Hereditary angio-oedema
E. Congestive cardiac failure
F. Toxic synovitis
G. Juvenile idiopathic arthritis
H. Pericarditis
I. Primary pulmonary hypertension
J. Congenital nephritic disease
K. Myocarditis
A

Juvenile idiopathic arthritis: This is juvenile idiopathic arthritis, also known as juvenile rheumatoid arthritis, or Still’s disease. It is the most common chronic arthropathy of children and there are several clinical subtypes. The diagnosis is clinical. Intra-articular steroids offer good control if only a few joints are affected. Methotrexate is also a commonly used disease-modifying agent. More resistant cases are treated with agents which block inflammatory cytokines. Around 10-20% of children with JIA are at risk of developing anterior uveitis and therefore all children with this diagnosis must undergo regular ophthalmological review for inflammation. Remember that symptoms vary according to subtype of disease, which laboratory tests may be useful in classifying. Note also that while this can be called juvenile RA, rheumatoid factor is only positive in a small minority of patients (2-7%).

340
Q

A 12 year old boy presents with polyarthritis and abdominal pain. He had a sore throat about a week ago. Examination reveals an early blowing diastolic murmur at the left sternal edge. Shortly afterwards, there are bilateral involuntary jerky movements worse when the patient is asked to make a movement.

A. Kawasaki disease
B. Acute rheumatic fever
C. Aortic stenosis
D. Hereditary angio-oedema
E. Congestive cardiac failure
F. Toxic synovitis
G. Juvenile idiopathic arthritis
H. Pericarditis
I. Primary pulmonary hypertension
J. Congenital nephritic disease
K. Myocarditis
A

Acute Rheumatic fever: Chorea features as part of the acute presentation in 5-10% of patients with rheumatic fever. It can also occur as an isolated event up to 6 months after the initial GABHS infection. It is named Sydenham chorea after the doctor who described St Vitus Dance in the 17th century. Choreiform movements can affect the whole body or just one side of the body, in which case it is referred to as hemi-chorea. The head is often involved with erratic facial movements that resemble grimaces, grins and growns, and the tongue may be affected to resemble a bag of worms when protruded, and protrusion cannot be maintained. In severe cases the patient may have an impaired ability to eat. Chorea disappears with sleep and is made worse by purposeful movements. When the patient is asked to grip the doctor’s hand, the patient will be unable to maintain grip and rhythmic squeezing occurs. There are two signs to look out for in these patients. The first is the spooning sign, which is a flexion at the wrist with finger extension when the hand is held extended. The pronator sign is the second which is when the palms turn outwards when held above the head. Both are consistent with chorea.
Remember that the 5 major manifestations of acute rheumatic fever are carditis, polyarthritis, chorea, erythema marginatum and SC nodules – the most common of which are carditis and polyarthritis. The murmur here is a manifestation of carditis. Primary episodes occur mainly in children aged 5-14 and are rare in those over 30. The greatest burden of disease remains in the developing countries and in populations of people living in poverty.

341
Q

A 16 year old boy presents with 5 month history of chest pain on exertion and two episodes of collapse in the last month. There is also progressive SOB on exertion and now he cannot walk up the stairs without stopping. Examination reveals a loud systolic murmur.

A. Kawasaki disease
B. Acute rheumatic fever
C. Aortic stenosis
D. Hereditary angio-oedema
E. Congestive cardiac failure
F. Toxic synovitis
G. Juvenile idiopathic arthritis
H. Pericarditis
I. Primary pulmonary hypertension
J. Congenital nephritic disease
K. Myocarditis
A

Aortic Stenosis: Aortic stenosis can present with chest pain, dyspnoea and syncope. It is characterised by a harsh ejection systolic murmur heard loudest at the right upper sternal edge at end expiration, which radiates up towards the carotids. The pulse pressure is narrow and there may be an associated slow-rising and plateau pulse. Doppler echo is vital for diagnosis and shows a pressure gradient across the narrowed valve orifice. This is congenital aortic stenosis due to an abnormally formed aortic valve. He may here be considered for surgical repair or TAVR.

342
Q

A 13 year old girl presents with increasing SOB, particularly when lying down at night to try to sleep. She has also noticed some ankle swelling. Examination reveals a raised JVP, tachycardia and an S3 gallop rhythm on cardiac auscultation.

A. Kawasaki disease
B. Acute rheumatic fever
C. Aortic stenosis
D. Hereditary angio-oedema
E. Congestive cardiac failure
F. Toxic synovitis
G. Juvenile idiopathic arthritis
H. Pericarditis
I. Primary pulmonary hypertension
J. Congenital nephritic disease
K. Myocarditis
A

CCF: The signs and symptoms this patient has points to CCF (congestive cardiac failure). SOB with orthopnoea due to the sudden increase in pre-load, indicates LV failure. Neck vein distension is also present, which is a major Framingham criteria for diagnosis. Tachycardia and ankle oedema are both minor criteria for diagnosis. Other major criteria for diagnosis include S3 gallop, cardiomegaly and hepatojugular reflux. For all patients, initial investigations should include ECG, CXR, TTE and bloods including BNP levels.

CXR may reveal pulmonary vascular redistribution to the upper zones, Kerley B lines, an increased CTR (cardiomegaly) and pleural effusion. CCF in children occurs as a result of various congenital abnormalities as well as rheumatic fever. Congenital causes include aortic stenosis, PDA and Eisenmenger’s syndrome.

343
Q

A 70-year-old diabetic male presents with severe pain in his left foot. The pain is present at rest and is alleviated by hanging his leg off the foot of the bed at the night. On examination you note advanced gangrene with superimposed infection of the left foot with absent dorsalis pedis and posterior tibial pulses.

A. Fasciotomy
B. Embolectomy
C. Femoral-femoral crossover graft
D. Femoro-popliteal bypass
E. Conservative management
F. Endarterectomy
G. Amputation
H. Anticoagulation
I. Percutaneous transluminal angioplasty
J. Antiplatelet drug
K. Thrombolysis
L. Aorto-bifemoral bypass
A

Amputation: Gangrene occurs as a complication of necrosis and characterised by the decay of body tissue. It can be due to ischaemia, trauma or infection, or a combination of these processes. There is ischaemic gangrene, which arises due to either arterial or venous obstruction. There is also infectious gangrene which include processes like gas gangrene cause by Clostridium perfringens and necrotising fasciitis which has many causes, commonly Streptococcus pyogenes. Diabetes is a risk factor here and is frequently associated with both infectious and ischaemic gangrene. High blood glucose and impaired immunity, peripheral neuropathy and arterial disease contribute to limb-threatening diabetic foot infections. The absent pulses and symptoms this patient is experiencing here (critical limb ischaemia) also suggests diabetic chronic peripheral arterial disease, which in diabetics tends to affect smaller arteries and affects a younger age group when compared to non-diabetics.There is advanced gangrene here, and in cases of severe limb sepsis, amputation is required. This is a two stage process which at first involves guillotine amputation and later, when the infection has cleared, a definitive amputation and wound closure is needed. If the extremity is not viable (such as a large amount of necrosis, profound anaesthesia/paralysis or an inaudible Doppler pulse) then the patient should undergo prompt amputation. Every effort should be made to preserve as many joints as possible in order to improve rehabilitation chances and to decrease the work of walking around with a prosthesis.

344
Q

A 55-year-old obese smoker presents with pain in his legs on walking 800 metres, which is immediately relieved by rest. His ankle-brachial pressure index is 0.9.

A. Fasciotomy
B. Embolectomy
C. Femoral-femoral crossover graft
D. Femoro-popliteal bypass
E. Conservative management
F. Endarterectomy
G. Amputation
H. Anticoagulation
I. Percutaneous transluminal angioplasty
J. Antiplatelet drug
K. Thrombolysis
L. Aorto-bifemoral bypass
A

Conservative management: This is peripheral vascular disease with classic symptoms of claudication (in reality, these classic symptoms only occur in a small minority of patients). ABPI should be performed in symptomatic patients and a result less than or equal to 0.9 is diagnostic for the presence of peripheral vascular disease. You need to however bear in mind that this test may not be accurate if the patient has non-compressible arteries (mainly in diabetic patients). This patient has only presented with claudication which is not severely lifestyle limiting. It depends on how much needing to rest every 800 metres or so bothers him. If he does not feel that this is really a functional disability then no additional treatment is required, but follow-up appointments with a doctor should be made to monitor the development of ischaemic symptoms or coronary and cerebrovascular complaints.

If the symptoms are lifestyle limiting then the patient should undergo a supervised exercise programme (only some rather limited quality cohort studies at the moment show an improvement in walking time and symptoms) and medication for symptomatic relief for a period of 3 months. Medication can include cilostazol, pentoxifylline (widely used but no more effective than placebo in RCTs) or naftidrofuryl. Risk factors should also continually be targetted – BP control, statins to lower LDL, beta blockers to target cardiovascular risk and antiplatelet therapy, for instance. If no improvement is made with this regime then patients should be referred to a vascular specialist to have their anatomy defined and assessed for possible revascularisation.

345
Q

A 65-year-old female presents with sudden-onset pain in her left calf. Although her patient notes are unavailable, she tells you that she is taking digoxin and verapamil for her ‘funny’ heart beat. On examination, the left leg is pale, cold and painful.

A. Fasciotomy
B. Embolectomy
C. Femoral-femoral crossover graft
D. Femoro-popliteal bypass
E. Conservative management
F. Endarterectomy
G. Amputation
H. Anticoagulation
I. Percutaneous transluminal angioplasty
J. Antiplatelet drug
K. Thrombolysis
L. Aorto-bifemoral bypass
A

Embolectomy: Have a think about the differential diagnosis of sudden onset limb pain. Do you remember the 6 Ps of critical limb ischaemia? This patient’s arrhythmia has caused an embolic event, leading to acute limb ischaemia. There is as a result a sudden decrease in limb perfusion with threatened tissue viability. An emergency vascular assessment needs to be done with duplex ultrasound. Treatment depends on whether the patient already has a history of significant atherosclerosis. If so, there will already be a built up collateral supply so there is potentially a longer time window to act and so anticoagulation and thrombolysis are options. Otherwise an embolectomy will be indicated with a Fogarty catheter if there is not a long enough time window. This is typically done by inserting a Fogarty catheter with an inflatable balloon attached to its tip into the offending atery and passing the tip beyond the clot. The balloon is then inflated and then the catheter is withdrawn to remove the clot.

346
Q

A 63-year-old male with a history of AF underwent an embolectomy a few hours ago after a clot was found in the popliteal artery. He is now complaining of increasing pain and tightness in the treated leg. O/E the leg appears swollen and there is pain on passive flexion of the foot.

A. Fasciotomy
B. Embolectomy
C. Femoral-femoral crossover graft
D. Femoro-popliteal bypass
E. Conservative management
F. Endarterectomy
G. Amputation
H. Anticoagulation
I. Percutaneous transluminal angioplasty
J. Antiplatelet drug
K. Thrombolysis
L. Aorto-bifemoral bypass
A

Fasciotomy: This patient has developed compartment syndrome most likely as a result of soft tissue injury or direct injury to the musculature following the recent embolectomy. Additional causes include fractures and compartment haemorrhage. This condition results from raised interstitial pressure in closed osseofascial compartments. The classical clinical diagnosis will be of the following 6 Ps: pain, pressure, pulselessness, paralysis, paraesthesia and pallor (uncommon). The history here of severe extremity pain and tightness following documented trauma is classical. The pain tends to be out of proportion to the injury and is made worse by passive stretching of the muscle groups which are contained by the affected compartment. Passive stretching of the muscles of the compartment which is involved will also elicit pain. Note that true paralysis is a late sign, as is loss of pulses and pallor. Paraesthesia is however an early seen sign. If the diagnosis is uncertain in an at risk patient then compartment pressure measurement is indicated. Measurements of serum CK and urine myoglobin will also be indicated and these may be elevated with muscle cell lysis and necrosis. This is not due to an occlusive dressing (if it were, the first line treatment would be to remove this dressing). Therefore, a fasciotomy is indicated regardless of time of diagnosis with fasciotomy of all compartments with elevated pressure. There is a clear 6 hour window whereby there are lower amputation and death rates compared to delays >6 hours, so this needs to be done as a matter of urgency. The incision needs to be long enough too! Wound care post-fascitomy is important to prevent the risk of secondary infection and to debride any necrotic tissue, or to consider skin grafts. Post-operatively, care will be MDT with physical and occupational therapies and a range of motion exercises to try an d get the patient fully functional.

347
Q

A 72-year-old male complains of right leg pain on walking 50 metres. Angiography reveals significant stenosis of the superficial femoral artery.

A. Fasciotomy
B. Embolectomy
C. Femoral-femoral crossover graft
D. Femoro-popliteal bypass
E. Conservative management
F. Endarterectomy
G. Amputation
H. Anticoagulation
I. Percutaneous transluminal angioplasty
J. Antiplatelet drug
K. Thrombolysis
L. Aorto-bifemoral bypass
A

Femoral-poplital bypass: Broadly speaking, if there is disease with less severe stenosis then endovascular revascularisation is the recommended approach. Surgical revascularisation is recommended if there is more severe stenosis. At this stage, there is no need to get bogged down by the exact recommendations. This significant stenosis will likely require some form of revascularisation. The best option on the list for disease occuring only on one side and localised to the SFA is a femoro-popliteal bypass (colloquially referred to as a fem-pop). Usually the patient’s own long saphenous vein is used as a graft. As you may remember from year 2 anatomy, the SFA becomes the popliteal artery at the popliteal fossa. As a brief summary, femoro-femoral cross-over grafts are done for cases of unilateral iliac artery occlusion. Aorto-bifemoral bypass is used for atherosclerosis of the infrarenal aorta and iliacs.

348
Q

A 62-year-old lorry driver presents with sudden-onset weakness of the right side of his body as well as ipsilateral loss of vision on the left, which he describes as like a ‘curtain’ descending over his field of vision. His symptoms resolve completely a few minutes later.

Duplex doppler ultrasound
CT scan
Ankle-brachial pressure index
No investigation necessary
Venography
Contrast angiography
Magnetic resonance venography
ESR
Coagulation studies
Brain MRI
Blood glucose level
EMG walking test
Serum CK
A

Duplex doppler ultrasound: This man is presenting with classic features of a TIA. A TIA is colloquially called a ‘mini stroke’ with symptoms typically lasting under an hour (and resolve within 24 hours). An antiplatelet drug such as aspirin is effective secondary prevention if the patient is not already anticoagulated. The patient will be anticoagulated if they have a likely or known cardioembolic source such as AF. Clopidogrel is an alternative in those who do not tolerate aspirin.
The description of transient visual disturbance like a curtain descending over the eye is characteristic of amaurosis fugax. Amaurosis fugax is a transient and painless loss of vision in one eye due to the passage of an embolus into the central retinal artery. The cause could be embolic from the internal carotid artery to cause an occlusion of the ipsilateral retinal artery. Patients presenting with TIAs should be investigated for carotid artery stenosis with a carotid Doppler ultrasound as there is a high risk of having a subsequent full blown stroke. Furthermore if there is a stenosis of >70%, the patient may be a candidate for carotid endarterectomy. Presence of ipsilateral carotid stenosis suggests artery-to-artery embolic event as the cause and this should be the target for surgical or interventional treatment. Follow up tests could be CT angiography or MRA to expand on the abnormal Doppler results. They are not appropriate first line investigations to do here. Head CT is usually normal in TIA. ECG should also be done to investigate for AF which is a common risk factor for embolic cerebral ischaemia.

349
Q

A 56-year-old man is scheduled for elective AAA repair. The extent and its relationship to the renal arteries need to be identified.

Duplex doppler ultrasound
CT scan
Ankle-brachial pressure index
No investigation necessary
Venography
Contrast angiography
Magnetic resonance venography
ESR
Coagulation studies
Brain MRI
Blood glucose level
EMG walking test
Serum CK
A

CT scan: Generally, elective surgical repair is indicated in patients with large symptomatic AAA – repair of aneuryms over 5.5cm offers a survival advantage. Also, young healthy patients and women in particular may benefit from early repair for smaller AAAs. Data suggests that EVAR (endovascular AAA repair) is equivalent to open repair in terms of overall survival but the rate of secondary interventions is higher with EVAR. Generally, those with a greater risk of perioperative morbidity and mortality, such as patients with co-morbidities such as COPD, may benefit from the less invasive approach (anatomy permitting). Younger and healthier patients may benefit from the relative durability of traditional open repair. A CT scan is useful for diagnosis aortic aneurysms which lie close to the origins or or proximal to the renal arteries. While abdominal ultrasound can also identify the AAA and aortic dilation, a CT scan is more useful in localising this lesion and its relationship to the renal vasculature.

350
Q

A 55 year-old man presents with cramping pains in his left leg that occur after walking 100 metres. The pain is effectively relieved by rest

Duplex doppler ultrasound
CT scan
Ankle-brachial pressure index
No investigation necessary
Venography
Contrast angiography
Magnetic resonance venography
ESR
Coagulation studies
Brain MRI
Blood glucose level
EMG walking test
Serum CK
A

ABPI: This is peripheral vascular disease (claudication) and the first investigation to do here is an ABPI. This is an ankle brachial pressure index and has a sensitivity of 95% and a specificity of 100%. It is however important to remember that it may not be accurate in patients who have non-compressible arteries – so beware, particularly in diabetics. Those with either severely stenotic or totally occluded arteries may have a normal ABPI if there is abundant collateral circulation. ABPI of les s than or equal to 0.9 is diagnostic for the presence of peripheral vascular disease. ABPI is performed by taking the systolic pressure of the left and right brachial arteries and the left and right PT and DP arteries pressure. The ABPI is the highest of the DP and PT pressure divided by the higher of the left and right arm brachial artery pressure. Finding the artery with the probe is a skill which may take a bit of practice so have a play around with the probe if you are stuck on a vascular attachment. ABPI is a marker of peripheral atherosclerosis as well as a predictor of vascular events. Risk factors for PVD include smoking, diabetes, old age, hyperlipidaemia and history of coronary or cerebrovascular disease.

351
Q

A 65-year-old man is brought into A&E after his son witnessed him collapse in his home. He reveals how his father complained of an excruciating pain in his lower back. On examination, the patient is pale and cold with shut-down perpheries. There is a palpable epigastric mass.

Duplex doppler ultrasound
CT scan
Ankle-brachial pressure index
No investigation necessary
Venography
Contrast angiography
Magnetic resonance venography
ESR
Coagulation studies
Brain MRI
Blood glucose level
EMG walking test
Serum CK
A

No investigation necessary: This is a history of a ruptured AAA. There is back pain here and shut down peripheries and pallor due to blood loss. This patient is in haemorrhagic shock. As this AAA has ruptured, this man will need urgent surgical repair, with of course standard resuscitation measures. Investigations would just waste time. The airway will needed to be managed with supplemental oxygen and ET intubation, a central venous catheter will need to be inserted, an arterial catheter and urinary catheter will also be needed for monitoring, and the target systolic BP is 50-70. Infusing too many fluids may increase the risk of death. The most effective form of surgical repair is an EVAR (endovascular AAA repair), anatomy permitting, otherwise traditional open repair is performed. Open repair has a mortality of 48%. Antibiotics will also be needed to cover bacteria to prevent graft infection. This will be prescribed in line with local protocols.

352
Q

A 39-year-old multiparous woman presents to the clinic with varciosities in both legs. Although asymptomatic, she wishes to undergo surgery to remove them as they are causing her great embarassment. In order to determine the best treatment plan, the surgeon would like to map out all the incompetent venous pathways

Duplex doppler ultrasound
CT scan
Ankle-brachial pressure index
No investigation necessary
Venography
Contrast angiography
Magnetic resonance venography
ESR
Coagulation studies
Brain MRI
Blood glucose level
EMG walking test
Serum CK
A

Doppler Ultrasound: Ablative procedures include stripping and ligation, the aim of which is to permanently remove the varicose vein. Radiofrequency ablation (RFA) can also be done, as well as endovenous laser therapy and foamed sclerotherapy. Phlebectomy or sclerotherapy can also be performed. This is generally reserved for symptomatic cases, although this woman has a cosmetic issue with the appearance of her legs which is causing her distress. There are complications of ablation which the patient will need to be made aware of though, such as bleeding, infection, saphenous nerve injury and neovascularisation.
A duplex ultrasound is the investigation which is required here. It can assess reversed flow and valve closure time. This should be done with the patient standing and with the leg in external rotation for best sensitivity. Specific segments which are affected by reflux can be delineated as the superficial and deep truncal veins, perforators and tributaries can all be visualised. Reflex in the great saphenous or common femoral can be detected with Valsalva while more distal reflux can be elicitied by compressing the leg above the Doppler probe to see if any blood is being forced back towards the feet.

353
Q

A 50-year-old diabetic lady, who has smoked 40 cigarettes a day for the last 30 years, presents with a year’s history of worsening bilateral calf pain when she walks. The pain goes away when she stops walking but recurs when she resumes. She has been started recently on hormone replacement therapy.

A. Dead limb
B. Rhabdomyolysis
C. Leriche syndrome
D. Acute limb ischaemia
E. Viable limb
F. Baker’s cyst
G. Ankylosing spondylitis
H. Critical limb ischaemia
I. Intermittent claudication
J. Compartment syndrome
K. Deep vein thrombosis
L. Spinal stenosis
M. Muscle tear
A

Intermittent Claudication: This is peripheral vascular disease with classic symptoms of claudication (in reality, these classic symptoms only occur in a small minority of patients). Remember also that intermittent claudication can also occur in the large muscle groups of the upper leg, which is indicative of narrowing of the deep femoral artery. ABPI should be performed in symptomatic patients and a result less than or equal to 0.9 is diagnostic for the presence of peripheral vascular disease. You need to however bear in mind that this test may not be accurate if the patient has non-compressible arteries (mainly in diabetic patients like this one). If she does not feel that this claudication is really a functional disability then no additional treatment is required, but follow-up appointments with a doctor should be made to monitor the development of ischaemic symptoms or coronary and cerebrovascular complaints.
If the symptoms are lifestyle limiting then the patient should undergo a supervised exercise programme (only some rather limited quality cohort studies at the moment show an improvement in walking time and symptoms) and medication for symptomatic relief for a period of 3 months. Medication can include cilostazol, pentoxifylline (widely used but no more effective than placebo in RCTs) or naftidrofuryl. Risk factors should also continually be targetted – BP control, statins to lower LDL, beta blockers to target cardiovascular risk and antiplatelet therapy, for instance. If no improvement is made with this regime then patients should be referred to a vascular specialist to have their anatomy defined and assessed for possible revascularisation.

354
Q

A 40-year-old man has been brought into A&E after a road traffic accident. He was a driver and involved in a head-on collision with another vehicle. It took the fire brigade 45 minutes to cut him out of the wreckage. This patient sustained a fracture of his right leg and mentions to you a few hours later that his leg feels hard. When you examine him, there is pain on passive flexion of the ankle.

A. Dead limb
B. Rhabdomyolysis
C. Leriche syndrome
D. Acute limb ischaemia
E. Viable limb
F. Baker’s cyst
G. Ankylosing spondylitis
H. Critical limb ischaemia
I. Intermittent claudication
J. Compartment syndrome
K. Deep vein thrombosis
L. Spinal stenosis
M. Muscle tear
A

Compartment Syndrome: This patient has developed compartment syndrome due to trauma and the fracture. This condition results from raised interstitial pressure in closed osseofascial compartments. The classical clinical diagnosis will be of the following 6 Ps: pain, pressure, pulselessness, paralysis, paraesthesia and pallor (uncommon). The history tends to be of severe extremity pain and tightness following documented trauma. The pain tends to be out of proportion to the injury and is made worse by passive stretching of the muscle groups which are contained by the affected compartment. Passive stretching of the muscles of the compartment which is involved will also elicit pain. Note that true paralysis is a late sign, as is loss of pulses and pallor. Paraesthesia is however an early seen sign. If the diagnosis is uncertain in an at risk patient then compartment pressure measurement is indicated. Measurements of serum CK and urine myoglobin will also be indicated and these may be elevated with muscle cell lysis and necrosis.
This is not due to an occlusive dressing (if it were, the first line treatment would be to remove this dressing). Therefore, a fasciotomy is indicated regardless of time of diagnosis with fasciotomy of all compartments with elevated pressure. There is a clear 6 hour window whereby there are lower amputation and death rates compared to delays >6 hours, so this needs to be done as a matter of urgency. The incision needs to be long enough too! Wound care post-fascitomy is important to prevent the risk of secondary infection and to debride any necrotic tissue, or to consider skin grafts. Post-operatively, care will be MDT with physical and occupational therapies and a range of motion exercises to try an d get the patient fully functional.

355
Q

A 45-year-old Type 1 diabetic lady is brought in unconscious. She has been ill recently and neglected to take her insulin. On secondary survey, you notice that the right leg is wrapped up in bandages. When these are removed, there is evidence of gangrene. In particular, there is fixed mottling of the skin up to the mid-shin level.

A. Dead limb
B. Rhabdomyolysis
C. Leriche syndrome
D. Acute limb ischaemia
E. Viable limb
F. Baker’s cyst
G. Ankylosing spondylitis
H. Critical limb ischaemia
I. Intermittent claudication
J. Compartment syndrome
K. Deep vein thrombosis
L. Spinal stenosis
M. Muscle tear
A

Dead Limb: This gangrene may well have mixed infectious and ischaemic aetiology – both of which have associations with DM. You will also hear infectious gangrene being referred to as ‘wet gangrene’ and ischaemic gangrene being referred to as ‘dry gangrene’. Atherosclerosis is associated with DM and forms an important cause of dry gangrene. This occurs due to chronic impairment of blood flow and typically presents with tissue which is dry, black and cold – in most cases, self-amputation eventually occurs. Wet gangrene includes causes such as necrotising fasciitis and gas gangrene. Evidence of non-viability and irreversibility of the affected limb include major tissue loss, sensory loss with rest pain, inaudible arterial Doppler signals and muscle weakness. Fixed mottling does not blanch on pressure and is frequently associated with a limb which is beyond salvage.

356
Q

You see a 75-year-old gentleman in clinic who has been followed up for his foot ulcers for over 10 years. He tells you during the consultation that he has recently had problems sleeping at night with leg pain waking him up. He notices that it helps if he hangs his foot over the edge of the bed.

A. Dead limb
B. Rhabdomyolysis
C. Leriche syndrome
D. Acute limb ischaemia
E. Viable limb
F. Baker’s cyst
G. Ankylosing spondylitis
H. Critical limb ischaemia
I. Intermittent claudication
J. Compartment syndrome
K. Deep vein thrombosis
L. Spinal stenosis
M. Muscle tear
A

Critical Limb Ischaemia: Is this patient’s ulcer an arterial ulcer with diabetic neuropathy, or a diabetic ulcer? Difficult to tell. However, this patient does have critical limb ischaemia. Critical limb ischaemia is chronic severe limb ischaemia. This patient has rest pain which is relieved by hanging the leg dependent – this is classical. These patients have chronic ischaemic symptoms of the leg such as ischaemic rest pain, gangrene and non-healing wounds. Ischaemic cause of these symptoms needs to be established urgently, or if the patient already has a documented history of PVD, then they can be referred to a vascular surgeon for revascularisation. Their arterial anatomy needs to be defined and assessed. Risk factors also need to continue to be aggressively targetted. If they are not a candidate for revascularisation then the patient should be assessed for amputation where necessary and be on appropriate risk factor reduction medication.

357
Q

A 65-year-old lady is referred to your vascular clinic by a GP. The patient has had a painful left leg for the last week. You find out that she smokes and is diabetic. Her symptoms started following a week’s illness when she was bedbound. You request an ultrasound which confirms your diagnosis.

A. Dead limb
B. Rhabdomyolysis
C. Leriche syndrome
D. Acute limb ischaemia
E. Viable limb
F. Baker’s cyst
G. Ankylosing spondylitis
H. Critical limb ischaemia
I. Intermittent claudication
J. Compartment syndrome
K. Deep vein thrombosis
L. Spinal stenosis
M. Muscle tear
A

DVT: This patient has a DVT. There is a history of smoking, DM and immobility. Other risk factors include recent major surgery, active malignancy, pregnancy and malignancy. A Wells score is determined in all patients with a suspected DVT with the condition being likely if the score is 2 or greater. The most definitive test is venography but it is invasive. Compression USS of the proximal deep venous system is preferred but Doppler venous flow testing can be used if other tests are unavailable and will demonstrate low flow in affected veins. This requires a trained technician. Anticoagulation is the mainstay of treatment with unfractionated heparin, a LMWH or an anti FXa agent such as fondaparinux.

358
Q

A 40-year old gentleman with a history of diabetes mellitus and CHD presents to your clinic complaining of pain in his buttocks after walking only 100 meters. He has recently married, and with much embarassment, reveals that he has had difficulty completing intercourse as he is unable to maintain an erection. O/E you notice his femoral pulses are absent.

A. Dead limb
B. Rhabdomyolysis
C. Leriche syndrome
D. Acute limb ischaemia
E. Viable limb
F. Baker’s cyst
G. Ankylosing spondylitis
H. Critical limb ischaemia
I. Intermittent claudication
J. Compartment syndrome
K. Deep vein thrombosis
L. Spinal stenosis
M. Muscle tear
A

Leriche Syndrome: Leriche’s syndrome, named after some French surgeon, is aortoiliac atherosclerotic occlusive disease and is one of those eponymous syndromes which can be found in your Oxford Handbook. This involves the abdominal aorta and/or both of the iliac arteries. It presents with the classic triad of buttock claudication, impotence and reduced or absent femoral pulses. This combination is referred to as Leriche’s syndrome.

359
Q

A 60-year old lady who has just from a STEMI requires ongoing treatment. The doctor asks you what would be appropriate first line prevention therapy.

A. Alteplase
B. Warfarin and heparin
C. Beta blocker
D. Warfarin
E. Simvastatin
F. ACE inhibitor
G. Glyceryl trinitrate
H. Clopidogrel and atorvastatin
I. Aspirin and clopidogrel
J. Dipyridamole
K. Aspirin and atorvastatin
L. Aspirin and lisinopril
A

Aspirin and clopidogrel: Dual antiplatelet therapy is recommended for at least 12 months in all patients whether or not they have been stented. Aspirin should be continued forever and clopidogrel for at least a year. Note that prasugrel, a platelet inhibitor, has been found in relatively recent studies to be superior to clopidogrel in outcome measures when given for at least a year – although there is an increased risk of bleeding in low weight patients and in those over 74 years of age (lower doses are recommended in these patients).

360
Q

What else can be given to this same patient post-MI which also has a favourable effect on ventricular remodelling?

A. Alteplase
B. Warfarin and heparin
C. Beta blocker
D. Warfarin
E. Simvastatin
F. ACE inhibitor
G. Glyceryl trinitrate
H. Clopidogrel and atorvastatin
I. Aspirin and clopidogrel
J. Dipyridamole
K. Aspirin and atorvastatin
L. Aspirin and lisinopril
A

ACE inhibitor: ACE inhibitors should be started early (when the patient is haemodynamically stable, optimally on the first day in hospital) for a favourable effect on ventricular remodelling, particularly for patients with a large anterior wall MI. There is good evidence to suggest that ACE inhibitors are more effective at reducing overall mortality and sudden cardiac death after 2-42 months compared to placebo, especially when started within 14 days of an acute MI.

361
Q

An 80-year-old lady who is prone to consuming fatty foods has had a stroke. She is allergic to aspirin. She asks you what she can take to prevent her having another stroke.

A. Alteplase
B. Warfarin and heparin
C. Beta blocker
D. Warfarin
E. Simvastatin
F. ACE inhibitor
G. Glyceryl trinitrate
H. Clopidogrel and atorvastatin
I. Aspirin and clopidogrel
J. Dipyridamole
K. Aspirin and atorvastatin
L. Aspirin and lisinopril
A

Clopidogrel and atorvastatin: Remember that in acute stroke, there is no current evidence that non-aspirin antiplatelet drugs such as dipyridamole, clopidogrel and glycoprotein IIb/IIIa inhibitors are effective so they are not indicated. However, clopidogrel is useful in the secondary prevention of stroke and is indicated in those who are sensitive to aspirin. A statin is also given here for its lipid lowering effects to lower the risk of stroke and other cardiovascular events. In terms of antiplatelet drugs for secondary prevention, a systematic review of the studies have found that the most cost-effective treatment is clopidogrel, followed by MRD (modified release dipyridamole) followed lastly by aspirin. Incidentally, in terms of cost-effectiveness, clopidogrel should be the first choice agent anyway.

362
Q

A 55 year old needs secondary prevention for stroke. He has had one stroke already which is due to cardioembolism. He has atrial fibrillation.

A. Alteplase
B. Warfarin and heparin
C. Beta blocker
D. Warfarin
E. Simvastatin
F. ACE inhibitor
G. Glyceryl trinitrate
H. Clopidogrel and atorvastatin
I. Aspirin and clopidogrel
J. Dipyridamole
K. Aspirin and atorvastatin
L. Aspirin and lisinopril
A

Warfarin: Anticoagulation with warfarin is indicated for atrial fibrillation in the secondary prevention of stroke, with target INR of 2.5 (range 2-3). This is for when the stroke (or indeed TIA) is following a cardioembolic source like in this case. If the stroke is not cardioembolic then these drugs may not confer any long term benefit and will increase the patient’s risk of serious bleeding. Those who are not appropriate candidates for warfarin should have clopidogrel added to aspirin to reduce the risk of stroke. Note that I have not included ‘Aspirin and warfarin’ as an option because this can be given in some cases, such as after multiple recurrent strokes caused by AF and co-existing coronary artery disease, but is controversial.

363
Q

A 62-year-old man has had an ischaemic stroke. He has normal blood pressure and is in sinus rhythm, although is noted to have peripheral vascular disease. You prescribe him some preventative medication.

A. Alteplase
B. Warfarin and heparin
C. Beta blocker
D. Warfarin
E. Simvastatin
F. ACE inhibitor
G. Glyceryl trinitrate
H. Clopidogrel and atorvastatin
I. Aspirin and clopidogrel
J. Dipyridamole
K. Aspirin and atorvastatin
L. Aspirin and lisinopril
A

Aspirin and atorvostatin: I will accept ‘Clopidogrel and atorvastatin’ too for this question, as clopidogrel is the most cost effective antiplatelet drug for the secondary prevention of ischaemic stroke or TIA. However, aspirin is what is generally given. The statin, as mentioned before, is given for its lipid lowering effects. It is mainly given for patients with LDL >2.6 and is recommended for those with atherosclerotic ischaemic stroke or TIA, to lower the risk. This patient has PVD which is due to atherosclerosis and suggests that statins will help. Aspirin and clopidogrel would not be given together in this patient as this would unnecessarily increase the risk of a major bleed.

364
Q

A 65-year-old lady presents with back pain. She has had it for about 3 days. Examination is normal but in her blood tests, ALP is elevated.

A. Volvulus 
B. Mesenteric infarction
C. Myocardial infarction
D. Renal colic
E. Hepatitis
F. Metastatic disease
G. Perforated duodenal ulcer
H. Dissecting aortic aneurysm
I. Pancreatitis
J. Pyelonephritis
K. Spinal stenosis
L. Ruptured abdominal aortic aneurysm
M. Addison’s disease
N. Pericarditis
A

Metastatic Disease: Back pain and a raised ALP is an ominous finding which is usually indicative of bone metastases. Alkaline phosphatase is an enzyme which is also a marker for bone turnover. There is often a history of maligancy before this back pain and commonly implicated malignancies include breast, lung, prostate, thyroid and kidney cancer. Neurological deficits may occur if the tumour destruction is extensive and causes compression of nerves. The patient may also have generalised systemic symptoms on examination including fever, chills, weight loss and focal tenderness. XR may demonstrate lysis of the vertebral body and MRI may show up a lytic or blastic lesion.

365
Q

A 70-year-old man with rheumatoid arthritis presents with severe abdominal pain. In A&E he is clearly unwell and hypotensive. He reveals that he has had black tarry stools intermittently for many years.

A. Volvulus 
B. Mesenteric infarction
C. Myocardial infarction
D. Renal colic
E. Hepatitis
F. Metastatic disease
G. Perforated duodenal ulcer
H. Dissecting aortic aneurysm
I. Pancreatitis
J. Pyelonephritis
K. Spinal stenosis
L. Ruptured abdominal aortic aneurysm
M. Addison’s disease
N. Pericarditis
A

Perforated Duodenal Ulcer: Long standing RA leads you towards high NSAID use. NSAIDs, along with H pylori infection are a major risk factors in peptic ulcer disease. The patient’s advanced age adds to this risk. This accounts for black tarry stools over many years which result from blood loss from the ulcer. The acute symptoms now of severe abdominal pain and hypotension is likely to be due to acute blood loss from a perforated ulcer. This patient needs emergency surgery. NSAID induced ulcers are more commonly gastric than duodenal. Stopping NSAID use will reduce ulcer recurrence. If NSAIDs cannot be discontinued, then prescribing a PPI alongside will reduce recurrence.

366
Q

An 80-year-old woman is brought into A&E complaining of diffuse abdominal pain and vomiting. O/E she has an irregularly irregular pulse of 110/min. Minutes later she appears confused and you notice bright red blood passing per rectum.

A. Volvulus 
B. Mesenteric infarction
C. Myocardial infarction
D. Renal colic
E. Hepatitis
F. Metastatic disease
G. Perforated duodenal ulcer
H. Dissecting aortic aneurysm
I. Pancreatitis
J. Pyelonephritis
K. Spinal stenosis
L. Ruptured abdominal aortic aneurysm
M. Addison’s disease
N. Pericarditis
A

Mesenteric Infarction: The irregularly irregular pulse is a hallmark sign of atrial fibrillation which has led to cardioembolism and subsequent occlusion of the mesenteric vasculature. Untreated AF can lead to a thrombus forming inside the heart which can then embolise like this case to the mesenteric vasculature. This person is also old, which is an additional risk due to comorbidities like atherosclerosis. This patient has the symptoms and signs of ischaemic bowel disease (which encompasses acute mesenteric ischaemia, chronic mesenteric ischaemia and colonic ischaemia). This is likely acute mesenteric ischaemia – something that in a person who presents like this, you should maintain a high index of suspicion for as the presentation can be quite non-specific but the condition can be deadly. You would likely in this case opt for surgical intervention without delay although you can consider some form of imaging first to localise the bleed.

367
Q

A 62 year old woman presents to A&E with searing back pain, nausea and vomiting. She is known to have an abdominal aortic aneurysm which is scanned every year. She appears jaundiced and the subsequent abdominal CT reveals an AAA of 5.0cm

A. Volvulus 
B. Mesenteric infarction
C. Myocardial infarction
D. Renal colic
E. Hepatitis
F. Metastatic disease
G. Perforated duodenal ulcer
H. Dissecting aortic aneurysm
I. Pancreatitis
J. Pyelonephritis
K. Spinal stenosis
L. Ruptured abdominal aortic aneurysm
M. Addison’s disease
N. Pericarditis
A

Pancreatitis: The AAA here has nothing to do with the question, and at 5cm, it is not massively prone at the moment to rupture. However, if the patient is otherwise young, fit and healthy, elective repair can soon be considered (the normal abdominal aorta is 1.5cm diameter, and remember that rupture is more common in females than males). This is acute pancreatitis. Jaundice here is suggestive of gallstone aetiology with obstruction to the common bile duct, though pancreatic oedema can itself cause jaundice. Nausea and vomiting is not uncommon and can occur with agitation and confusion. Complicated haemorrhagic pancreatitis may exhibit Cullen’s sign, Grey-Turner’s sign and Fox’s sign. Make sure you know what these are and you are familiar with the causes of acute pancreatitis (GET SMASHED). Those caused by hypocalcaemia may also display Chvostek’s sign and Trousseau’s sign.
Key to diagnosis is serum amylase or lipase levels which are massively elevated. Prognostic criteria are outlined in Ranson’s criteria applied on admission and after 48 hours, or the modified Glasgow score which you can find in your Oxford Handbook.

368
Q

A 65-year-old man is referred to you for an emergency appointment after attending his GP earlier that morning. You are called by the nurse to the waiting room, where you see this patient looking pale, sweaty and unwell. You establish that he has pain in his chest radiating to his back. You call a crash team. Later on, you find out that his Troponin I was 0.032 and that he was taken to theatre.

A. Volvulus 
B. Mesenteric infarction
C. Myocardial infarction
D. Renal colic
E. Hepatitis
F. Metastatic disease
G. Perforated duodenal ulcer
H. Dissecting aortic aneurysm
I. Pancreatitis
J. Pyelonephritis
K. Spinal stenosis
L. Ruptured abdominal aortic aneurysm
M. Addison’s disease
N. Pericarditis
A

Ruptured AAA: This is a history of a ruptured AAA. There is abdominal (which the patient states as chest) pain radiating around to the back here and pallor due to blood loss suggesting this diagnosis. As this AAA has ruptured, this man will need urgent surgical repair, with of course standard resuscitation measures. Investigations would just waste time although it seems he has had a troponin, and likely ECG too. Misdiagnosing this condition is pretty poor of the doctors with the history here of pain which radiates to the back. The airway will needed to be managed with supplemental oxygen and ET intubation, a central venous catheter will need to be inserted, an arterial catheter and urinary catheter will also be needed for monitoring, and the target systolic BP is 50-70. Infusing too many fluids may increase the risk of death. The most effective form of surgical repair is an EVAR (endovascular AAA repair), anatomy permitting, otherwise traditional open repair is performed. Open repair has a mortality of 48%. Antibiotics will also be needed to cover bacteria to prevent graft infection. This will be prescribed in line with local protocols.

369
Q

A 59-year-old man is bought in by ambulance after collapsing. The A&E Sister fast-bleeps you because the blood pressures in both arms are significantly different.

A. Volvulus 
B. Mesenteric infarction
C. Myocardial infarction
D. Renal colic
E. Hepatitis
F. Metastatic disease
G. Perforated duodenal ulcer
H. Dissecting aortic aneurysm
I. Pancreatitis
J. Pyelonephritis
K. Spinal stenosis
L. Ruptured abdominal aortic aneurysm
M. Addison’s disease
N. Pericarditis
A

Dissecting Aortic Aneurysm: I don’t know why the A&E Sister has fast bleeped a medical student here but BP differential between the 2 arms is a hallmark feature of this condition. Pulse differences may also be present in the lower limbs. Aortic dissection, however, typically presents with tearing chest pain (but this isn’t something a collapsed patient is in a position to report). There may also be interscapular pain with dissection of the descending aorta. Dissecting aneurysms are either type A, which involves the ascending aorta, or type B. Type A dissections require urgent surgery whereas type B can be managed medically if it is not complicated by end organ ischaemia. There may also be the diastolic murmur of AR in proximal dissections. A CT scan is indicated as soon as a diagnosis of aortic dissection is suspected and should be from the chest to the pelvis to see the full extent of the dissecting aneurysm. What you will see is the intimal flap. MRI is more sensitive and specific but is more difficult to obtain acutely.
Of course, the investigations are irrelevant when you have an emergency case and a collapsed patient in front of you who needs immediate resuscitation measures starting with your ABC. Strong risk factors include hypertension, atherosclerotic aneurysmal disease, Marfan’s or Ehlers-Danlos, bicuspid aortic valve, coarctation, smoking, FH and annulo-aortic ectasia.

370
Q

An ambulance crew is dispatched to attend a 999 call made by passers-by. A 71-year-old diabetic lady at a bus stop collapsed, could not get up and was complaining of back pain. After basic investigations, she was taken to hospital. Subsequent CT showed no abnormalities.

A. Volvulus 
B. Mesenteric infarction
C. Myocardial infarction
D. Renal colic
E. Hepatitis
F. Metastatic disease
G. Perforated duodenal ulcer
H. Dissecting aortic aneurysm
I. Pancreatitis
J. Pyelonephritis
K. Spinal stenosis
L. Ruptured abdominal aortic aneurysm
M. Addison’s disease
N. Pericarditis
A

MI: Given the list of options here, this is likely to be an atypical MI which is more common in diabetics and the elderly, likely to be due to autonomic neuropathy. They are known as ‘silent’ MI and should be excluded in all causes of collapse. An ECG is indicated here after the clear CT scan (although may have already been done on admission by alert staff). STEMI, new LBBB or confirmed posterior MI is an indication for PCI/thrombolysis. It is worth noting that RV infarction is present in 40% of inferior infarcts so in this case, right sided ECG leads should also be obtained.

371
Q

A 91 year old man is referred to you by the urologists. He has an abdominal aortic aneurysm on examination and on ultrasound, an 8.8 cm infra-renal aneurysm is identified. He multiple co-morbidities but is given the green light for treatment to take place.

A. Endarterectomy
B. Methyldopa
C. Open repair of aneurysm
D. Angiography
E. Ultrasound
F. Alpha blocker
G. Embolectomy
H. Femoral-distal bypass
I. Aortobifemoral bypass
J. Angioplasty
K. Endovascular aneurysm repair
A

Endovascular aneurysm repair: Incidental finding of a large AAA requires elective surgical repair (exceeding 5.5cm in men, 5cm in women – repair of aneuryms greater or equal to 5.5cm offers a survival advantage). Additionally, rapid increase in size is also an indication for elective repair. Young and healthy patients, particularly women, may benefit from early repair of smaller AAAs. (>5cm). Data suggests EVAR is equivalent to open repair in terms of overall survival but there is a higher rate of secondary interventions with EVAR. Therefore younger and healthier patients may benefit more from open repair. Patient 1 is however is elderly and has co-morbidities. An EVAR is the best way forward here. However note also that EVAR could entail a complication of endovascular repair leak, which would require corrective treatment. Endoleak is persistent blood flow outside the graft and within the aneurysm sac. There is 24% risk after EVAR. However, this is not a complication of open repair, which is probably preferred in most cases in those who are fit and healthy enough to have it such as Patient 4. Management of this complication would depend on the type of endoleak.

372
Q

A 55-year-old lady describes 10 minutes yesterday when she was unable to see out of her left eye. The symptoms have resolved but on duplex scan, her internal carotid artery is 75% stenosed.

A. Endarterectomy
B. Methyldopa
C. Open repair of aneurysm
D. Angiography
E. Ultrasound
F. Alpha blocker
G. Embolectomy
H. Femoral-distal bypass
I. Aortobifemoral bypass
J. Angioplasty
K. Endovascular aneurysm repair
A

Endarterectomy: Amaurosis fugax is a transient and painless loss of vision in one eye due to the passage of an embolus into the central retinal artery. This temporary arrest of blood flow leads to vision loss. The cause could be embolic from the internal carotid artery to cause an occlusion of the ipsilateral retinal artery. Patients presenting in this way should be investigated for carotid artery stenosis with a carotid Doppler ultrasound and if there is a stenosis of >70%, the patient may be a candidate for carotid endarterectomy. Presence of ipsilateral carotid stenosis suggests artery-to-artery embolic event as the cause here and this should be the target for surgical or interventional treatment.

373
Q

A 68-year-old man has been scanned annually for 10 years. His abdominal aortic aneurysm last year was 4.9 cm in diameter. This year, the aneurysm is 5.0 cm in diameter.

A. Endarterectomy
B. Methyldopa
C. Open repair of aneurysm
D. Angiography
E. Ultrasound
F. Alpha blocker
G. Embolectomy
H. Femoral-distal bypass
I. Aortobifemoral bypass
J. Angioplasty
K. Endovascular aneurysm repair
A

Ultrasound: For asymptomatic small AAA, surveillance is indicated. Infra and juxtarenal AAAs between 4.0-5.4cm in diameter (bear in mind that young and healthy patients with >5cm may benefit from repair!) should be monitored by USS or CT every 6-12 months. There is good quality evidence that the risk of rupture is 20% for aneurysms larger than 5.0-6.0cm in diameter. Note also that AAAs

374
Q

A 52 year old otherwise fit and healthy man is found to have a 6.3cm AAA. He is very surprised and requests that it is treated so he does not die suddenly in the future.

A. Endarterectomy
B. Methyldopa
C. Open repair of aneurysm
D. Angiography
E. Ultrasound
F. Alpha blocker
G. Embolectomy
H. Femoral-distal bypass
I. Aortobifemoral bypass
J. Angioplasty
K. Endovascular aneurysm repair
A

Open repair of aneurysm: Incidental finding of a large AAA requires elective surgical repair (exceeding 5.5cm in men, 5cm in women – repair of aneuryms greater or equal to 5.5cm offers a survival advantage). Additionally, rapid increase in size is also an indication for elective repair. Young and healthy patients, particularly women, may benefit from early repair of smaller AAAs. (>5cm). Data suggests EVAR is equivalent to open repair in terms of overall survival but there is a higher rate of secondary interventions with EVAR. Therefore younger and healthier patients may benefit more from open repair. Patient 1 is however is elderly and has co-morbidities. An EVAR is the best way forward here. However note also that EVAR could entail a complication of endovascular repair leak, which would require corrective treatment. Endoleak is persistent blood flow outside the graft and within the aneurysm sac. There is 24% risk after EVAR. However, this is not a complication of open repair, which is probably preferred in most cases in those who are fit and healthy enough to have it such as Patient 4. Management of this complication would depend on the type of endoleak.

375
Q

A 45-year-old man had an arteriovenous fistula formed yesterday for dialysis access. A Doppler scan shows that this is not working today

A. Endarterectomy
B. Methyldopa
C. Open repair of aneurysm
D. Angiography
E. Ultrasound
F. Alpha blocker
G. Embolectomy
H. Femoral-distal bypass
I. Aortobifemoral bypass
J. Angioplasty
K. Endovascular aneurysm repair
A

Angioplasty: Stenosis and thrombosis of AV fistulae may lead to loss of vascular access sites which presents a problem for chronic haemodialysis patients. Percutaneous transluminal angioplasty is a commonly employed technique used to correct the stenosis and restore viability. All patients who are having haemodialysis should be educated about vein preservation with limiting venepuncture and IV access in the access arm.

376
Q

A 62-year-old lady has been having difficulties walking for some years. A scan this year shows occlusion of the popliteal artery in her left leg. Previous angioplasties have failed.

A. Endarterectomy
B. Methyldopa
C. Open repair of aneurysm
D. Angiography
E. Ultrasound
F. Alpha blocker
G. Embolectomy
H. Femoral-distal bypass
I. Aortobifemoral bypass
J. Angioplasty
K. Endovascular aneurysm repair
A

Femoral-distal by pass: Previous angioplasties have failed so there is clearly not point in having another endovascular intervention such as PTA (percutaneous transluminal angioplasty) with balloon dilation, stents, laser, atherectomy or thermal angioplasty. The occlusion here is at the level of the popliteal artery so a femoral-distal bypass is indicated. Surgical revascularisation is recommended for lesions involving the popliteal artery. Make sure you know your vascular anatomy! (At least the big vessels).